SlideShare a Scribd company logo
1 of 121
© Sandeep Gupta – Ivy-GMAT, Koramangala Bangalore. sandeepgupta01@gmail.com, 9739561394
Ultimate CR
Strategies for
the GMAT
by Sandeep Gupta (Ivy-GMAT,
Koramangala Bangalore)
sandeepgupta01@gmail.com
9739561394
© Sandeep Gupta – Ivy-GMAT, Koramangala Bangalore. sandeepgupta01@gmail.com, 9739561394
Further Resources:
1. Manhattan GMAT CR Guide – 4th
Edition
2. CR Ultimate Collection – a set of GMAT CR Questions (by
Sandeep Gupta (Ivy-GMAT)) – Must Solve
3. 700-800 CR Questions – a set of GMAT CR Questions (by
Sandeep Gupta (Ivy-GMAT)) – Must Solve
4. OG 12, 11, 10, OG Verbal Review 1st
and 2nd
Editions
5. GMAT-Prep Verbal Forum Questions from ManhattanGMAT.com
6. 1000 CR (not necessary, though)
© Sandeep Gupta – Ivy-GMAT, Koramangala Bangalore. sandeepgupta01@gmail.com, 9739561394
CR Strategies
Note:
Refer to the NOTES at the bottom of each slide for
explanations.
© Sandeep Gupta – Ivy-GMAT, Koramangala Bangalore. sandeepgupta01@gmail.com, 9739561394
CR Questions… the structure
• A short passage (called the argument) followed by a specific
question related to the passage with 5 answer choices
• Directions: For this question type, select the best of the given
answer choices. For example:
Most serious students are happy students, and most
serious students go to graduate school. Furthermore,
all students who go to graduate school are
overworked.
Which one of the following can be properly inferred from the
statements above?
(A) Most overworked students are happy students.
(B) Some happy students are overworked.
(C) All overworked students are serious students.
(D) Some unhappy students go to graduate school.
(E) All serious students are overworked.
© Sandeep Gupta – Ivy-GMAT, Koramangala Bangalore. sandeepgupta01@gmail.com, 9739561394
The Nitty-gritty
• CR is largely composed of arguments and arguments parts
• It checks skills of Logical Thinking and Critical Evaluation. One
needs eye for detail.
• There are 13-14 questions interspersed throughout the Verbal
Section
• It is possibly the best tool to check Managerial Aptitude
© Sandeep Gupta – Ivy-GMAT, Koramangala Bangalore. sandeepgupta01@gmail.com, 9739561394
Argument Parts
• An argument (passage) generally consists of
– FACT (S) & CONCLUSION (s)
– Facts support the conclusion: also called Assertion, Evidence,
Premise, Proof, Supposition, Data, Information, Research,
Consideration, Pattern, Explanation, Justification, Situation,
Circumstance, Reasoning, Line of Reasoning, Generalization,
Observation, Support, Example, Finding, Phenomenon, Illustration,
Reason, To Acknowledge, etc.
– Conclusion is what the author tries to establish using facts. Also
known as Judgment, Opinion, Suggestion, View, Idea, Belief, Proposal,
Warning, Forecast, Claim, Stand, Prediction, Hypothesis, POSITION,
Stance, Point, Main Point, Inference, Implication, Generalization,
Contention, Consequence, Argument, To Posit, etc.
© Sandeep Gupta – Ivy-GMAT, Koramangala Bangalore. sandeepgupta01@gmail.com, 9739561394
So, finding the correct conclusion is
the half the job done in CR. If we
can identify the conclusion,
everything else becomes FACT
automatically.
© Sandeep Gupta – Ivy-GMAT, Koramangala Bangalore. sandeepgupta01@gmail.com, 9739561394
1.In a study, infant monkeys given a choice between two
surrogate mothers—a bare wire structure equipped
with a milk bottle, or a soft, suede-covered wire
structure equipped with a milk bottle—unhesitatingly
chose the latter. When given a choice between a bare
wire structure equipped with a milk bottle and a soft,
suede-covered wire structure lacking a milk bottle,
they unhesitatingly chose the former.
Find the conclusion
© Sandeep Gupta – Ivy-GMAT, Koramangala Bangalore. sandeepgupta01@gmail.com, 9739561394
2. Journalist: A free marketplace of ideas ensures that all
ideas get a fair hearing. Even ideas tainted with prejudice
and malice can prompt beneficial outcomes. In most
countries, however, the government is responsible for over
half the information released to the public through all media.
For this reason, the power of governments over information
needs to be curtailed. Everyone grants that governments
should not suppress free expression, yet governments
continue to construct near monopolies on the publication
and dissemination of enormous amounts of information.
Find the conclusion
© Sandeep Gupta – Ivy-GMAT, Koramangala Bangalore. sandeepgupta01@gmail.com, 9739561394
3. Last month OCF, Inc., announced what it described as a
unique new product: an adjustable computer workstation.
Three days later ErgoTech unveiled an almost identical
product. The two companies claim that the similarities are
coincidental and occurred because the designers
independently reached the same solution to the same
problem. The similarities are too fundamental to be mere
coincidence, however. The two products not only look alike,
but they also work alike. Both are oddly shaped with
identically placed control panels with the same types of
controls. Both allow the same types of adjustments and the
same types of optional enhancements.
Find the conclusion
© Sandeep Gupta – Ivy-GMAT, Koramangala Bangalore. sandeepgupta01@gmail.com, 9739561394
4. It is well known that many species adapt to their
environment, but it is usually assumed that only the most
highly evolved species alter their environment in ways that
aid their own survival. However, this characteristic is actually
quite common. Certain species of plankton, for example,
generate a gas that is converted in the atmosphere into
particles of sulfate. These particles cause water vapor to
condense, thus forming clouds. Indeed, the formation of
clouds over the ocean largely depends on the presence of
these particles. More cloud cover means more sunlight is
reflected, and so the Earth absorbs less heat. Thus plankton
cause the surface of the Earth to be cooler and this benefits
the plankton.
Find the conclusion (s)
© Sandeep Gupta – Ivy-GMAT, Koramangala Bangalore. sandeepgupta01@gmail.com, 9739561394
5. Rain-soaked soil contains less oxygen than does drier soil. The roots of melon
plants perform less efficiently under the low-oxygen conditions present in
rain-soaked soil. When the efficiency of melon roots is impaired, the roots do not
supply sufficient amounts of the proper nutrients for the plants to perform
photosynthesis at their usual levels. It follows that melon plants have a lower-
than-usual rate of photosynthesis when their roots are in rain-soaked
soil. When the photosynthesis of the plants slows, sugar stored in the fruits is
drawn off to supply the plants with energy. Therefore, ripe melons harvested after a
prolonged period of heavy rain should be less sweet than other ripe melons.
In the argument given, the two portions in boldface play which of the following
roles?
• The first states the conclusion of the argument as a whole; the second provides
support for that conclusion.
• The first provides support for the conclusion of the argument as a whole; the
second provides evidence that supports an objection to that conclusion.
• The first provides support for an intermediate conclusion that supports a further
conclusion stated in the argument; the second states that intermediate conclusion.
• The first serves as an intermediate conclusion that supports a further conclusion
stated in the argument; the second states the position that the argument as a whole
opposes.
• The first states the position that the argument as a whole opposes; the second
supports the conclusion of the argument.
© Sandeep Gupta – Ivy-GMAT, Koramangala Bangalore. sandeepgupta01@gmail.com, 9739561394
CR – types of Questions
• Conclusion / Inference
• Assumptions
• Weaken
• Strengthen
• Evaluate
• Paradox
• Boldface
• Miscellaneous
© Sandeep Gupta – Ivy-GMAT, Koramangala Bangalore. sandeepgupta01@gmail.com, 9739561394
Type 1: Conclusion / Inference
1. Which of the following conclusions / assertions /
inferences / statements is most strongly supported by
the passage?
2. Which of the following must be true as per the
passage?
3. Which of the following can be inferred from the
passage?
4. The author is arguing that…
5. The main point of the passage is that
6. Which of the following can complete the passage?
7. Which of the following is an example of the … given in
the passage?
© Sandeep Gupta – Ivy-GMAT, Koramangala Bangalore. sandeepgupta01@gmail.com, 9739561394
How to solve “conclusion” questions?
• Word Matching and TRANSLATION are the most
critical skills.
• Any statement that is inconsistent with the passage is
always wrong.
• The correct answer MUST be true, not MAY be true.
• Elimination is the best strategy. Don’t select the
correct answer. Eliminate wrong ones.
© Sandeep Gupta – Ivy-GMAT, Koramangala Bangalore. sandeepgupta01@gmail.com, 9739561394
Choose the correct conclusion or conclusions
(any number of answers may be right)
The minimum voting age for males in
district X is 18 years.
A. Every male above 18 can vote in district X.
B. Someone who is not 18 can not vote in district X.
C. A male who is not 18 can not vote in district X.
D. For a male to be eligible to vote in district X, he
must be 18 years old.
E. For a male to be eligible to vote in district X, he
must be at least 18 years old.
© Sandeep Gupta – Ivy-GMAT, Koramangala Bangalore. sandeepgupta01@gmail.com, 9739561394
Choose the correct conclusion or conclusions
(any number of answers may be right)
Every male above 18 is allowed to vote in
county X.
A. A male who is not 18 can not vote in county X.
B. For a male to be eligible to vote in county X, he
must be more than 18 years old.
C. For a male to be eligible to vote in county X, he
must be at least 18 years old.
D. Males below 18 are not allowed to vote in county X.
E. Age is the only criterion by which the eligibility to
vote for males is decided in county X.
© Sandeep Gupta – Ivy-GMAT, Koramangala Bangalore. sandeepgupta01@gmail.com, 9739561394
Understanding Conditional “If-then”
Consider the following statement:
If one plays in the rain, one gets cold.
Now consider which one follows logically from
above:
1. If one doesn’t play in the rain, one doesn’t get
cold.
2. If one has got cold, one has played in the rain.
3. If one hasn’t got cold, one hasn’t played in the
rain.
© Sandeep Gupta – Ivy-GMAT, Koramangala Bangalore. sandeepgupta01@gmail.com, 9739561394
Understanding Conditional “If-then”
So if X → Y then the following conclusions are
wrong:
1. Y → X
2. X (NOT) → Y (NOT)
Only the following conclusion is right:
Y (not) → X (not)
© Sandeep Gupta – Ivy-GMAT, Koramangala Bangalore. sandeepgupta01@gmail.com, 9739561394
1. Mystery stories often feature a brilliant detective and the detective’s dull
companion. Clues are presented in the story, and the companion wrongly
infers an inaccurate solution to the mystery using the same clues that the
detective uses to deduce the correct solution. Thus, the author’s strategy of
including the dull companion gives readers a chance to solve the mystery
while also diverting them from the correct solution.
Which of the following conclusions can be correctly drawn from the
passage above?
A. Most mystery stories feature a brilliant detective who solves the mystery
presented in the story.
B. Mystery readers often solve the mystery in a story simply by spotting the
mistakes in the reasoning of the detective’s dull companion in that story.
C. Some mystery stories give readers enough clues to infer the correct solution
to the mystery.
D. The actions of the brilliant detective in a mystery story rarely divert readers
from the actions of the detective’s dull companion.
E. The detective’s dull companion in a mystery story generally uncovers the
misleading clues that divert readers from the mystery’s correct solution.
© Sandeep Gupta – Ivy-GMAT, Koramangala Bangalore. sandeepgupta01@gmail.com, 9739561394
Some Modifier Words
• All = 100
• Some = 1 to 99 or even 100
• Most = 51 to 99 or even 100
• Not all = 0 to 99
• None = 0
• Often = Not defined
• Many = Not defined
• Much = Not defined
© Sandeep Gupta – Ivy-GMAT, Koramangala Bangalore. sandeepgupta01@gmail.com, 9739561394
2. Newtonian physics dominated science for over two centuries. It found
consistently successful application, becoming one of the most highly
substantiated and accepted theories in the history of science.
Nevertheless, Einstein’s theories came to show the fundamental limits
of Newtonian physics and to surpass the Newtonian view in the early
1900s, giving rise once again to a physics that has so far enjoyed wide
success.
Which one of the following logically follows from the statements
above?
A. The history of physics is characterized by a pattern of one successful
theory subsequently surpassed by another.
B. Long-standing success or substantiation of a theory of physics is no
guarantee that the theory will continue to be dominant indefinitely.
C. Every theory of physics, no matter how successful, is eventually
surpassed by one that is more successful.
D. Once a theory of physics is accepted, it will remain dominant for
centuries.
E. If a long-accepted theory of physics is surpassed, it must be surpassed
by a theory that is equally successful.
© Sandeep Gupta – Ivy-GMAT, Koramangala Bangalore. sandeepgupta01@gmail.com, 9739561394
3. People with serious financial problems are so worried about
money that they cannot be happy. Their misery makes
everyone close to them—family, friends, and colleagues—
unhappy as well. Only if their financial problems are solved
can they and those around them be happy.
Which one of the following statements can be properly
inferred from the passage?
A. Only serious problems make people unhappy.
B. People who solve their serious financial problems will be
happy.
C.People who do not have serious financial problems will be
happy.
D.If people are unhappy, they have serious financial problems.
E. If people are happy, they do not have serious financial
problems.
© Sandeep Gupta – Ivy-GMAT, Koramangala Bangalore. sandeepgupta01@gmail.com, 9739561394
4. The head baker at Barry’s Bagels can either purchase flour in-person from the
local flour mill, Larry’s Local Mill, or order a shipment of flour from an out-of-state
mill, Isadore’s Interstate Mill. The cost of the flour from Isadore’s Interstate Mill is
10 percent less than the cost of the flour from Larry’s Local Mill. Even after
shipping and handling fees are added, it is still cheaper to order flour that has to
be shipped from Isadore’s than to buy flour locally from Larry’s.
The statements above, if true, best support which of the following
assertions?
A. Production costs at Isadore’s Interstate Mill are 10 percent below those at Larry’s
Local Mill.
B. Buying flour from Isadore’s Interstate Mill will eliminate 10 percent of the local
flour mill jobs.
C. The shipping and handling fees for a batch of flour purchased from Isadore’s
Interstate Mill are less than 10 percent of the cost of an identical batch of flour
purchased from Larry’s Local Mill.
D. The shipping and handling fees for a batch of flour purchased from Isadore’s
Interstate Mill are more than 10 percent of the cost of Isadore’s flour.
E. Isadore’s Interstate Mill produces flour 10% more efficiently than Larry’s Local
Mill does.
© Sandeep Gupta – Ivy-GMAT, Koramangala Bangalore. sandeepgupta01@gmail.com, 9739561394
5. Last January, in an attempt to lower the number of traffic fatalities, the
state legislature passed its “Click It or Ticket” law. Under the new law,
motorists can be pulled over and ticketed for not wearing their seat
belts, even if an additional driving infraction has not been committed.
Lawyers and citizens’ groups are already protesting the law, saying it
unfairly infringes on the rights of the state’s drivers. Law enforcement
groups counter these claims by stating that the new regulations will save
countless additional lives.
Which of the following inferences is best supported by the passage
above?
A. Prior to the “Click It or Ticket” law, motorists could not be stopped simply
for not wearing a seat belt.
B. The “Click It or Ticket” law violates current search and seizure laws.
C. Laws similar to “Click It or Ticket” have effectively reduced traffic
fatalities in a number of states.
D. The previous seatbelt laws were ineffective in saving lives.
E. Law enforcement groups, rather than citizens groups, should determine
how to best ensure the safety of motorists.
© Sandeep Gupta – Ivy-GMAT, Koramangala Bangalore. sandeepgupta01@gmail.com, 9739561394
6. Meteorite explosions in the Earth’s atmosphere as large as the one that
destroyed forests in Siberia, with approximately the force of a twelve-
megaton nuclear blast, occur about once a century. The response of
highly automated systems controlled by complex computer programs to
unexpected circumstances is unpredictable.
Which of the following conclusions can most properly be drawn, if
the statements above are true, about a highly automated nuclear-
missile defense system controlled by a complex computer
program?
A. Within a century after its construction, the system would react
inappropriately and might accidentally start a nuclear war.
B. The system would be destroyed if an explosion of a large meteorite
occurred in the Earth’s atmosphere.
C. It would be impossible for the system to distinguish the explosion of a
large meteorite from the explosion of a nuclear weapon.
D. Whether the system would respond inappropriately to the explosion of a
large meteorite would depend on the location of the blast.
E. It is not certain what the system’s response to the explosion of a large
meteorite would be, if its designers did not plan for such a contingency.
© Sandeep Gupta – Ivy-GMAT, Koramangala Bangalore. sandeepgupta01@gmail.com, 9739561394
7. Laws requiring the use of headlights during daylight hours can prevent automobile
collisions. However, since daylight visibility is worse in countries farther from the
equator, any such laws would obviously be more effective in preventing collisions in
those countries. In fact, the only countries that actually have such laws are farther
from the equator than is the continental United States.
Which of the following conclusions could be most properly drawn from the
information given above?
A. Drivers in the continental United States who used their headlines during the day
would be just as likely to become involved in a collision as would drivers who did
not use their headlights.
B. In many countries that are farther from the equator than is the continental United
States poor daylight visibility is the single most important factor in automobile
collisions.
C. The proportion of automobile collisions that occur in the daytime is greater in the
continental United States than in the countries that have daytime headlight laws.
D. Fewer automobile collisions probably occur each year in countries that have
daytime headlight laws than occur within the continental United States.
E. Daytime headlight laws would probably do less to prevent automobile collisions in
the continental United States than they do in the countries that have the laws.
© Sandeep Gupta – Ivy-GMAT, Koramangala Bangalore. sandeepgupta01@gmail.com, 9739561394
8. High levels of fertilizer and pesticides, needed when farmers try to
produce high yield of the same crop year after year, pollute water supplies.
Experts therefore urge farmers to diversify their crops and to rotate their
plantings yearly. To receive governmental price-support benefits for a crop,
farmers must have produced that same crop for the past several years.
The statements above, if true, best support which of the following
conclusions?
A. The rules for governmental support of farm prices work against efforts to
reduce water pollution.
B. The only solution to the problem of water pollution from fertilizers and
pesticides is to take farmland out of production.
C. Farmers can continue to make a profit by rotating diverse crops, thus
reducing costs for chemicals, but not by planting the same crop each year.
D. New farming techniques will be developed to make it possible for farmers
to reduce the application of fertilizers and pesticides.
E. Governmental price supports for farm products are set at levels that are
not high enough to allow farmers to get out of debt.
© Sandeep Gupta – Ivy-GMAT, Koramangala Bangalore. sandeepgupta01@gmail.com, 9739561394
9. United States hospitals have traditionally relied primarily on revenues from paying
patients to offset losses from unreimbursed care. Almost all paying patients now
rely on governmental or private health insurance to pay hospital bills. Recently,
insurers have been strictly limiting what they pay hospitals for the care of insured
patients to amounts at or below actual costs.
Which of the following conclusions is best supported by the information
above?
A. Although the advance of technology has made expensive medical procedures
available to the wealthy, such procedures are out of the reach of low-income
patients.
B. If hospitals do not find ways to raising additional income for unreimbursed care,
they must either deny some of that care of suffer losses if they give it.
C. Some patients have incomes too high for eligibility for governmental health
insurance but are unable to afford private insurance for hospital care.
D. If the hospitals reduce their costs in providing care, insurance companies will
maintain the current level of reimbursement, thereby providing more funds for
unreimbursed care.
E. Even though philanthropic donations have traditionally provided some support for
the hospitals, such donations are at present declining.
© Sandeep Gupta – Ivy-GMAT, Koramangala Bangalore. sandeepgupta01@gmail.com, 9739561394
10. Which of the following best completes the passage below?
In a survey of job applicants, two-fifths admitted to being at
least a little dishonest. However, the survey may
underestimate the proportion of job applicants who are
dishonest, because____.
A. some dishonest people taking the survey might have
claimed on the survey to be honest
B. some generally honest people taking the survey might have
claimed on the survey to be dishonest
C. some people who claimed on the survey to be at least a little
dishonest may be very dishonest
D. some people who claimed on the survey to be dishonest
may have been answering honestly
E. some people who are not job applicants are probably at
least a little dishonest
© Sandeep Gupta – Ivy-GMAT, Koramangala Bangalore. sandeepgupta01@gmail.com, 9739561394
CR Type 2: Boldface Questions
• Structure:
– Usually in a long paragraph, two statements (or
statement parts) are written in BOLD, and one has to
choose the option that tells the function (role) played
by these bold parts.
– Involves Argument structure concepts – one should
be able to tell which is a fact, conclusion and how the
argument is structured.
© Sandeep Gupta – Ivy-GMAT, Koramangala Bangalore. sandeepgupta01@gmail.com, 9739561394
– One must know all the terms that are commonly used for FACT
and CONCLUSION.
– Facts support the conclusion: also called Assertion, Evidence,
Premise, Proof, Supposition, Data, Information, Research,
Consideration, Pattern, Explanation, Justification, Situation,
Circumstance, Reasoning, Line of Reasoning, Generalization,
Observation, Support, Example, Finding, Phenomenon,
Illustration, Reason, To Acknowledge, etc.
– Conclusion is what the author tries to establish using facts.
Also known as Judgment, Opinion, View, Suggestion, Idea,
Belief, Proposal, Warning, Forecast, Claim, Stand, Prediction,
Hypothesis, POSITION, Stance, Point, Main Point, Inference,
Implication, Generalization, Contention, Consequence,
Argument, To Posit, etc.
© Sandeep Gupta – Ivy-GMAT, Koramangala Bangalore. sandeepgupta01@gmail.com, 9739561394
• Sometimes the argument may contain two conclusions.
• The statement that can be used to prove the main
conclusion is usually the sub-conclusion.
• In such a situation, the ONE LINE CRISP MESSAGE of
the argument is the MAIN conclusion and the other is the
sub-conclusion. Asking yourself about author’s
position will help you get the main conclusion.
• Sometimes, the two conclusions may be contradictory to
each other (especially when two sides are involved –
author and the critics etc.)
© Sandeep Gupta – Ivy-GMAT, Koramangala Bangalore. sandeepgupta01@gmail.com, 9739561394
Steps to solve BF questions
– STEP 1: Identify the conclusion (s). Do not jump to
choices.
– STEP 2: Map the argument (F/F/C… etc.) and have
the flow in mind. Which statement supports the
conclusion and which supports some alternative
conclusion.
– STEP 3: Go to choices and eliminate.
© Sandeep Gupta – Ivy-GMAT, Koramangala Bangalore. sandeepgupta01@gmail.com, 9739561394
1. Astronomer: Observations of the Shoemaker-Levi comet on its collision course with
Jupiter showed that the comet broke into fragments before entering Jupiter’s atmosphere
in 1994, but they did not show how big those fragments were. Nevertheless, some
indication of their size can be inferred from spectrographic analyses of Jupiter’s outer
atmosphere. After the fragments’ entry, these analyses revealed unprecedented
traces of sulfur. The fragments themselves almost certainly contained no sulfur, but
astronomers believe that the cloud layer below Jupiter’s outer atmosphere does contain
sulfur. Since sulfur would have seeped into the outer atmosphere if comet fragments
had penetrated this cloud layer, it is likely that some of the fragments were at least large
enough to have passed through Jupiter’s outer atmosphere without being burned up.
In the astronomer’s argument, the two portions in BOLDFACE play which of the
following roles?
A. The first is a claim that the astronomer seeks to show is true; the second acknowledges a
consideration that weighs against the truth of that claim.
B. The first is a claim that the astronomer seeks to show is true; the second provides
evidence in support of the truth of that claim.
C. The first and the second are each considerations advanced in support of the conclusion of
the argument.
D. The first provides evidence in support of the conclusion of the argument; the second is that
conclusion.
E. The first is a circumstance for which the astronomer seeks to provide an explanation; the
second acknowledges a consideration that weighs against the explanation provided by the
astronomer.
© Sandeep Gupta – Ivy-GMAT, Koramangala Bangalore. sandeepgupta01@gmail.com, 9739561394
2. Economist: Tropicorp, which constantly seeks profitable investment
opportunities, has been buying and clearing sections of tropical forest for cattle
ranching, although pastures newly created there become useless for grazing after
just a few years. The company has not gone into rubber tapping, even though
greater profits can be made from rubber tapping, which leaves the forest intact.
Thus, some environmentalists conclude that Tropicorp has not acted wholly
out of economic self-interest. However, these environmentalists are probably
wrong. The initial investment required for a successful rubber-tapping operation is
larger than that needed for a cattle ranch. Furthermore, there is a shortage of
workers employable in rubber-tapping operations, and finally, taxes are higher on
profits from rubber tapping than on profits from cattle ranching. In the
economist’s argument, the two BOLDFACED portions play which of the
following roles?
A. The first supports the conclusion of the economist’s argument; the second calls
that conclusion into question.
B. The first states the conclusion of the economist’s argument; the second supports
that conclusion.
C. The first supports the environmentalists’ conclusion; the second states that
conclusion.
D. The first states the environmentalists’ conclusion; the second states the conclusion
of the economist’s argument.
E. Each supports the conclusion of the economist’s argument.
© Sandeep Gupta – Ivy-GMAT, Koramangala Bangalore. sandeepgupta01@gmail.com, 9739561394
3. Local authorities are considering an amendment to the litter law that would
raise the fine for littering in the community picnic area to $1,000. Since the
inception of the litter law, incremental increases in the littering fine have
proven to be consistently effective at further reducing the amount of
litter in the community picnic area. However, raising the fine to $1,000
would actually have the unintended effect of increasing the amount of litter in
the picnic area. Picnic area users would perceive this fine to be
unreasonable and unenforceable, and would disregard the litter law
altogether.
In the argument, the two portions in BOLDFACE play which of the
following roles?
A. The first is irrefutable evidence that the author offers in support of a prediction;
the second is that prediction.
B. The first is a statement of causation that the author predicts will be repeated in
the case at hand; the second raises evidence against this prediction.
C. The first is a statement of fact that the author accepts to be true; the second is
presented as a consequence of this fact.
D. The first is evidence that weakens the main position that the author defends;
the second is that position.
E. The first is a statement of causation that the author predicts will not hold in the
case at hand; the second offers a line of reasoning to support this prediction.
© Sandeep Gupta – Ivy-GMAT, Koramangala Bangalore. sandeepgupta01@gmail.com, 9739561394
4. Consumer advocate: it is generally true, at least in this state, that lawyers who advertise
a specific service charge less for that service than lawyers who do not advertise. It is
also true that each time restrictions on the advertising of legal services have been
eliminated, the number of lawyers advertising their services has increased and
legal costs to consumers have declined in consequence. However, eliminating the
state requirement that legal advertisements must specify fees for specific services
would almost certainly increase rather than further reduce consumer’s legal costs.
Lawyers would no longer have an incentive to lower their fees when they begin
advertising and if no longer required to specify fee arrangements, many lawyers
who now advertise would increase their fees.
In the consumer advocate’s argument, the two portions in boldface play which of the
following roles?
A. The first is a generalization that the consumer advocate accepts as true; the second is
presented as a consequence that follows from the truth of that generalization.
B. The first is a pattern of cause and effect that the consumer advocate argues will be
repeated in the case at issue; the second acknowledges a circumstance in which that
pattern would not hold.
C. The first is pattern of cause and effect that the consumer advocate predicts will not hold
in the case at issue; the second offers a consideration in support of that prediction.
D. The first is evidence that the consumer advocate offers in support of a certain
prediction; the second is that prediction.
E. The first acknowledges a consideration that weighs against the main position that the
consumer advocate defends; the second is that position.
© Sandeep Gupta – Ivy-GMAT, Koramangala Bangalore. sandeepgupta01@gmail.com, 9739561394
5. Traditionally, video game manufacturers have been most strongly influenced
by serious video gamers. Because devoted gamers have historically
purchased the majority of video games, companies react to the desires of this
market segment. Normally, devoted gamers crave speed and action; thus,
most manufacturers continue to produce games with faster chips and
flashier graphics. Unfortunately, faster chips and flashier graphics are no
longer in the industry’s best interest. The devoted gaming market is deeply
stagnant, and it won’t soon expand. To infuse new life into the video game
market, manufacturers must simplify the functionality of their games. By doing
so, current non-gamers will be attracted to join the ranks of video game fans.
In the argument, the two portions in boldface play which of the following roles?
A. The first is a situation that the author believes to be true; the second offers
evidence to explain this situation.
B. The first is a situation that the author argues should not continue; the second
provides evidence that supports the author’s position.
C. The first is a statement of fact that contradicts the author’s position; the
second is the author’s position.
D. The first is a statement of fact that supports the author’s position; the second
is a consideration that weighs against the author’s position.
E. The first is a prediction that the author believes should not hold in this case;
the second is an assumption that weighs against the author’s position.
© Sandeep Gupta – Ivy-GMAT, Koramangala Bangalore. sandeepgupta01@gmail.com, 9739561394
CR Type 3: Assumption Questions
• Assumptions are unstated facts that must be true if the
conclusion is true…
• Approach: The Assumption Formula (GAP FILL)
Facts + Assumptions = Conclusion.
• So the steps are:
– ID the conclusion (every other statement becomes FACT)
– Try to place answer choices between the fact (s) and the
conclusion. The choice that makes a flowing, sound and
convincing argument is the assumption.
© Sandeep Gupta – Ivy-GMAT, Koramangala Bangalore. sandeepgupta01@gmail.com, 9739561394
Assumption as GAP Fills
For example:
1. Amy is less than 5’6” tall. Therefore she can not be a
successful model.
1. Every male above 18 is allowed to vote in city X. Therefore,
Jack must be allowed to vote in city X.
2. Company owner to manager: “Give this man this job. He will
remain jobless otherwise.”
3. Company X doesn’t pay very high salaries to inexperienced
people. Therefore John’s salary should be lowered.
© Sandeep Gupta – Ivy-GMAT, Koramangala Bangalore. sandeepgupta01@gmail.com, 9739561394
• Assumption Questions will use one of
these words:
– Assumption / Assumed / Assumes
– Presupposition / Presupposed / Presupposes
– Justify the conclusion
– Inserted as an additional premise
– The conclusion can not be true unless which
of the following is true
– The conclusion will be more properly drawn
if…
© Sandeep Gupta – Ivy-GMAT, Koramangala Bangalore. sandeepgupta01@gmail.com, 9739561394
SMART TIPS
1. Something that is not mentioned in the facts but
mentioned in the conclusion has to be mentioned in the
assumption. Also, a connection (BRIDGE/LINK) between
X and Y should occur in the assumption.
2. Something that is mentioned both in the facts and in the
conclusion will usually not figure in the assumption.
© Sandeep Gupta – Ivy-GMAT, Koramangala Bangalore. sandeepgupta01@gmail.com, 9739561394
Assumptions Type 2 : The only (or the best) way
• If the conclusion says: the only way (or the best
way) to achieve X is Y, the following is a valid
assumption:
– There is no other (or no better) way to achieve X than
Y.
5.Girl Power magazine published an article
proclaiming that one can lose up to 20 pounds a
month by eating only soup. Kelly concludes that
the only way for her to lose 40 pounds in two
months is to eat only soup for 2 months.
© Sandeep Gupta – Ivy-GMAT, Koramangala Bangalore. sandeepgupta01@gmail.com, 9739561394
Assumptions Type 3: Cause and Effect
• If the conclusion of an argument is A causes B,
the following are some valid assumptions on the
GMAT:
– B does not cause A
– C does not cause B
• For example:
6.Researchers in the field have noticed that older
antelope are more cautious. Therefore, they have
concluded that the quality of caution increases with
age in antelope.
© Sandeep Gupta – Ivy-GMAT, Koramangala Bangalore. sandeepgupta01@gmail.com, 9739561394
Type 4 Assumption: Negate and Weaken
An argument can be analogized to a house:
• The FACTS are like the walls
• The CONCLUSION is like the roof
• The ASSUMPTIONS are like the foundation
– As with a house foundation, an assumption is a hidden part of the
structure, but critical to the integrity of the structure—all the other
elements rest upon it.
• If the conclusion is valid, then the assumption must be true.
• So if the assumption is negated, the argument falls apart.
© Sandeep Gupta – Ivy-GMAT, Koramangala Bangalore. sandeepgupta01@gmail.com, 9739561394
METHOD:
• ID the conclusion
• Logically negate the choices
• The negated choice MUST UNDERMINE the conclusion
Look at this example:
7. His get-rich-quick scheme is simple: he will use a metal
detector to find hidden treasures in the sand. Then he
will sell the treasures to a local pawn broker.
© Sandeep Gupta – Ivy-GMAT, Koramangala Bangalore. sandeepgupta01@gmail.com, 9739561394
1. If something would have been justifiably regretted if it had occurred, then it is
something that one should not have desired in the first place.
Many forgone pleasures would have been justifiably regretted.
It follows that many forgone pleasures should not have been desired in the
first place.
The conclusion above follows logically if which one of the following is
assumed?
A. One should never regret one’s pleasures.
B. Forgone pleasures that were not desired would not have been justifiably
regretted.
C. Everything that one desires and then regrets not having is a forgone pleasure.
D. Many forgone pleasures would have been justifiably regretted.
E. Nothing that one should not have desired in the first place fails to be a
pleasure.
© Sandeep Gupta – Ivy-GMAT, Koramangala Bangalore. sandeepgupta01@gmail.com, 9739561394
2. Psychiatrist: Take any visceral emotion you care to
consider. There are always situations in which it is healthy
to try to express that emotion.
Anger is a visceral emotion.
So, there are always situations in which it is healthy to try
to express one’s anger.
The conclusion of the argument follows logically if
which one of the following is assumed?
A. Anger is always expressible.
B. Anger is a visceral emotion.
C. Some kinds of emotions are unhealthy to express.
D. All emotions that are healthy to express are visceral.
E. An emotion is visceral only if it is healthy to express.
© Sandeep Gupta – Ivy-GMAT, Koramangala Bangalore. sandeepgupta01@gmail.com, 9739561394
3. To prevent some conflicts of interest, Congress could prohibit high-
level government officials from accepting positions as lobbyists for
three years after such officials leave government service. One such
official concluded, however, that such a prohibition would be
unfortunate because it would prevent high-level government officials
from earning a livelihood for three years.
The official’s conclusion logically depends on which of the
following assumptions?
A. Laws should not restrict the behavior of former government officials.
B. Lobbyists are typically people who have previously been high-level
government officials.
C. Low-level government officials do not often become lobbyists when
they leave government service.
D. High-level government officials who leave government service are
capable of earning a livelihood only as lobbyists.
E. High-level government officials who leave government service are
currently permitted to act as lobbyists for only three years.
© Sandeep Gupta – Ivy-GMAT, Koramangala Bangalore. sandeepgupta01@gmail.com, 9739561394
4. When limitations were in effect on nuclear-arms testing, people
tended to save more of their money, but when nuclear-arms testing
increased, people tended to spend more of their money. The
perceived threat of nuclear catastrophe, therefore, decreases the
willingness of people to postpone consumption for the sake of saving
money.
The argument above assumes that
A. the perceived threat of nuclear catastrophe has increased over the
years.
B. most people supported the development of nuclear arms
C. people’s perception of the threat of nuclear catastrophe depends on
the amount of nuclear-arms testing being done
D. the people who saved the most money when nuclear-arms testing
was limited were the ones who supported such limitations
E. there are more consumer goods available when nuclear-arms testing
increases
© Sandeep Gupta – Ivy-GMAT, Koramangala Bangalore. sandeepgupta01@gmail.com, 9739561394
5. If the airspace around centrally located airports were restricted to commercial
airliners and only those private planes equipped with radar, most of the
private-plane traffic would be forced to use outlying airfields. Such a reduction
in the amount of private-plane traffic would reduce the risk of midair collision
around the centrally located airports.
The conclusion drawn in the first sentence depends on which of the
following assumptions?
A. Outlying airfields would be as convenient as centrally located airports for most
pilots of private planes.
B. Most outlying airfields are not equipped to handle commercial-airline traffic.
C. Most private planes that use centrally located airports are not equipped with
radar.
D. Commercial airliners are at greater risk of becoming involved in midair
collisions than are private planes.
E. A reduction in the risk of midair collision would eventually lead to increases in
commercial-airline traffic.
© Sandeep Gupta – Ivy-GMAT, Koramangala Bangalore. sandeepgupta01@gmail.com, 9739561394
6. In recent years many cabinetmakers have been winning acclaim as
artists. But since furniture must be useful, cabinetmakers must
exercise their craft with an eye to the practical utility of their product.
For this reason, cabinet-making is not art.
Which of the following is an assumption that supports drawing
the conclusion above from the reason given for that conclusion?
A. Some furniture is made to be placed in museums, where it will not be
used by anyone.
B. Some cabinetmakers are more concerned than others with the
practical utility of the products they produce.
C. Cabinetmakers should be more concerned with the practical utility of
their products than they currently are.
D. An object is not an art object if its maker pays attention to the object’s
practical utility.
E. Artists are not concerned with the monetary value of their products.
© Sandeep Gupta – Ivy-GMAT, Koramangala Bangalore. sandeepgupta01@gmail.com, 9739561394
7. Traditionally, decision-making by managers that is reasoned step-by-step has
been considered preferable to intuitive decision-making. However, a recent
study found that top managers used intuition significantly more than did most
middle-or lower-level managers. This confirms the alternative view that
intuition is actually more effective than careful, methodical reasoning.
The conclusion above is based on which of the following assumptions?
A. Methodical, step-by-step reasoning is inappropriate for making many real-life
management decisions.
B. Top managers have the ability to use either intuitive reasoning or methodical,
step-by-step reasoning in making decisions.
C. The decisions made by middle-and lower-level managers can be made as
easily by using methodical reasoning as by using intuitive reasoning.
D. Top managers use intuitive reasoning in making the majority of their decisions.
E. Top managers are more effective at decision-making than middle-or lower-
level managers
© Sandeep Gupta – Ivy-GMAT, Koramangala Bangalore. sandeepgupta01@gmail.com, 9739561394
8. Emissions from automobiles that burn gasoline and automobiles that
burn diesel fuel are threatening the quality of life on our planet,
contaminating both urban air and global atmosphere. Therefore, the
only effective way to reduce such emissions is to replace the
conventional diesel fuel and gasoline used in automobiles with
cleaner-burning fuels, such as methanol, that create fewer emissions.
Which one of the following is an assumption on which the
argument depends?
A. Reducing the use of automobiles would not be a more effective
means to reduce automobile emissions than the use of methanol.
B. There is no fuel other than methanol that is cleaner-burning than both
diesel fuel and gasoline.
C. If given a choice of automobile fuels, automobile owners would not
select gasoline over methanol.
D. Automobile emissions constitute the most serious threat to the global
environment.
E. At any given time there is a direct correlation between the level of
urban air pollution and the level of contamination present in the global
atmosphere.
© Sandeep Gupta – Ivy-GMAT, Koramangala Bangalore. sandeepgupta01@gmail.com, 9739561394
9. Every year many people become ill because of airborne mold spores
in their homes. After someone becomes ill, specialists are often hired
to eradicate the mold. These specialists look in damp areas of the
house, since mold is almost always found in places where there is
substantial moisture. If one wishes to avoid mold poisoning, then, one
should make sure to keep all internal plumbing in good condition to
prevent leakage that could serve as a breeding ground for mold.
Which of the following is an assumption on which the argument
depends?
A. Mold itself does not create moisture.
B. Most homeowners know enough about plumbing to determine
whether theirs is in good condition.
C. Mold cannot grow in dry areas.
D. No varieties of mold are harmless.
E. Mold spores cannot be filtered from the air.
© Sandeep Gupta – Ivy-GMAT, Koramangala Bangalore. sandeepgupta01@gmail.com, 9739561394
10. Smoking is a known cause of certain serious health problems,
including emphysema and lung cancer. Now, an additional concern
can be added to the list of maladies caused by smoking. A recent
study surveyed both smokers and nonsmokers, and found that
smokers are significantly more anxious and nervous than
nonsmokers.
Which of the following is an assumption on which the argument
rests?
A. Anxiety and nervousness can lead to serious health problems.
B. Anxiety and nervousness do not make individuals more likely to start
smoking.
C. Equivalent numbers of smokers and nonsmokers were surveyed for
the study.
D. Smokers are aware of the various health problems attributed to
smoking, including lung cancer and emphysema.
E. Smokers who had smoked a cigarette immediately before responding
to the survey were more anxious and nervous than smokers who had
not smoked for several hours.
© Sandeep Gupta – Ivy-GMAT, Koramangala Bangalore. sandeepgupta01@gmail.com, 9739561394
11. A researcher discovered that people who have low levels of immune-
system activity tend to score much lower on tests of mental health
than do people with normal or high immune-system activity. The
researcher concluded from this experiment that the immune system
protects against mental illness as well as against physical disease.
The researcher’s conclusion depends on which of the following
assumptions?
A. High immune-system activity protects against mental illness better
than normal immune-system activity does.
B. Mental illness is similar to physical disease in its effects on body
systems.
C. People with high immune-system activity cannot develop mental
illness.
D. Mental illness does not cause people’s immune-system activity to
decrease.
E. Psychological treatment of mental illness is not as effective as is
medical treatment.
© Sandeep Gupta – Ivy-GMAT, Koramangala Bangalore. sandeepgupta01@gmail.com, 9739561394
12. A famous singer recently won a lawsuit against an advertising firm for
using another singer in a commercial to evoke the famous singer’s
well-known rendition of a certain song. As a result of the lawsuit,
advertising firms will stop using imitators in commercials. Therefore,
advertising costs will rise, since famous singers’ services cost more
than those of their imitators.
The conclusion above is based on which of the following
assumptions?
A. Most people are unable to distinguish a famous singer’s rendition of a
song from a good imitator’s rendition of the same song.
B. Commercials using famous singers are usually more effective than
commercials using imitators of famous singers.
C. The original versions of some well-known songs are unavailable for
use in commercials.
D. Advertising firms will continue to use imitators to mimic the physical
mannerisms of famous singers.
E. The advertising industry will use well-known renditions of songs in
commercials.
© Sandeep Gupta – Ivy-GMAT, Koramangala Bangalore. sandeepgupta01@gmail.com, 9739561394
13. Although there has been great scientific debate for decades over
global warming, most scientists now agree that human activity is
causing the Earth’s temperature to rise. Though predictions vary,
many global warming experts believe that average global
temperatures will rise between three and eight degrees Fahrenheit
during the next century. Such an increase would cause an alarming
rise in sea levels, displacing millions of people by destroying major
population centers along the world’s coastlines.
Which of the following is an assumption in support of the
argument’s conclusion?
A. New technological developments in the next century will not divert
rising seas from the world’s coastal cities.
B. Individuals will not become more aware of the steps they can take to
reduce the emission of greenhouse gases.
C. Rising sea levels similarly affect all coastal population centers.
D. Some global warming experts predict a greater than eight degree
Fahrenheit increase in global temperatures during the next century.
E. Human activity is the sole cause of increasing global temperatures.
© Sandeep Gupta – Ivy-GMAT, Koramangala Bangalore. sandeepgupta01@gmail.com, 9739561394
14. The media claim that the economy is entering a phase of growth and
prosperity. They point to lower unemployment rates and increased
productivity. This analysis is false, though. The number of people filing
for bankruptcy has increased every month for the last six months, and
bankruptcy lawyers report that they are busier than they have been in
years.
Which of the following is an assumption on which the argument
depends?
A. Unemployment rates are not useful indicators of growth and
prosperity.
B. Economic growth cannot be measured in terms of productivity.
C. Legislation has not been recently passed to make legal bankruptcy
easier to obtain.
D. There has not been an increase in the number of bankruptcy lawyers.
E. The media often misrepresent the current state of economic affairs.
© Sandeep Gupta – Ivy-GMAT, Koramangala Bangalore. sandeepgupta01@gmail.com, 9739561394
15. Doctor: Research shows that adolescents who play video games on a
regular basis are three times as likely to develop carpal tunnel
syndrome as are adolescents who do not play video games. Federal
legislation that prohibits the sale of video games to minors would help
curb this painful wrist condition among adolescents.
The doctor’s conclusion depends on which of the following
assumptions?
A. The majority of federal legislators would vote for a bill that prohibits
the sale of video games to minors.
B. Not all adolescents who play video games on a regular basis suffer
from carpal tunnel syndrome.
C. Playing video games is the only way an adolescent can develop
carpal tunnel syndrome.
D. Most parents would refuse to purchase video games for their
adolescent children.
E. The regular playing of video games by adolescents does not produce
such beneficial effects as better hand-eye coordination and improved
reaction time.
© Sandeep Gupta – Ivy-GMAT, Koramangala Bangalore. sandeepgupta01@gmail.com, 9739561394
CR Type 4: Weaken
• All weaken questions will have either a FACT-
CONCLUSION or a FACT-REASON (A causes
B) relationship.
• We can never negate a fact. We can negate only
the conclusion or the reason. So the model will
be:
F  C  OR F  R 
© Sandeep Gupta – Ivy-GMAT, Koramangala Bangalore. sandeepgupta01@gmail.com, 9739561394
Weaken Questions
• Options have to contain OUTSIDE (EXTRA)
INFORMATION (FACT)
• Weaken does not mean DESTROY.
• So even mild choices may be true.
© Sandeep Gupta – Ivy-GMAT, Koramangala Bangalore. sandeepgupta01@gmail.com, 9739561394
Weaken
1.Example: As the Indian cricket team performed
badly in the last World Cup, it should not be sent
for the next World Cup.
– FACT: the Indian cricket team performed badly in the
last World Cup
– Conclusion: it should not be sent for the next World
Cup.
• How to weaken it?
© Sandeep Gupta – Ivy-GMAT, Koramangala Bangalore. sandeepgupta01@gmail.com, 9739561394
Weaken
2. Over the last 2 months, a police station has started
receiving many more calls about crime in the adjoining
areas. This is a clear indication of the fact that crime is
on the rise in the adjoining areas.
– FACT: Over the last 2 month, a police station has started
receiving many more calls about crime in the adjoining areas.
– Conclusion: crime is on the rise in the adjoining areas.
• How to weaken it?
• This is A causes B… i.e., more calls about crime  more
crime…
• We can not question facts but only the reason…
© Sandeep Gupta – Ivy-GMAT, Koramangala Bangalore. sandeepgupta01@gmail.com, 9739561394
Process
• ID the conclusion
• Try to come up with your own answer
• New-information choices are welcome
• Undermine the conclusion with the help of
a choice
• Look out for “ALTERNATIVE
EXPLANATIONS” in FACT-REASON
relationships.
© Sandeep Gupta – Ivy-GMAT, Koramangala Bangalore. sandeepgupta01@gmail.com, 9739561394
How to weaken A causes B
• Show that C (something else) causes B
• Show that when A happened, B did not happen.
• Show that when B happened, A did not happen.
• Show that B causes A (reverse relation)
© Sandeep Gupta – Ivy-GMAT, Koramangala Bangalore. sandeepgupta01@gmail.com, 9739561394
A causes B can be written as…
• caused by
• because of
• responsible for
• reason for
• leads to
• induced by
• promoted by
• determined by
• produced by
• product of
• played a role in
• was a factor in
• is an effect of
© Sandeep Gupta – Ivy-GMAT, Koramangala Bangalore. sandeepgupta01@gmail.com, 9739561394
Common Words Used…
• weaken
• attack
• undermine
• refute
• argue against
• call into question
• cast doubt
• challenge
• damage
• counter
• absurdity
• vulnerable
• flaw
• counter
• questionable
• fallacious
© Sandeep Gupta – Ivy-GMAT, Koramangala Bangalore. sandeepgupta01@gmail.com, 9739561394
1. The number of airplanes equipped with a new anti-collision device has
increased steadily during the past two years. During the same period, it
has become increasingly common for key information about an
airplane’s altitude and speed to disappear suddenly from air traffic
controllers’ screens. The new anti-collision device, which operates at the
same frequency as air traffic radar, is therefore responsible for the
sudden disappearance of key information.
Which one of the following, if true, most seriously weakens the
argument?
A. The new anti-collision device has already prevented a considerable
number of mid-air collisions.
B. It was not until the new anti-collision device was introduced that key
information first began disappearing suddenly from controllers’ screens.
C. The new anti-collision device is scheduled to be moved to a different
frequency within the next two to three months.
D. Key information began disappearing from controllers’ screens three
months before the new anti-collision device was first installed.
E. The sudden disappearance of key information from controllers’ screens
has occurred only at relatively large airports.
© Sandeep Gupta – Ivy-GMAT, Koramangala Bangalore. sandeepgupta01@gmail.com, 9739561394
2. The average life expectancy for the United States population as a whole is
73.9 years, but children born in Hawaii will live an average of 77 years,
and those born in Louisiana, 71.7 years. If a newlywed couple from
Louisiana were to begin their family in Hawaii, therefore, their children
would be expected to live longer than would be the case if the family
remained in Louisiana.
Which of the following, if true, would most seriously weaken the
conclusion drawn in the passage?
A. Insurance company statisticians do not believe that moving to Hawaii will
significantly lengthen the average Louisianan’s life.
B. The governor of Louisiana has falsely alleged that statistics for his state
are inaccurate.
C. The longevity ascribed to Hawaii’s current population is attributable mostly
to genetically determined factors.
D. Thirty percent of all Louisianans can expect to live longer than 77 years.
E. Most of the Hawaiian Islands have levels of air pollution well below the
national average for the United States.
© Sandeep Gupta – Ivy-GMAT, Koramangala Bangalore. sandeepgupta01@gmail.com, 9739561394
3. A program instituted in a particular state allows parents to prepay their
children’s future college tuition at current rates. The program then pays
the tuition annually for the child at any of the state’s public colleges in
which the child enrolls. Parents should participate in the program as a
means of decreasing the cost for their children’s college education.
Which of the following, if true, is the most appropriate reason for
parents NOT to participate in the program?
A. The parents are unsure about which public college in the state the child
will attend.
B. The amount of money accumulated by putting the prepayment funds in an
interest-bearing account today will be greater than the total cost of tuition
for any of the public colleges when the child enrolls.
C. The annual cost of tuition at the state’s public colleges is expected to
increase at a faster rate than the annual increase in the cost of living.
D. Some of the state’s public colleges are contemplating large increases in
tuition next year.
E. The prepayment plan would not cover the cost of room and board at any
of the state’s public colleges.
© Sandeep Gupta – Ivy-GMAT, Koramangala Bangalore. sandeepgupta01@gmail.com, 9739561394
4. The ice on the front windshield of the car had formed when moisture
condensed during the night. The ice melted quickly after the car was
warmed up the next morning because the defrosting vent, which
blows on the front windshield, was turned on full force.
Which of the following, if true, most seriously jeopardizes the validity
of the explanation for the speed with which the ice melted?
A. The side windows had no ice condensation on them
B. Even though no attempt was made to defrost the back window, the ice
there melted at the same rate as did the ice on the front windshield.
C. The speed at which ice on a window melts increases as the
temperature of the air blown on the window increases
D. The warm air from the defrosting vent for the front windshield cools
rapidly as it dissipates throughout the rest of the car.
E. The defrosting vent operates efficiently even when the heater, which
blows warm air toward the feet or faces of the driver and passengers,
is on.
© Sandeep Gupta – Ivy-GMAT, Koramangala Bangalore. sandeepgupta01@gmail.com, 9739561394
5. A conservation group in the United States is trying to change the long-standing
image of bats as frightening creatures. The group contends that bats are
feared and persecuted solely because they are shy animals that are active
only at night.
Which of the following, if true, would cast the most serious doubt on the
accuracy of the group’s contention?
A. Bats are steadily losing natural roosting places such as caves and hollow
trees and are thus turning to more developed areas for roosting.
B. Bats are the chief consumers of nocturnal insects and thus can help make
their hunting territory more pleasant for humans.
C. Bats are regarded as frightening creatures not only in the United States but
also in Europe, Africa, and South America.
D. Raccoons and owls are shy and active only at night; yet they are not generally
feared and persecuted.
E. People know more about the behavior of other greatly feared animal species,
such as lions, alligators, and greatly feared animal species, such as lions,
alligators, and snakes, than they do about the behavior of bats.
© Sandeep Gupta – Ivy-GMAT, Koramangala Bangalore. sandeepgupta01@gmail.com, 9739561394
6. Beverage company representative: The plastic rings that hold six-packs of
beverage cans together pose a threat to wild animals, which often become
entangled in the discarded rings and suffocate as a result. Following our lead, all
beverage companies will soon use only those rings consisting of a new plastic
that disintegrates after only three days’ exposure to sunlight. Once we all
complete the switchover from the old to the new plastic rings, therefore, the
threat of suffocation that plastic rings pose to wild animals will be eliminated.
Which one of the following, if true, most seriously weakens the
representative’s argument?
A. The switchover to the new plastic rings will take at least two more years to
complete.
B. After the beverage companies have switched over to the new plastic rings, a
substantial number of the old plastic rings will persist in most aquatic and
woodland environments.
C. The new plastic rings are slightly less expensive than the old rings.
D. The new plastic rings rarely disintegrate during shipping of beverage six-packs
because most trucks that transport canned beverages protect their cargo from
sunlight.
E. The new plastic rings disintegrate into substances that are harmful to aquatic
animals when ingested in substantial quantities by them.
© Sandeep Gupta – Ivy-GMAT, Koramangala Bangalore. sandeepgupta01@gmail.com, 9739561394
7. Opponents of laws that require automobile drivers and passengers to wear
seat belts argue that in a free society people have the right to take risks as
long as the people do not harm other as a result of taking the risks. As a
result, they conclude that it should be each person’s decision whether or not to
wear a seat belt.
Which of the following, if true, most seriously weakens the conclusion
drawn above?
A. Many new cars are built with seat belts that automatically fasten when
someone sits in the front seat.
B. Automobile insurance rates for all automobile owners are higher because of
the need to pay for the increased injuries or deaths of people not wearing seat
belts.
C. Passengers in airplanes are required to wear seat belts during takeoffs and
landings.
D. The rate of automobile fatalities in states that do not have mandatory seat belt
laws is greater than the rate of fatalities in states that do have such laws.
E. In automobile accidents, a greater number of passengers who do not wear
seat belts are injured than are passengers who do wear seat belts.
© Sandeep Gupta – Ivy-GMAT, Koramangala Bangalore. sandeepgupta01@gmail.com, 9739561394
8. During the Second World War, about 375,000 civilians died in the United
States and about 408,000 members of the United States armed forces died
overseas. On the basis the those figures, it can be concluded that it was not
much more dangerous to be overseas in the armed forces during the Second
World War than it was to stay at home as a civilian.
Which of the following would reveal most clearly the absurdity of the
conclusion drawn above?
A. Counting deaths among members of the armed forces who served in the
United State in addition to deaths among members of the armed forces
serving overseas
B. Expressing the difference between the numbers of deaths among civilians and
members of the armed forces as a percentage of the total number of deaths
C. Separating deaths caused by accidents during service in the armed forces
from deaths caused by combat injuries
D. Comparing death rates per thousand members of each group rather than
comparing total numbers of deaths
E. Comparing deaths caused by accidents in the United States to deaths caused
by combat in the armed forces
© Sandeep Gupta – Ivy-GMAT, Koramangala Bangalore. sandeepgupta01@gmail.com, 9739561394
9. Recently in City X, developers have stopped buying land, contractors
have found themselves going without work for longer periods, and
banks have issued fewer mortgages. There must be fewer new
residents moving to City X than there were previously.
Which of the following indicates a flaw in the reasoning above?
A. This year several housing blocks have gone on the market after being
held up for months by legal red tape.
B. The average size of a new home has increased significantly over the
past several years.
C. Re-sales of condominiums have increased over the past six months.
D. The cost of materials such as lumber and cement has decreased over
the past year.
E. Sales of other big-ticket items, such as automobiles and boats, has
remained steady over the past year.
© Sandeep Gupta – Ivy-GMAT, Koramangala Bangalore. sandeepgupta01@gmail.com, 9739561394
10. Companies that offer “employer sponsored insurance” (ESI) pay a portion of
employees’ health care costs. In the manufacturing sector last year,
companies that offered ESI had worker absentee rates 22% lower, on
average, than those at companies that did not offer ESI.
If, on the basis of the evidence above, it is argued that ESI decreases
worker absenteeism, which of the following, if true, would most
seriously weaken that argument?
A. Results similar to those cited for the manufacturing sector have been found in
other sectors of the economy where ESI is offered.
B. At companies that offer ESI, employees have access to preventative health
care such as regular check-ups, routine laboratory tests, and nutrition
counseling.
C. Because initiating an ESI plan requires a lot of paperwork for the company,
employees, and the insurance provider, doing so is complex and time-
consuming.
D. Many firms in the manufacturing sector have improved workplace safety and
decreased the occurrence of on-the-job injuries in the last five years, and most
of these companies introduced ESI at the same time.
E. In manufacturing firms where ESI is offered, the average productivity is 2%
higher than it is in those firms where workers are not covered by an ESI plan.
© Sandeep Gupta – Ivy-GMAT, Koramangala Bangalore. sandeepgupta01@gmail.com, 9739561394
Numbers and Percentages Misconceptions
1. Increasing percentages automatically lead to increasing
numbers.
2. Decreasing percentages automatically lead to decreasing
numbers.
3. Increasing numbers automatically lead to increasing
percentages.
4. Decreasing numbers automatically lead to decreasing
percentages.
5. Large numbers automatically mean large percentages, and
small numbers automatically mean small percentages.
6. Large percentages automatically mean large numbers, and
small percentages automatically mean small numbers.
© Sandeep Gupta – Ivy-GMAT, Koramangala Bangalore. sandeepgupta01@gmail.com, 9739561394
Common terms
Words used to
introduce NUMBERS:
• Amount
• Quantity
• Sum
• Total
• Count
• Tally
Words used to
introduce percentage
ideas:
• Percent
• Proportion
• Fraction
• Ratio
• Incidence
• Likelihood
• Probability
• Segment
• Share
© Sandeep Gupta – Ivy-GMAT, Koramangala Bangalore. sandeepgupta01@gmail.com, 9739561394
Market Share
Market share is a comparative term, as opposed to an absolute term.
Market share is simply the portion of a market that a company controls.
The market share can be measured either in terms of revenues (sales) or
units sold.
A company can gain market share (percentage) if the market shrinks and it
maintains a constant size, or if it grows in an unchanging market.
However, a company losing market share does not mean that its sales
decreased, only that they became a smaller entity in the market relative to
the whole (the market grew and they stayed the same size, for example).
Similarly, a company could lose sales and still gain market share if the
overall market became smaller.
The total market share must always add up to 100%.
© Sandeep Gupta – Ivy-GMAT, Koramangala Bangalore. sandeepgupta01@gmail.com, 9739561394
CR Type 5: Strengthen
• All strengthen questions will have either a FACT-
CONCLUSION or a FACT-REASON (A causes
B) relationship.
• We can never strengthen a fact. We can
strengthen only the conclusion or the reason by
providing some extra facts. So the model will be:
F  C  OR F  R 
© Sandeep Gupta – Ivy-GMAT, Koramangala Bangalore. sandeepgupta01@gmail.com, 9739561394
Strengthen Questions
• Options have to contain OUTSIDE (EXTRA)
INFORMATION (FACT)
• Strengthen does not mean PROVE.
• So even mild choices may be true.
© Sandeep Gupta – Ivy-GMAT, Koramangala Bangalore. sandeepgupta01@gmail.com, 9739561394
Strengthen
• Example: As the Indian cricket team performed
badly in the last World Cup, it should not be sent
for the next World Cup.
– FACT: the Indian cricket team performed badly in the
last World Cup
– Conclusion: it should not be sent for the next World
Cup.
• How to strengthen it?
© Sandeep Gupta – Ivy-GMAT, Koramangala Bangalore. sandeepgupta01@gmail.com, 9739561394
Terms used…
• strengthens
• supports
• helps
• most justifies
• bolsters
• buttresses
© Sandeep Gupta – Ivy-GMAT, Koramangala Bangalore. sandeepgupta01@gmail.com, 9739561394
How to strengthen A causes B
• Show that nothing else causes B
• Show that when A did not happen, B did not
happen.
• Show that when B did not happen, A did not
happen.
• Show that B does not cause A
© Sandeep Gupta – Ivy-GMAT, Koramangala Bangalore. sandeepgupta01@gmail.com, 9739561394
Strengthen – cause effect…
Navigation systems found in most aircraft are
made with low-power circuitry, which is
susceptible to interference. Recently, one plane
veered off course during landing, when a
passenger turned on a laptop computer. Clearly,
aircraft navigation systems are being put at risk
by the electronic devices that passengers carry
on board, such as cassette players and laptop
computers.
Find at least 2 ways to strengthen it…
© Sandeep Gupta – Ivy-GMAT, Koramangala Bangalore. sandeepgupta01@gmail.com, 9739561394
Process
• ID the conclusion
• Try to come up with your own answer
• New-information choices are welcome
• Strengthen the conclusion with the help of a
choice
© Sandeep Gupta – Ivy-GMAT, Koramangala Bangalore. sandeepgupta01@gmail.com, 9739561394
1. Medical doctor: Sleep deprivation is the cause of many social ills,
ranging from irritability to potentially dangerous instances of impaired
decision making. Most people today suffer from sleep deprivation to
some degree. Therefore we should restructure the workday to allow
people flexibility in scheduling their work hours. Which one of the
following, if true, would most strengthen the medical doctor’s argument?
A. The primary cause of sleep deprivation is overwork.
B. Employees would get more sleep if they had greater latitude in
scheduling their work hours.
C. Individuals vary widely in the amount of sleep they require.
D. More people would suffer from sleep deprivation today than did in the
past if the average number of hours worked per week had not
decreased.
E. The extent of one’s sleep deprivation is proportional to the length of
one’s workday.
© Sandeep Gupta – Ivy-GMAT, Koramangala Bangalore. sandeepgupta01@gmail.com, 9739561394
2. Galanin is a protein found in the brain. In an experiment, rats that
consistently chose to eat fatty foods when offered a choice between lean
and fatty foods were found to have significantly higher concentrations of
galanin in their brains than did rats that consistently chose lean over
fatty foods. These facts strongly support the conclusion that galanin
causes rats to crave fatty foods. Which one of the following, if true, most
supports the argument?
A. The craving for fatty foods does not invariably result in a rat’s choosing
those foods over lean foods.
B. The brains of the rats that consistently chose to eat fatty foods did not
contain significantly more fat than did the brains of rats that consistently
chose lean foods.
C. The chemical components of galanin are present in both fatty foods and
lean foods.
D. The rats that preferred fatty foods had the higher concentrations of
galanin in their brains before they were offered fatty foods.
E. Rats that metabolize fat less efficiently than do other rats develop high
concentrations of galanin in their brains.
© Sandeep Gupta – Ivy-GMAT, Koramangala Bangalore. sandeepgupta01@gmail.com, 9739561394
3. In the 18th and 19th centuries, it was believed in many coastal
American cities that the waterfront was an undesirable location for
residential buildings. As a result, much of the waterfront in these cities
was never developed aesthetically and instead was left to industry
and commerce. Today, however, waterfront properties are generally
seen as prestigious, as evidenced by the large sums paid for homes
along the beach front. A developer who wishes to make a large profit
would be wise to buy urban waterfront lots and erect residential
buildings on them.
Which of the following, if true, most strongly supports the claim
made about urban waterfront properties?
A. People today have more money, relatively speaking, to spend on real
estate than they did in previous centuries.
B. Homeowners will be willing to spend large sums on residential
properties in traditionally industrial or commercial districts.
C. Many urban waterfront lots are available for purchase.
D. Many coastal American cities are encouraging developers to
rehabilitate the waterfront through tax incentives.
E. Properties in interior residential districts in coastal American cities are
significantly more expensive than those along the waterfront.
© Sandeep Gupta – Ivy-GMAT, Koramangala Bangalore. sandeepgupta01@gmail.com, 9739561394
4. Some animals, such as dolphins, dogs, and African grey parrots,
seem to exhibit cognitive functions typically associated with higher-
order primates such as chimpanzees, gorillas, and humans. Some
parrots, for example, have vocabularies of hundreds of words that
they can string together in a comprehensible syntax. This clearly
shows that humans and primates are not the only animals capable of
using language to communicate. One parrot, named Alex, has been
known to ask to be petted or kissed and will exhibit aggression if the
gesture offered is not the specific one requested.
Which of the following, if true, would most strengthen the conclusion
above?
A. Dolphins can be trained to assist divers in ocean rescues.
B. Gorillas in captivity often learn hand signals for food and water.
C. Dogs are capable of sensing their owners' moods and often exhibit
concern if they sense sadness.
D. Chimpanzees can memorize long sequences of key punches on
machines that dispense food.
E. Alex does not exhibit aggression when offered a gesture that he
specifically requested.
© Sandeep Gupta – Ivy-GMAT, Koramangala Bangalore. sandeepgupta01@gmail.com, 9739561394
5. In general, jobs are harder to get in times of economic recession because
many businesses cut back operations. However, any future recessions in
Vargonia will probably not reduce the availability of teaching jobs at
government-funded schools. This is because Vargonia has just introduced a
legal requirement that education in government-funded schools be available,
free of charge, to all Vargonian children regardless of the state of the
economy, and that current student-teacher ratios not be exceeded.
Which of the following, if true, most strengthens the argument?
A. The current student-teacher ratio at Vargonia’s government-funded schools is
higher than it was during the most recent period of economic recession.
B. During recent periods when the Vargonian economy has been strong, almost
25 percent of Vargonian children have attended privately funded schools,
many of which charge substantial fees.
C. Nearly 20 percent more teachers are currently employed in Vargonia’s
government-funded schools than had been employed in those schools in the
period before the last economic recession.
D. Teachers in Vargonia’s government-funded schools are well paid relative to
teachers in most privately funded schools in Vargonia, many of which rely
heavily on part-time teachers.
E. During the last economic recession in Vargonia, the government permanently
closed a number of the schools that it had funded.
© Sandeep Gupta – Ivy-GMAT, Koramangala Bangalore. sandeepgupta01@gmail.com, 9739561394
6. The average life expectancy for the United States population as a
whole is 73.9 years, but children born in Hawaii will live an average of
77 years, and those born in Louisiana, 71.7 years. If a newlywed
couple from Louisiana were to begin their family in Hawaii, therefore,
their children would be expected to live longer than would be the case
if the family remained in Louisiana.
Which of the following statements, if true, would most
significantly strengthen the conclusion drawn in the passage?
A. As population density increases in Hawaii, life expectancy figures for
that state are likely to be revised downward.
B. Environmental factors tending to favor longevity are abundant in
Hawaii and less numerous in Louisiana.
C. Twenty-five percent of all Louisianans who move to Hawaii live longer
than 77 years.
D. Over the last decade, average life expectancy has risen at a higher
rate for Louisianans than for Hawaiians.
E. Studies show that the average life expectancy for Hawaiians who
move permanently to Louisiana is roughly equal to that of Hawaiians
who remain in Hawaii.
© Sandeep Gupta – Ivy-GMAT, Koramangala Bangalore. sandeepgupta01@gmail.com, 9739561394
7. A cost-effective solution to the problem of airport congestion is to
provide high-speed ground transportation between major cities lying
200 to 500 miles apart. The successful implementation of this plan
would cost far less than expanding existing airports and would also
reduce the number of airplanes clogging both airports and airways.
Which of the following, if true, could be proponents of the plan
above most appropriately cite as a piece of evidence for the
soundness of their plan?
A. An effective high-speed ground-transportation system would require
major repairs to many highways and mass-transit improvements.
B. One-half of all departing flights in the nation’s busiest airport head for
a destination in a major city 225 miles away.
C. The majority of travelers departing from rural airports are flying to
destinations in cities over 600 miles away.
D. Many new airports are being built in areas that are presently served
by high-speed ground-transportation systems.
E. A large proportion of air travelers are vacationers who are taking long-
distance flights.
© Sandeep Gupta – Ivy-GMAT, Koramangala Bangalore. sandeepgupta01@gmail.com, 9739561394
8. A recent spate of launching and operating mishaps with television satellites led
to a corresponding surge in claims against companies underwriting satellite
insurance. As a result, insurance premiums shot up, making satellites more
expensive to launch and operate. This, in turn, has added to the pressure to
squeeze more performance out of currently operating satellites.
Which of the following, if true, taken together with the information
above, best supports the conclusion that the cost of television satellites
will continue to increase?
A. Since the risk to insurers of satellites is spread over relatively few units,
insurance premiums are necessarily very high.
B. When satellites reach orbit and then fail, the causes of failure are generally
impossible to pinpoint with confidence.
C. The greater the performance demands placed on satellites, the more
frequently those satellites break down.
D. Most satellites are produced in such small numbers that no economies of
scale can be realized.
E. Since many satellites are built by unwieldy international consortia,
inefficiencies are inevitable.
© Sandeep Gupta – Ivy-GMAT, Koramangala Bangalore. sandeepgupta01@gmail.com, 9739561394
CR Type 6: Evaluate the Argument
• “EVALUATE” means to check the soundness of
conclusion…
• All EVALUATE questions will have a FACT-
CONCLUSION relationship.
• The given argument will be unsound, i.e., there will be
inadequate facts to support the conclusion.
• In order to prove or disprove the argument, one will need
more information. This is what the question asks…
– What extra piece of information will help one evaluate
the conclusion better?
© Sandeep Gupta – Ivy-GMAT, Koramangala Bangalore. sandeepgupta01@gmail.com, 9739561394
Approach…
• ID the conclusion.
• Take the choices to the extremes.
• The Extremes Test: One extreme should
weaken and the other should strengthen the
conclusion or vice versa.
© Sandeep Gupta – Ivy-GMAT, Koramangala Bangalore. sandeepgupta01@gmail.com, 9739561394
An Example…
1. Advertisement: Only Gem World sends its
jewelry to the most reliable Gem Testing Lab in
the country. No other shop sends its jewelry for
any kind of testing whatsoever. So buy jewelry
from Gem World, the most reliable jewelry
shop in town, and be assured of the quality of
gems you purchase!
The answer to which question will help one
evaluate the authenticity of the claim made in the
advertisement above?
© Sandeep Gupta – Ivy-GMAT, Koramangala Bangalore. sandeepgupta01@gmail.com, 9739561394
Another Example…
2. Because of a rare type of fungus that killed off many
cacao trees in Brazil, there was an unusually meager
harvest of cocoa beans this year. The wholesale price
of cocoa solids and cocoa butter has increased
significantly and does not look likely to fall in the
foreseeable future. As a result, the retail price of
chocolate is certain to increase within six months.
The answer to which question would provide
information relevant to evaluating the claims made
in the argument above?
© Sandeep Gupta – Ivy-GMAT, Koramangala Bangalore. sandeepgupta01@gmail.com, 9739561394
Another Example…
3. In an experiment, two different types of recorded music
were played for neonates in adjacent nurseries in a
hospital. In nursery A, classical music was played; in
nursery B, rock music was played. After two weeks, it
was found that the babies in nursery A cried less,
suffered fewer minor ailments, and gained more weight
than did the babies in nursery B.
In evaluating the validity of the conclusion
suggested by the experiment above, it would be
most important to know which of the following?
© Sandeep Gupta – Ivy-GMAT, Koramangala Bangalore. sandeepgupta01@gmail.com, 9739561394
1. Profits for one of Company X's flagship products have been declining
slowly for several years. The CFO investigated and determined that
inflation has raised the cost of producing the product but consumers
who were surveyed reported that they weren't willing to pay more than
the current price. As a result, the CFO recommended that the
company stop producing this product because the CEO only wants
products whose profit margins are increasing.
The answer to which of the following questions would be most
useful in evaluating whether the CFO's decision to divest the
company of its flagship product is warranted?
A. Does the company have new and profitable products available with
which to replace the flagship product?
B. Will the rest of Company X's management team agree with the CFO's
recommendation?
C. Are there additional features which could be added to the product and
for which consumers might be willing to pay a higher price?
D. Is there a way to alter the manufacturing or distribution processes in
order to reduce the cost to produce the flagship product?
E. What percentage of Company X's revenues is represented by sales of
the flagship product in question?
© Sandeep Gupta – Ivy-GMAT, Koramangala Bangalore. sandeepgupta01@gmail.com, 9739561394
2. Scientists have determined that an effective way to lower cholesterol
is to eat three servings of whole grains every day. Studies have shown
that the cholesterol levels of people who did so were significantly
lower after six months than were those of people who did not, even
though the cholesterol levels of the two groups were the same before
the studies began. Clearly, eating whole grains can have an
appreciable effect on cholesterol levels.
The answer to which of the following questions, if true, would be
most useful in evaluating the claim about whole grains above?
A. Is it realistic to expect people to eat three servings of whole grains per
day?
B. Were the two groups of people in the study involved in the same
exercise program?
C. Can the same drop in cholesterol be achieved through medication?
D. Did the study continue to track the subjects beyond six months?
E. Are most consumers aware of the different between whole grains and
processed grains?
© Sandeep Gupta – Ivy-GMAT, Koramangala Bangalore. sandeepgupta01@gmail.com, 9739561394
3. Guitar strings often go “dead”—become less responsive and bright in
tone—after a few weeks of intense use. A researcher whose son is a
classical guitarist hypothesized that dirt and oil, rather than changes in
the material properties of the string, were responsible.
Which of the following investigations is most likely to yield
significant information that would help to evaluate the
researcher’s hypothesis?
A. Determining if a metal alloy is used to make the strings used by
classical guitarists
B. Determining whether classical guitarists make their strings go dead
faster than do folk guitarists
C. Determining whether identical lengths of string, of the same gauge, go
dead at different rates when strung on various brands of guitars.
D. Determining whether a dead string and a new string produce different
qualities of sound
E. Determining whether smearing various substances on new guitar
strings causes them to go dead
© Sandeep Gupta – Ivy-GMAT, Koramangala Bangalore. sandeepgupta01@gmail.com, 9739561394
4. The recording industry is fighting a losing battle: it simply does not
have the resources to prosecute all of the individuals who illegally
download music from the Internet. Because the number of individuals
who will be charged with a crime is so limited, the actions of the
recording industry will have a minimal impact on the number of people
who illegally download music.
The answer to which of the following questions would best help
evaluate the accuracy of the conclusion above?
A. Will recording industry lawyers dedicate the majority of their time to
prosecuting those who illegally download music?
B. Is a small minority of individuals responsible for the majority of illegal
song downloads?
C. Do many individuals who illegally download songs share their music
files with other Internet users?
D. Will new Internet security technology permit the recording industry to
more quickly and easily identify individuals who illegally download
music?
E. Will the threat of prosecution alter the behavior of those who illegally
download music?
© Sandeep Gupta – Ivy-GMAT, Koramangala Bangalore. sandeepgupta01@gmail.com, 9739561394
5. Columnist: George Orwell’s book 1984 has exercised
much influence on a great number of this newspaper’s
readers. One thousand readers were surveyed and asked
to name the one book that had the most influence on their
lives. The book chosen most often was the Bible; 1984
was second. The answer to which one of the following
questions would most help in evaluating the columnist’s
argument?
A. How many books had each person surveyed read?
B. How many people chose books other than 1984?
C. How many people read the columnist’s newspaper?
D. How many books by George Orwell other than 1984 were
chosen?
E. How many of those surveyed had actually read the books
they chose?
© Sandeep Gupta – Ivy-GMAT, Koramangala Bangalore. sandeepgupta01@gmail.com, 9739561394
6. The proportion of women among students enrolled in higher education
programs has increased over the past decades. This is partly shown by
the fact that in 1959, only 11 percent of the women between twenty and
twenty-one were enrolled in college, while in 1981, 30 percent of the
women between twenty and twenty-one were enrolled in college.
To evaluate the argument above, it would be most useful to
compare 1959 and 1981 with regard to which of the following
characteristics?
A. The percentage of women between twenty and twenty-one who were
not enrolled in college
B. The percentage of women between twenty and twenty-five who
graduated from college
C. The percentage of women who, after attending college, entered highly
paid professions
D. The percentage of men between twenty and twenty-one who were
enrolled in college
E. The percentage of men who graduated from high school
© Sandeep Gupta – Ivy-GMAT, Koramangala Bangalore. sandeepgupta01@gmail.com, 9739561394
CR Type 7: Resolve the Paradox
• Structure:
– Two seemingly contradictory facts are
presented in the passage, creating a paradox
(a surprising phenomenon). F1  F2
– One has to resolve (explain) the Paradox by
choosing the most credible choice that
explains both sides.
– MODEL:
F1F1 F2F2
© Sandeep Gupta – Ivy-GMAT, Koramangala Bangalore. sandeepgupta01@gmail.com, 9739561394
Paradox Questions
• No conclusion
• Language of contradiction (But, However, Yet,
Although, Paradoxically, Surprisingly
• Two sides presented (both to be resolved)
• Eliminate Out-of-scope choices
• Choices are taken as TRUE, no matter
howsoever absurd / ridiculous / counter-
intuitive…
© Sandeep Gupta – Ivy-GMAT, Koramangala Bangalore. sandeepgupta01@gmail.com, 9739561394
Terms generally used…
• Resolve
• Explain
• Reconcile
• Justify
• Paradox
• Discrepancy
• Contradiction
• Conflict
• Puzzle
© Sandeep Gupta – Ivy-GMAT, Koramangala Bangalore. sandeepgupta01@gmail.com, 9739561394
Paradox Questions
1. An anti-theft device is known to reduce theft, but cars using the
antitheft device are stolen at a higher rate than cars without the
device.
2. A surgeon has a low success rate while operating, but the director
of the hospital claims the surgeon is the best on the staff.
3. Country X contains many rivers that flow down from its high
mountains. These rivers have been dammed to harness the
hydroelectric power that can be derived from this resource. More
than enough power is generated from these dams to meet the
country's energy needs. Yet, citizens of Country X often
experience power shortages or even outages.
© Sandeep Gupta – Ivy-GMAT, Koramangala Bangalore. sandeepgupta01@gmail.com, 9739561394
Paradox Questions
4. Some communities in Florida are
populated almost exclusively by retired
people and contain few, if any, families
with small children. Yet these
communities are home to thriving
businesses specializing in the rental of
furniture for infants and small children.
© Sandeep Gupta – Ivy-GMAT, Koramangala Bangalore. sandeepgupta01@gmail.com, 9739561394
1. Calories consumed in excess of those with which the body needs to be
provided to maintain its weight are normally stored as fat and the body
gains weight. Alcoholic beverages are laden with calories. However,
those people who regularly drink two or three alcoholic beverages a day
and thereby exceed the caloric intake necessary to maintain their weight
do not in general gain weight. Which one of the following, if true, most
helps to resolve the apparent discrepancy?
A. Some people who regularly drink two or three alcoholic beverages a day
avoid exceeding the caloric intake necessary to maintain their weight by
decreasing caloric intake from other sources.
B. Excess calories consumed by people who regularly drink two or three
alcoholic beverages a day tend to be dissipated as heat.
C. Some people who do not drink alcoholic beverages but who eat high-
calorie foods do not gain weight.
D. Many people who regularly drink more than three alcoholic beverages a
day do not gain weight.
E. Some people who take in fewer calories than are normally necessary to
maintain their weight do not lose weight.
© Sandeep Gupta – Ivy-GMAT, Koramangala Bangalore. sandeepgupta01@gmail.com, 9739561394
2. In Asia, where palm trees are non-native, the trees’ flowers have traditionally
been pollinated by hand, which has kept palm fruit productivity unnaturally low.
When weevils known to be efficient pollinators of palm flowers were introduced
into Asia in 1980, palm fruit productivity increased-by up to fifty percent in
some areas-but then decreased sharply in 1984.
Which of the following statements, if true, would best explain the 1984
decrease in productivity?
A. Prices for palm fruit fell between 1980 and 1984 following the rise in
production and a concurrent fall in demand.
B. Imported trees are often more productive than native trees because the
imported ones have left behind their pests and diseases in their native lands.
C. Rapid increases in productivity tend to deplete trees of nutrients needed for
the development of the fruit-producing female flowers.
D. The weevil population in Asia remained at approximately the same level
between 1980 and 1984.
E. Prior to 1980 another species of insect pollinated the Asian palm trees, but not
as efficiently as the species of weevil that was introduced in 1980.
© Sandeep Gupta – Ivy-GMAT, Koramangala Bangalore. sandeepgupta01@gmail.com, 9739561394
3. Provinces and states with stringent car safety requirements, including required
use of seat belts and annual safety inspections, have on average higher rates
of accidents per kilometer driven than do provinces and states with less
stringent requirements. Nevertheless, most highway safety experts agree that
more stringent requirements do reduce accident rates.
Which one of the following, if true, most helps to reconcile the safety
experts’ belief with the apparently contrary evidence described above?
A. Annual safety inspections ensure that car tires are replaced before they grow
old.
B. Drivers often become overconfident after their cars have passed a thorough
safety inspection.
C. The roads in provinces and states with stringent car safety programs are far
more congested and therefore dangerous than in other provinces and states.
D. Psychological studies show that drivers who regularly wear seat belts often
come to think of themselves as serious drivers, which for a few people
discourages reckless driving.
E. Provinces and states with stringent car safety requirements have, on average,
many more kilometers of roads then do other provinces and states.
© Sandeep Gupta – Ivy-GMAT, Koramangala Bangalore. sandeepgupta01@gmail.com, 9739561394
4. Industry experts expect improvements in job safety training to lead to
safer work environments. A recent survey indicated, however, that for
manufacturers who improved job safety training during the 1980s, the
number of on-the-job accidents tended to increase in the months
immediately following the changes in the training programs.
Which one of the following, if true, most helps to resolve the
apparent discrepancy in the passage above?
A. A similar survey found that the number of on- the-job accidents
remained constant after job safety training in the transportation sector
was improved.
B. Manufacturers tend to improve their job safety training only when they
are increasing the size of their workforce.
C. Manufacturers tend to improve job safety training only after they have
noticed that the number of on-the-job accidents has increased.
D. It is likely that the increase in the number of on-the-job accidents
experienced by many companies was not merely a random
fluctuation.
E. Significant safety measures, such as protective equipment and
government safety inspections, were in place well before the
improvements in job safety training.
© Sandeep Gupta – Ivy-GMAT, Koramangala Bangalore. sandeepgupta01@gmail.com, 9739561394
5. Cigarette companies claim that manufacturing both low- and high-
nicotine cigarettes allows smokers to choose how much nicotine they
want. However, a recent study has shown that the levels of nicotine
found in the blood of smokers who smoke one pack of cigarettes per
day are identical at the end of a day’s worth of smoking, whatever the
level of nicotine in the cigarettes they smoke.
Which one of the following, if true, most helps to explain the
finding of the nicotine study?
A. Blood cannot absorb more nicotine per day than that found in the
smoke from a package of the lowest-nicotine cigarettes available.
B. Smokers of the lowest-nicotine cigarettes available generally smoke
more cigarettes per day than smokers of high-nicotine cigarettes.
C. Most nicotine is absorbed into the blood of a smoker even if it is
delivered in smaller quantities.
D. The level of tar in cigarettes is higher in low-nicotine cigarettes than it
is in some high-nicotine cigarettes.
E. When taking in nicotine by smoking cigarettes is discontinued, the
level of nicotine in the blood decreases steadily.
Cr concepts   the best resource for gmat cr from ivy-gmat (sandeep gupta)
Cr concepts   the best resource for gmat cr from ivy-gmat (sandeep gupta)

More Related Content

What's hot

PAYTM - HOW PAYTM POSITIONING THE MARKET
PAYTM - HOW PAYTM POSITIONING THE MARKETPAYTM - HOW PAYTM POSITIONING THE MARKET
PAYTM - HOW PAYTM POSITIONING THE MARKETSanjeet Chhikara
 
Amazon india distribution strategy
Amazon india   distribution strategyAmazon india   distribution strategy
Amazon india distribution strategyTirthankar Sutradhar
 
Gas Air Heater (Air Pre Heater)
Gas Air Heater (Air Pre Heater)Gas Air Heater (Air Pre Heater)
Gas Air Heater (Air Pre Heater)Hammad Akber
 
MAJOR SALES : Who Really Does the Buying?
MAJOR SALES: Who Really Does the Buying?MAJOR SALES: Who Really Does the Buying?
MAJOR SALES : Who Really Does the Buying?Habib Abou Saleh
 
Adani power Practice School
Adani power Practice SchoolAdani power Practice School
Adani power Practice SchoolNemish Kanwar
 
AFBC Boilers, Losses discussions
AFBC Boilers, Losses discussionsAFBC Boilers, Losses discussions
AFBC Boilers, Losses discussionsManohar Tatwawadi
 
A study on Groupon.com
A study on Groupon.comA study on Groupon.com
A study on Groupon.comVinny Wu
 
Customer based brand equity
Customer based brand equityCustomer based brand equity
Customer based brand equityAshek Imran
 
Zomato: Strategic Advantage
Zomato: Strategic AdvantageZomato: Strategic Advantage
Zomato: Strategic AdvantageMalay Wani
 
Waste Heat Recovery Project
Waste Heat Recovery ProjectWaste Heat Recovery Project
Waste Heat Recovery ProjectSUMIT JINDAL
 
Promotions strategy of flipkart
Promotions strategy of flipkartPromotions strategy of flipkart
Promotions strategy of flipkartSameer Mathur
 

What's hot (14)

PAYTM - HOW PAYTM POSITIONING THE MARKET
PAYTM - HOW PAYTM POSITIONING THE MARKETPAYTM - HOW PAYTM POSITIONING THE MARKET
PAYTM - HOW PAYTM POSITIONING THE MARKET
 
Amazon india distribution strategy
Amazon india   distribution strategyAmazon india   distribution strategy
Amazon india distribution strategy
 
Gas Air Heater (Air Pre Heater)
Gas Air Heater (Air Pre Heater)Gas Air Heater (Air Pre Heater)
Gas Air Heater (Air Pre Heater)
 
MAJOR SALES : Who Really Does the Buying?
MAJOR SALES: Who Really Does the Buying?MAJOR SALES: Who Really Does the Buying?
MAJOR SALES : Who Really Does the Buying?
 
Adani power Practice School
Adani power Practice SchoolAdani power Practice School
Adani power Practice School
 
AFBC Boilers, Losses discussions
AFBC Boilers, Losses discussionsAFBC Boilers, Losses discussions
AFBC Boilers, Losses discussions
 
A study on Groupon.com
A study on Groupon.comA study on Groupon.com
A study on Groupon.com
 
Shouldice hospital
Shouldice hospitalShouldice hospital
Shouldice hospital
 
Customer based brand equity
Customer based brand equityCustomer based brand equity
Customer based brand equity
 
Zomato: Strategic Advantage
Zomato: Strategic AdvantageZomato: Strategic Advantage
Zomato: Strategic Advantage
 
Air heater
Air heaterAir heater
Air heater
 
Big basket
Big basketBig basket
Big basket
 
Waste Heat Recovery Project
Waste Heat Recovery ProjectWaste Heat Recovery Project
Waste Heat Recovery Project
 
Promotions strategy of flipkart
Promotions strategy of flipkartPromotions strategy of flipkart
Promotions strategy of flipkart
 

Viewers also liked

Gmat-sentence-correction-guide
Gmat-sentence-correction-guideGmat-sentence-correction-guide
Gmat-sentence-correction-guideRushabh Vora
 
Free gmat-flashcards
Free gmat-flashcardsFree gmat-flashcards
Free gmat-flashcardsSivaprakash
 
GMAT Critical Reasoning - everything you need to know
GMAT Critical Reasoning - everything you need to know GMAT Critical Reasoning - everything you need to know
GMAT Critical Reasoning - everything you need to know GMAT Prep Now
 
Magoosh's Complete Guide to the GMAT
Magoosh's Complete Guide to the GMATMagoosh's Complete Guide to the GMAT
Magoosh's Complete Guide to the GMATMagoosh
 
GMAT Math Flashcards
GMAT Math FlashcardsGMAT Math Flashcards
GMAT Math FlashcardsGMAT Prep Now
 
Quant 125 awesome questions
Quant 125 awesome questionsQuant 125 awesome questions
Quant 125 awesome questionsRushabh Vora
 
Arithmatic & algebra
Arithmatic & algebraArithmatic & algebra
Arithmatic & algebraRushabh Vora
 
SAT / ACT / GRE / GMAT Prep Presentation
SAT / ACT / GRE / GMAT Prep PresentationSAT / ACT / GRE / GMAT Prep Presentation
SAT / ACT / GRE / GMAT Prep PresentationSuccess Prep
 
Gre vocabulary slides
Gre vocabulary slidesGre vocabulary slides
Gre vocabulary slidesNavdeep Kumar
 
Carey Business School
Carey Business SchoolCarey Business School
Carey Business SchoolCrackVerbal
 

Viewers also liked (14)

Gmat-sentence-correction-guide
Gmat-sentence-correction-guideGmat-sentence-correction-guide
Gmat-sentence-correction-guide
 
Free gmat-flashcards
Free gmat-flashcardsFree gmat-flashcards
Free gmat-flashcards
 
GMAT Critical Reasoning - everything you need to know
GMAT Critical Reasoning - everything you need to know GMAT Critical Reasoning - everything you need to know
GMAT Critical Reasoning - everything you need to know
 
Magoosh's Complete Guide to the GMAT
Magoosh's Complete Guide to the GMATMagoosh's Complete Guide to the GMAT
Magoosh's Complete Guide to the GMAT
 
GMAT Math Flashcards
GMAT Math FlashcardsGMAT Math Flashcards
GMAT Math Flashcards
 
Quant 125 awesome questions
Quant 125 awesome questionsQuant 125 awesome questions
Quant 125 awesome questions
 
Gmat Maths
Gmat MathsGmat Maths
Gmat Maths
 
Arithmatic & algebra
Arithmatic & algebraArithmatic & algebra
Arithmatic & algebra
 
GMAT Classes in Bangalore for Regular & Weekend batches
GMAT Classes in Bangalore for Regular & Weekend batchesGMAT Classes in Bangalore for Regular & Weekend batches
GMAT Classes in Bangalore for Regular & Weekend batches
 
SAT / ACT / GRE / GMAT Prep Presentation
SAT / ACT / GRE / GMAT Prep PresentationSAT / ACT / GRE / GMAT Prep Presentation
SAT / ACT / GRE / GMAT Prep Presentation
 
Gre vocabulary slides
Gre vocabulary slidesGre vocabulary slides
Gre vocabulary slides
 
Carey Business School
Carey Business SchoolCarey Business School
Carey Business School
 
My project
My projectMy project
My project
 
Gmat Ppt
Gmat PptGmat Ppt
Gmat Ppt
 

Similar to Cr concepts the best resource for gmat cr from ivy-gmat (sandeep gupta)

Business Communication & Value Science
Business Communication & Value ScienceBusiness Communication & Value Science
Business Communication & Value ScienceKathirvel Ayyaswamy
 
Share & Care July 2013
Share & Care July 2013Share & Care July 2013
Share & Care July 2013A V MANIVANNAN
 
Creating strategy in a small business
Creating strategy in a small businessCreating strategy in a small business
Creating strategy in a small businessEd Kless
 
Et young leaders programme
Et young leaders programmeEt young leaders programme
Et young leaders programmeVINOD SHANKAR
 
Et young leaders programme
Et young leaders programmeEt young leaders programme
Et young leaders programmeIFFCO Canada
 
1st year Resume Writing Guidelines for Students.pdf
1st year Resume Writing Guidelines for Students.pdf1st year Resume Writing Guidelines for Students.pdf
1st year Resume Writing Guidelines for Students.pdfRuchi645726
 
Startups : A New Milestone in Indian Scenario
Startups : A New Milestone in Indian ScenarioStartups : A New Milestone in Indian Scenario
Startups : A New Milestone in Indian ScenarioVARUN KUMAR
 
LETTER MEMO CIRCULAR.pptx
LETTER MEMO CIRCULAR.pptxLETTER MEMO CIRCULAR.pptx
LETTER MEMO CIRCULAR.pptxsmilesmorales
 
Sym model of communication - A communication model created by Arvind
Sym model of communication - A communication model created by ArvindSym model of communication - A communication model created by Arvind
Sym model of communication - A communication model created by ArvindArvind Bhardwaj [AB]
 
SearchLove London 2018 - Dom Woodman - A year of SEO split testing changed ho...
SearchLove London 2018 - Dom Woodman - A year of SEO split testing changed ho...SearchLove London 2018 - Dom Woodman - A year of SEO split testing changed ho...
SearchLove London 2018 - Dom Woodman - A year of SEO split testing changed ho...Distilled
 
SearchLove San Diego - Dom Woodman - A Year of SEO Split Testing Changed How ...
SearchLove San Diego - Dom Woodman - A Year of SEO Split Testing Changed How ...SearchLove San Diego - Dom Woodman - A Year of SEO Split Testing Changed How ...
SearchLove San Diego - Dom Woodman - A Year of SEO Split Testing Changed How ...Distilled
 
HMD project.docx
HMD project.docxHMD project.docx
HMD project.docxbkbk37
 
Feedback the mechanics inspiration for you to do feedback workshops june 2020...
Feedback the mechanics inspiration for you to do feedback workshops june 2020...Feedback the mechanics inspiration for you to do feedback workshops june 2020...
Feedback the mechanics inspiration for you to do feedback workshops june 2020...Nels Karsvang
 
Job Applications 101 - Pakistan
Job Applications 101 - PakistanJob Applications 101 - Pakistan
Job Applications 101 - PakistanTalalK10
 
Job App 101 - Pakistan
Job App 101 - PakistanJob App 101 - Pakistan
Job App 101 - PakistanTalalK10
 
How a year of SEO split testing changed how I thought SEO worked
How a year of SEO split testing changed how I thought SEO workedHow a year of SEO split testing changed how I thought SEO worked
How a year of SEO split testing changed how I thought SEO workedDominic Woodman
 

Similar to Cr concepts the best resource for gmat cr from ivy-gmat (sandeep gupta) (20)

Business Communication & Value Science
Business Communication & Value ScienceBusiness Communication & Value Science
Business Communication & Value Science
 
Share & Care July 2013
Share & Care July 2013Share & Care July 2013
Share & Care July 2013
 
Letterwriting types
Letterwriting typesLetterwriting types
Letterwriting types
 
Creating strategy in a small business
Creating strategy in a small businessCreating strategy in a small business
Creating strategy in a small business
 
Et young leaders programme
Et young leaders programmeEt young leaders programme
Et young leaders programme
 
Et young leaders programme
Et young leaders programmeEt young leaders programme
Et young leaders programme
 
The case study method
The case study methodThe case study method
The case study method
 
1st year Resume Writing Guidelines for Students.pdf
1st year Resume Writing Guidelines for Students.pdf1st year Resume Writing Guidelines for Students.pdf
1st year Resume Writing Guidelines for Students.pdf
 
Triangular Perspectives
Triangular PerspectivesTriangular Perspectives
Triangular Perspectives
 
Startups : A New Milestone in Indian Scenario
Startups : A New Milestone in Indian ScenarioStartups : A New Milestone in Indian Scenario
Startups : A New Milestone in Indian Scenario
 
LETTER MEMO CIRCULAR.pptx
LETTER MEMO CIRCULAR.pptxLETTER MEMO CIRCULAR.pptx
LETTER MEMO CIRCULAR.pptx
 
Sym model of communication - A communication model created by Arvind
Sym model of communication - A communication model created by ArvindSym model of communication - A communication model created by Arvind
Sym model of communication - A communication model created by Arvind
 
GRE
GREGRE
GRE
 
SearchLove London 2018 - Dom Woodman - A year of SEO split testing changed ho...
SearchLove London 2018 - Dom Woodman - A year of SEO split testing changed ho...SearchLove London 2018 - Dom Woodman - A year of SEO split testing changed ho...
SearchLove London 2018 - Dom Woodman - A year of SEO split testing changed ho...
 
SearchLove San Diego - Dom Woodman - A Year of SEO Split Testing Changed How ...
SearchLove San Diego - Dom Woodman - A Year of SEO Split Testing Changed How ...SearchLove San Diego - Dom Woodman - A Year of SEO Split Testing Changed How ...
SearchLove San Diego - Dom Woodman - A Year of SEO Split Testing Changed How ...
 
HMD project.docx
HMD project.docxHMD project.docx
HMD project.docx
 
Feedback the mechanics inspiration for you to do feedback workshops june 2020...
Feedback the mechanics inspiration for you to do feedback workshops june 2020...Feedback the mechanics inspiration for you to do feedback workshops june 2020...
Feedback the mechanics inspiration for you to do feedback workshops june 2020...
 
Job Applications 101 - Pakistan
Job Applications 101 - PakistanJob Applications 101 - Pakistan
Job Applications 101 - Pakistan
 
Job App 101 - Pakistan
Job App 101 - PakistanJob App 101 - Pakistan
Job App 101 - Pakistan
 
How a year of SEO split testing changed how I thought SEO worked
How a year of SEO split testing changed how I thought SEO workedHow a year of SEO split testing changed how I thought SEO worked
How a year of SEO split testing changed how I thought SEO worked
 

Recently uploaded

18-04-UA_REPORT_MEDIALITERAСY_INDEX-DM_23-1-final-eng.pdf
18-04-UA_REPORT_MEDIALITERAСY_INDEX-DM_23-1-final-eng.pdf18-04-UA_REPORT_MEDIALITERAСY_INDEX-DM_23-1-final-eng.pdf
18-04-UA_REPORT_MEDIALITERAСY_INDEX-DM_23-1-final-eng.pdfssuser54595a
 
Science 7 - LAND and SEA BREEZE and its Characteristics
Science 7 - LAND and SEA BREEZE and its CharacteristicsScience 7 - LAND and SEA BREEZE and its Characteristics
Science 7 - LAND and SEA BREEZE and its CharacteristicsKarinaGenton
 
POINT- BIOCHEMISTRY SEM 2 ENZYMES UNIT 5.pptx
POINT- BIOCHEMISTRY SEM 2 ENZYMES UNIT 5.pptxPOINT- BIOCHEMISTRY SEM 2 ENZYMES UNIT 5.pptx
POINT- BIOCHEMISTRY SEM 2 ENZYMES UNIT 5.pptxSayali Powar
 
Introduction to ArtificiaI Intelligence in Higher Education
Introduction to ArtificiaI Intelligence in Higher EducationIntroduction to ArtificiaI Intelligence in Higher Education
Introduction to ArtificiaI Intelligence in Higher Educationpboyjonauth
 
Grant Readiness 101 TechSoup and Remy Consulting
Grant Readiness 101 TechSoup and Remy ConsultingGrant Readiness 101 TechSoup and Remy Consulting
Grant Readiness 101 TechSoup and Remy ConsultingTechSoup
 
CARE OF CHILD IN INCUBATOR..........pptx
CARE OF CHILD IN INCUBATOR..........pptxCARE OF CHILD IN INCUBATOR..........pptx
CARE OF CHILD IN INCUBATOR..........pptxGaneshChakor2
 
Alper Gobel In Media Res Media Component
Alper Gobel In Media Res Media ComponentAlper Gobel In Media Res Media Component
Alper Gobel In Media Res Media ComponentInMediaRes1
 
A Critique of the Proposed National Education Policy Reform
A Critique of the Proposed National Education Policy ReformA Critique of the Proposed National Education Policy Reform
A Critique of the Proposed National Education Policy ReformChameera Dedduwage
 
Micromeritics - Fundamental and Derived Properties of Powders
Micromeritics - Fundamental and Derived Properties of PowdersMicromeritics - Fundamental and Derived Properties of Powders
Micromeritics - Fundamental and Derived Properties of PowdersChitralekhaTherkar
 
Measures of Central Tendency: Mean, Median and Mode
Measures of Central Tendency: Mean, Median and ModeMeasures of Central Tendency: Mean, Median and Mode
Measures of Central Tendency: Mean, Median and ModeThiyagu K
 
The Most Excellent Way | 1 Corinthians 13
The Most Excellent Way | 1 Corinthians 13The Most Excellent Way | 1 Corinthians 13
The Most Excellent Way | 1 Corinthians 13Steve Thomason
 
MENTAL STATUS EXAMINATION format.docx
MENTAL     STATUS EXAMINATION format.docxMENTAL     STATUS EXAMINATION format.docx
MENTAL STATUS EXAMINATION format.docxPoojaSen20
 
Introduction to AI in Higher Education_draft.pptx
Introduction to AI in Higher Education_draft.pptxIntroduction to AI in Higher Education_draft.pptx
Introduction to AI in Higher Education_draft.pptxpboyjonauth
 
Arihant handbook biology for class 11 .pdf
Arihant handbook biology for class 11 .pdfArihant handbook biology for class 11 .pdf
Arihant handbook biology for class 11 .pdfchloefrazer622
 
microwave assisted reaction. General introduction
microwave assisted reaction. General introductionmicrowave assisted reaction. General introduction
microwave assisted reaction. General introductionMaksud Ahmed
 
BASLIQ CURRENT LOOKBOOK LOOKBOOK(1) (1).pdf
BASLIQ CURRENT LOOKBOOK  LOOKBOOK(1) (1).pdfBASLIQ CURRENT LOOKBOOK  LOOKBOOK(1) (1).pdf
BASLIQ CURRENT LOOKBOOK LOOKBOOK(1) (1).pdfSoniaTolstoy
 
Solving Puzzles Benefits Everyone (English).pptx
Solving Puzzles Benefits Everyone (English).pptxSolving Puzzles Benefits Everyone (English).pptx
Solving Puzzles Benefits Everyone (English).pptxOH TEIK BIN
 

Recently uploaded (20)

18-04-UA_REPORT_MEDIALITERAСY_INDEX-DM_23-1-final-eng.pdf
18-04-UA_REPORT_MEDIALITERAСY_INDEX-DM_23-1-final-eng.pdf18-04-UA_REPORT_MEDIALITERAСY_INDEX-DM_23-1-final-eng.pdf
18-04-UA_REPORT_MEDIALITERAСY_INDEX-DM_23-1-final-eng.pdf
 
Science 7 - LAND and SEA BREEZE and its Characteristics
Science 7 - LAND and SEA BREEZE and its CharacteristicsScience 7 - LAND and SEA BREEZE and its Characteristics
Science 7 - LAND and SEA BREEZE and its Characteristics
 
POINT- BIOCHEMISTRY SEM 2 ENZYMES UNIT 5.pptx
POINT- BIOCHEMISTRY SEM 2 ENZYMES UNIT 5.pptxPOINT- BIOCHEMISTRY SEM 2 ENZYMES UNIT 5.pptx
POINT- BIOCHEMISTRY SEM 2 ENZYMES UNIT 5.pptx
 
Introduction to ArtificiaI Intelligence in Higher Education
Introduction to ArtificiaI Intelligence in Higher EducationIntroduction to ArtificiaI Intelligence in Higher Education
Introduction to ArtificiaI Intelligence in Higher Education
 
Grant Readiness 101 TechSoup and Remy Consulting
Grant Readiness 101 TechSoup and Remy ConsultingGrant Readiness 101 TechSoup and Remy Consulting
Grant Readiness 101 TechSoup and Remy Consulting
 
Model Call Girl in Tilak Nagar Delhi reach out to us at 🔝9953056974🔝
Model Call Girl in Tilak Nagar Delhi reach out to us at 🔝9953056974🔝Model Call Girl in Tilak Nagar Delhi reach out to us at 🔝9953056974🔝
Model Call Girl in Tilak Nagar Delhi reach out to us at 🔝9953056974🔝
 
CARE OF CHILD IN INCUBATOR..........pptx
CARE OF CHILD IN INCUBATOR..........pptxCARE OF CHILD IN INCUBATOR..........pptx
CARE OF CHILD IN INCUBATOR..........pptx
 
Alper Gobel In Media Res Media Component
Alper Gobel In Media Res Media ComponentAlper Gobel In Media Res Media Component
Alper Gobel In Media Res Media Component
 
A Critique of the Proposed National Education Policy Reform
A Critique of the Proposed National Education Policy ReformA Critique of the Proposed National Education Policy Reform
A Critique of the Proposed National Education Policy Reform
 
Micromeritics - Fundamental and Derived Properties of Powders
Micromeritics - Fundamental and Derived Properties of PowdersMicromeritics - Fundamental and Derived Properties of Powders
Micromeritics - Fundamental and Derived Properties of Powders
 
Measures of Central Tendency: Mean, Median and Mode
Measures of Central Tendency: Mean, Median and ModeMeasures of Central Tendency: Mean, Median and Mode
Measures of Central Tendency: Mean, Median and Mode
 
The Most Excellent Way | 1 Corinthians 13
The Most Excellent Way | 1 Corinthians 13The Most Excellent Way | 1 Corinthians 13
The Most Excellent Way | 1 Corinthians 13
 
Model Call Girl in Bikash Puri Delhi reach out to us at 🔝9953056974🔝
Model Call Girl in Bikash Puri  Delhi reach out to us at 🔝9953056974🔝Model Call Girl in Bikash Puri  Delhi reach out to us at 🔝9953056974🔝
Model Call Girl in Bikash Puri Delhi reach out to us at 🔝9953056974🔝
 
Staff of Color (SOC) Retention Efforts DDSD
Staff of Color (SOC) Retention Efforts DDSDStaff of Color (SOC) Retention Efforts DDSD
Staff of Color (SOC) Retention Efforts DDSD
 
MENTAL STATUS EXAMINATION format.docx
MENTAL     STATUS EXAMINATION format.docxMENTAL     STATUS EXAMINATION format.docx
MENTAL STATUS EXAMINATION format.docx
 
Introduction to AI in Higher Education_draft.pptx
Introduction to AI in Higher Education_draft.pptxIntroduction to AI in Higher Education_draft.pptx
Introduction to AI in Higher Education_draft.pptx
 
Arihant handbook biology for class 11 .pdf
Arihant handbook biology for class 11 .pdfArihant handbook biology for class 11 .pdf
Arihant handbook biology for class 11 .pdf
 
microwave assisted reaction. General introduction
microwave assisted reaction. General introductionmicrowave assisted reaction. General introduction
microwave assisted reaction. General introduction
 
BASLIQ CURRENT LOOKBOOK LOOKBOOK(1) (1).pdf
BASLIQ CURRENT LOOKBOOK  LOOKBOOK(1) (1).pdfBASLIQ CURRENT LOOKBOOK  LOOKBOOK(1) (1).pdf
BASLIQ CURRENT LOOKBOOK LOOKBOOK(1) (1).pdf
 
Solving Puzzles Benefits Everyone (English).pptx
Solving Puzzles Benefits Everyone (English).pptxSolving Puzzles Benefits Everyone (English).pptx
Solving Puzzles Benefits Everyone (English).pptx
 

Cr concepts the best resource for gmat cr from ivy-gmat (sandeep gupta)

  • 1. © Sandeep Gupta – Ivy-GMAT, Koramangala Bangalore. sandeepgupta01@gmail.com, 9739561394 Ultimate CR Strategies for the GMAT by Sandeep Gupta (Ivy-GMAT, Koramangala Bangalore) sandeepgupta01@gmail.com 9739561394
  • 2. © Sandeep Gupta – Ivy-GMAT, Koramangala Bangalore. sandeepgupta01@gmail.com, 9739561394 Further Resources: 1. Manhattan GMAT CR Guide – 4th Edition 2. CR Ultimate Collection – a set of GMAT CR Questions (by Sandeep Gupta (Ivy-GMAT)) – Must Solve 3. 700-800 CR Questions – a set of GMAT CR Questions (by Sandeep Gupta (Ivy-GMAT)) – Must Solve 4. OG 12, 11, 10, OG Verbal Review 1st and 2nd Editions 5. GMAT-Prep Verbal Forum Questions from ManhattanGMAT.com 6. 1000 CR (not necessary, though)
  • 3. © Sandeep Gupta – Ivy-GMAT, Koramangala Bangalore. sandeepgupta01@gmail.com, 9739561394 CR Strategies Note: Refer to the NOTES at the bottom of each slide for explanations.
  • 4. © Sandeep Gupta – Ivy-GMAT, Koramangala Bangalore. sandeepgupta01@gmail.com, 9739561394 CR Questions… the structure • A short passage (called the argument) followed by a specific question related to the passage with 5 answer choices • Directions: For this question type, select the best of the given answer choices. For example: Most serious students are happy students, and most serious students go to graduate school. Furthermore, all students who go to graduate school are overworked. Which one of the following can be properly inferred from the statements above? (A) Most overworked students are happy students. (B) Some happy students are overworked. (C) All overworked students are serious students. (D) Some unhappy students go to graduate school. (E) All serious students are overworked.
  • 5. © Sandeep Gupta – Ivy-GMAT, Koramangala Bangalore. sandeepgupta01@gmail.com, 9739561394 The Nitty-gritty • CR is largely composed of arguments and arguments parts • It checks skills of Logical Thinking and Critical Evaluation. One needs eye for detail. • There are 13-14 questions interspersed throughout the Verbal Section • It is possibly the best tool to check Managerial Aptitude
  • 6. © Sandeep Gupta – Ivy-GMAT, Koramangala Bangalore. sandeepgupta01@gmail.com, 9739561394 Argument Parts • An argument (passage) generally consists of – FACT (S) & CONCLUSION (s) – Facts support the conclusion: also called Assertion, Evidence, Premise, Proof, Supposition, Data, Information, Research, Consideration, Pattern, Explanation, Justification, Situation, Circumstance, Reasoning, Line of Reasoning, Generalization, Observation, Support, Example, Finding, Phenomenon, Illustration, Reason, To Acknowledge, etc. – Conclusion is what the author tries to establish using facts. Also known as Judgment, Opinion, Suggestion, View, Idea, Belief, Proposal, Warning, Forecast, Claim, Stand, Prediction, Hypothesis, POSITION, Stance, Point, Main Point, Inference, Implication, Generalization, Contention, Consequence, Argument, To Posit, etc.
  • 7. © Sandeep Gupta – Ivy-GMAT, Koramangala Bangalore. sandeepgupta01@gmail.com, 9739561394 So, finding the correct conclusion is the half the job done in CR. If we can identify the conclusion, everything else becomes FACT automatically.
  • 8. © Sandeep Gupta – Ivy-GMAT, Koramangala Bangalore. sandeepgupta01@gmail.com, 9739561394 1.In a study, infant monkeys given a choice between two surrogate mothers—a bare wire structure equipped with a milk bottle, or a soft, suede-covered wire structure equipped with a milk bottle—unhesitatingly chose the latter. When given a choice between a bare wire structure equipped with a milk bottle and a soft, suede-covered wire structure lacking a milk bottle, they unhesitatingly chose the former. Find the conclusion
  • 9. © Sandeep Gupta – Ivy-GMAT, Koramangala Bangalore. sandeepgupta01@gmail.com, 9739561394 2. Journalist: A free marketplace of ideas ensures that all ideas get a fair hearing. Even ideas tainted with prejudice and malice can prompt beneficial outcomes. In most countries, however, the government is responsible for over half the information released to the public through all media. For this reason, the power of governments over information needs to be curtailed. Everyone grants that governments should not suppress free expression, yet governments continue to construct near monopolies on the publication and dissemination of enormous amounts of information. Find the conclusion
  • 10. © Sandeep Gupta – Ivy-GMAT, Koramangala Bangalore. sandeepgupta01@gmail.com, 9739561394 3. Last month OCF, Inc., announced what it described as a unique new product: an adjustable computer workstation. Three days later ErgoTech unveiled an almost identical product. The two companies claim that the similarities are coincidental and occurred because the designers independently reached the same solution to the same problem. The similarities are too fundamental to be mere coincidence, however. The two products not only look alike, but they also work alike. Both are oddly shaped with identically placed control panels with the same types of controls. Both allow the same types of adjustments and the same types of optional enhancements. Find the conclusion
  • 11. © Sandeep Gupta – Ivy-GMAT, Koramangala Bangalore. sandeepgupta01@gmail.com, 9739561394 4. It is well known that many species adapt to their environment, but it is usually assumed that only the most highly evolved species alter their environment in ways that aid their own survival. However, this characteristic is actually quite common. Certain species of plankton, for example, generate a gas that is converted in the atmosphere into particles of sulfate. These particles cause water vapor to condense, thus forming clouds. Indeed, the formation of clouds over the ocean largely depends on the presence of these particles. More cloud cover means more sunlight is reflected, and so the Earth absorbs less heat. Thus plankton cause the surface of the Earth to be cooler and this benefits the plankton. Find the conclusion (s)
  • 12. © Sandeep Gupta – Ivy-GMAT, Koramangala Bangalore. sandeepgupta01@gmail.com, 9739561394 5. Rain-soaked soil contains less oxygen than does drier soil. The roots of melon plants perform less efficiently under the low-oxygen conditions present in rain-soaked soil. When the efficiency of melon roots is impaired, the roots do not supply sufficient amounts of the proper nutrients for the plants to perform photosynthesis at their usual levels. It follows that melon plants have a lower- than-usual rate of photosynthesis when their roots are in rain-soaked soil. When the photosynthesis of the plants slows, sugar stored in the fruits is drawn off to supply the plants with energy. Therefore, ripe melons harvested after a prolonged period of heavy rain should be less sweet than other ripe melons. In the argument given, the two portions in boldface play which of the following roles? • The first states the conclusion of the argument as a whole; the second provides support for that conclusion. • The first provides support for the conclusion of the argument as a whole; the second provides evidence that supports an objection to that conclusion. • The first provides support for an intermediate conclusion that supports a further conclusion stated in the argument; the second states that intermediate conclusion. • The first serves as an intermediate conclusion that supports a further conclusion stated in the argument; the second states the position that the argument as a whole opposes. • The first states the position that the argument as a whole opposes; the second supports the conclusion of the argument.
  • 13. © Sandeep Gupta – Ivy-GMAT, Koramangala Bangalore. sandeepgupta01@gmail.com, 9739561394 CR – types of Questions • Conclusion / Inference • Assumptions • Weaken • Strengthen • Evaluate • Paradox • Boldface • Miscellaneous
  • 14. © Sandeep Gupta – Ivy-GMAT, Koramangala Bangalore. sandeepgupta01@gmail.com, 9739561394 Type 1: Conclusion / Inference 1. Which of the following conclusions / assertions / inferences / statements is most strongly supported by the passage? 2. Which of the following must be true as per the passage? 3. Which of the following can be inferred from the passage? 4. The author is arguing that… 5. The main point of the passage is that 6. Which of the following can complete the passage? 7. Which of the following is an example of the … given in the passage?
  • 15. © Sandeep Gupta – Ivy-GMAT, Koramangala Bangalore. sandeepgupta01@gmail.com, 9739561394 How to solve “conclusion” questions? • Word Matching and TRANSLATION are the most critical skills. • Any statement that is inconsistent with the passage is always wrong. • The correct answer MUST be true, not MAY be true. • Elimination is the best strategy. Don’t select the correct answer. Eliminate wrong ones.
  • 16. © Sandeep Gupta – Ivy-GMAT, Koramangala Bangalore. sandeepgupta01@gmail.com, 9739561394 Choose the correct conclusion or conclusions (any number of answers may be right) The minimum voting age for males in district X is 18 years. A. Every male above 18 can vote in district X. B. Someone who is not 18 can not vote in district X. C. A male who is not 18 can not vote in district X. D. For a male to be eligible to vote in district X, he must be 18 years old. E. For a male to be eligible to vote in district X, he must be at least 18 years old.
  • 17. © Sandeep Gupta – Ivy-GMAT, Koramangala Bangalore. sandeepgupta01@gmail.com, 9739561394 Choose the correct conclusion or conclusions (any number of answers may be right) Every male above 18 is allowed to vote in county X. A. A male who is not 18 can not vote in county X. B. For a male to be eligible to vote in county X, he must be more than 18 years old. C. For a male to be eligible to vote in county X, he must be at least 18 years old. D. Males below 18 are not allowed to vote in county X. E. Age is the only criterion by which the eligibility to vote for males is decided in county X.
  • 18. © Sandeep Gupta – Ivy-GMAT, Koramangala Bangalore. sandeepgupta01@gmail.com, 9739561394 Understanding Conditional “If-then” Consider the following statement: If one plays in the rain, one gets cold. Now consider which one follows logically from above: 1. If one doesn’t play in the rain, one doesn’t get cold. 2. If one has got cold, one has played in the rain. 3. If one hasn’t got cold, one hasn’t played in the rain.
  • 19. © Sandeep Gupta – Ivy-GMAT, Koramangala Bangalore. sandeepgupta01@gmail.com, 9739561394 Understanding Conditional “If-then” So if X → Y then the following conclusions are wrong: 1. Y → X 2. X (NOT) → Y (NOT) Only the following conclusion is right: Y (not) → X (not)
  • 20. © Sandeep Gupta – Ivy-GMAT, Koramangala Bangalore. sandeepgupta01@gmail.com, 9739561394 1. Mystery stories often feature a brilliant detective and the detective’s dull companion. Clues are presented in the story, and the companion wrongly infers an inaccurate solution to the mystery using the same clues that the detective uses to deduce the correct solution. Thus, the author’s strategy of including the dull companion gives readers a chance to solve the mystery while also diverting them from the correct solution. Which of the following conclusions can be correctly drawn from the passage above? A. Most mystery stories feature a brilliant detective who solves the mystery presented in the story. B. Mystery readers often solve the mystery in a story simply by spotting the mistakes in the reasoning of the detective’s dull companion in that story. C. Some mystery stories give readers enough clues to infer the correct solution to the mystery. D. The actions of the brilliant detective in a mystery story rarely divert readers from the actions of the detective’s dull companion. E. The detective’s dull companion in a mystery story generally uncovers the misleading clues that divert readers from the mystery’s correct solution.
  • 21. © Sandeep Gupta – Ivy-GMAT, Koramangala Bangalore. sandeepgupta01@gmail.com, 9739561394 Some Modifier Words • All = 100 • Some = 1 to 99 or even 100 • Most = 51 to 99 or even 100 • Not all = 0 to 99 • None = 0 • Often = Not defined • Many = Not defined • Much = Not defined
  • 22. © Sandeep Gupta – Ivy-GMAT, Koramangala Bangalore. sandeepgupta01@gmail.com, 9739561394 2. Newtonian physics dominated science for over two centuries. It found consistently successful application, becoming one of the most highly substantiated and accepted theories in the history of science. Nevertheless, Einstein’s theories came to show the fundamental limits of Newtonian physics and to surpass the Newtonian view in the early 1900s, giving rise once again to a physics that has so far enjoyed wide success. Which one of the following logically follows from the statements above? A. The history of physics is characterized by a pattern of one successful theory subsequently surpassed by another. B. Long-standing success or substantiation of a theory of physics is no guarantee that the theory will continue to be dominant indefinitely. C. Every theory of physics, no matter how successful, is eventually surpassed by one that is more successful. D. Once a theory of physics is accepted, it will remain dominant for centuries. E. If a long-accepted theory of physics is surpassed, it must be surpassed by a theory that is equally successful.
  • 23. © Sandeep Gupta – Ivy-GMAT, Koramangala Bangalore. sandeepgupta01@gmail.com, 9739561394 3. People with serious financial problems are so worried about money that they cannot be happy. Their misery makes everyone close to them—family, friends, and colleagues— unhappy as well. Only if their financial problems are solved can they and those around them be happy. Which one of the following statements can be properly inferred from the passage? A. Only serious problems make people unhappy. B. People who solve their serious financial problems will be happy. C.People who do not have serious financial problems will be happy. D.If people are unhappy, they have serious financial problems. E. If people are happy, they do not have serious financial problems.
  • 24. © Sandeep Gupta – Ivy-GMAT, Koramangala Bangalore. sandeepgupta01@gmail.com, 9739561394 4. The head baker at Barry’s Bagels can either purchase flour in-person from the local flour mill, Larry’s Local Mill, or order a shipment of flour from an out-of-state mill, Isadore’s Interstate Mill. The cost of the flour from Isadore’s Interstate Mill is 10 percent less than the cost of the flour from Larry’s Local Mill. Even after shipping and handling fees are added, it is still cheaper to order flour that has to be shipped from Isadore’s than to buy flour locally from Larry’s. The statements above, if true, best support which of the following assertions? A. Production costs at Isadore’s Interstate Mill are 10 percent below those at Larry’s Local Mill. B. Buying flour from Isadore’s Interstate Mill will eliminate 10 percent of the local flour mill jobs. C. The shipping and handling fees for a batch of flour purchased from Isadore’s Interstate Mill are less than 10 percent of the cost of an identical batch of flour purchased from Larry’s Local Mill. D. The shipping and handling fees for a batch of flour purchased from Isadore’s Interstate Mill are more than 10 percent of the cost of Isadore’s flour. E. Isadore’s Interstate Mill produces flour 10% more efficiently than Larry’s Local Mill does.
  • 25. © Sandeep Gupta – Ivy-GMAT, Koramangala Bangalore. sandeepgupta01@gmail.com, 9739561394 5. Last January, in an attempt to lower the number of traffic fatalities, the state legislature passed its “Click It or Ticket” law. Under the new law, motorists can be pulled over and ticketed for not wearing their seat belts, even if an additional driving infraction has not been committed. Lawyers and citizens’ groups are already protesting the law, saying it unfairly infringes on the rights of the state’s drivers. Law enforcement groups counter these claims by stating that the new regulations will save countless additional lives. Which of the following inferences is best supported by the passage above? A. Prior to the “Click It or Ticket” law, motorists could not be stopped simply for not wearing a seat belt. B. The “Click It or Ticket” law violates current search and seizure laws. C. Laws similar to “Click It or Ticket” have effectively reduced traffic fatalities in a number of states. D. The previous seatbelt laws were ineffective in saving lives. E. Law enforcement groups, rather than citizens groups, should determine how to best ensure the safety of motorists.
  • 26. © Sandeep Gupta – Ivy-GMAT, Koramangala Bangalore. sandeepgupta01@gmail.com, 9739561394 6. Meteorite explosions in the Earth’s atmosphere as large as the one that destroyed forests in Siberia, with approximately the force of a twelve- megaton nuclear blast, occur about once a century. The response of highly automated systems controlled by complex computer programs to unexpected circumstances is unpredictable. Which of the following conclusions can most properly be drawn, if the statements above are true, about a highly automated nuclear- missile defense system controlled by a complex computer program? A. Within a century after its construction, the system would react inappropriately and might accidentally start a nuclear war. B. The system would be destroyed if an explosion of a large meteorite occurred in the Earth’s atmosphere. C. It would be impossible for the system to distinguish the explosion of a large meteorite from the explosion of a nuclear weapon. D. Whether the system would respond inappropriately to the explosion of a large meteorite would depend on the location of the blast. E. It is not certain what the system’s response to the explosion of a large meteorite would be, if its designers did not plan for such a contingency.
  • 27. © Sandeep Gupta – Ivy-GMAT, Koramangala Bangalore. sandeepgupta01@gmail.com, 9739561394 7. Laws requiring the use of headlights during daylight hours can prevent automobile collisions. However, since daylight visibility is worse in countries farther from the equator, any such laws would obviously be more effective in preventing collisions in those countries. In fact, the only countries that actually have such laws are farther from the equator than is the continental United States. Which of the following conclusions could be most properly drawn from the information given above? A. Drivers in the continental United States who used their headlines during the day would be just as likely to become involved in a collision as would drivers who did not use their headlights. B. In many countries that are farther from the equator than is the continental United States poor daylight visibility is the single most important factor in automobile collisions. C. The proportion of automobile collisions that occur in the daytime is greater in the continental United States than in the countries that have daytime headlight laws. D. Fewer automobile collisions probably occur each year in countries that have daytime headlight laws than occur within the continental United States. E. Daytime headlight laws would probably do less to prevent automobile collisions in the continental United States than they do in the countries that have the laws.
  • 28. © Sandeep Gupta – Ivy-GMAT, Koramangala Bangalore. sandeepgupta01@gmail.com, 9739561394 8. High levels of fertilizer and pesticides, needed when farmers try to produce high yield of the same crop year after year, pollute water supplies. Experts therefore urge farmers to diversify their crops and to rotate their plantings yearly. To receive governmental price-support benefits for a crop, farmers must have produced that same crop for the past several years. The statements above, if true, best support which of the following conclusions? A. The rules for governmental support of farm prices work against efforts to reduce water pollution. B. The only solution to the problem of water pollution from fertilizers and pesticides is to take farmland out of production. C. Farmers can continue to make a profit by rotating diverse crops, thus reducing costs for chemicals, but not by planting the same crop each year. D. New farming techniques will be developed to make it possible for farmers to reduce the application of fertilizers and pesticides. E. Governmental price supports for farm products are set at levels that are not high enough to allow farmers to get out of debt.
  • 29. © Sandeep Gupta – Ivy-GMAT, Koramangala Bangalore. sandeepgupta01@gmail.com, 9739561394 9. United States hospitals have traditionally relied primarily on revenues from paying patients to offset losses from unreimbursed care. Almost all paying patients now rely on governmental or private health insurance to pay hospital bills. Recently, insurers have been strictly limiting what they pay hospitals for the care of insured patients to amounts at or below actual costs. Which of the following conclusions is best supported by the information above? A. Although the advance of technology has made expensive medical procedures available to the wealthy, such procedures are out of the reach of low-income patients. B. If hospitals do not find ways to raising additional income for unreimbursed care, they must either deny some of that care of suffer losses if they give it. C. Some patients have incomes too high for eligibility for governmental health insurance but are unable to afford private insurance for hospital care. D. If the hospitals reduce their costs in providing care, insurance companies will maintain the current level of reimbursement, thereby providing more funds for unreimbursed care. E. Even though philanthropic donations have traditionally provided some support for the hospitals, such donations are at present declining.
  • 30. © Sandeep Gupta – Ivy-GMAT, Koramangala Bangalore. sandeepgupta01@gmail.com, 9739561394 10. Which of the following best completes the passage below? In a survey of job applicants, two-fifths admitted to being at least a little dishonest. However, the survey may underestimate the proportion of job applicants who are dishonest, because____. A. some dishonest people taking the survey might have claimed on the survey to be honest B. some generally honest people taking the survey might have claimed on the survey to be dishonest C. some people who claimed on the survey to be at least a little dishonest may be very dishonest D. some people who claimed on the survey to be dishonest may have been answering honestly E. some people who are not job applicants are probably at least a little dishonest
  • 31. © Sandeep Gupta – Ivy-GMAT, Koramangala Bangalore. sandeepgupta01@gmail.com, 9739561394 CR Type 2: Boldface Questions • Structure: – Usually in a long paragraph, two statements (or statement parts) are written in BOLD, and one has to choose the option that tells the function (role) played by these bold parts. – Involves Argument structure concepts – one should be able to tell which is a fact, conclusion and how the argument is structured.
  • 32. © Sandeep Gupta – Ivy-GMAT, Koramangala Bangalore. sandeepgupta01@gmail.com, 9739561394 – One must know all the terms that are commonly used for FACT and CONCLUSION. – Facts support the conclusion: also called Assertion, Evidence, Premise, Proof, Supposition, Data, Information, Research, Consideration, Pattern, Explanation, Justification, Situation, Circumstance, Reasoning, Line of Reasoning, Generalization, Observation, Support, Example, Finding, Phenomenon, Illustration, Reason, To Acknowledge, etc. – Conclusion is what the author tries to establish using facts. Also known as Judgment, Opinion, View, Suggestion, Idea, Belief, Proposal, Warning, Forecast, Claim, Stand, Prediction, Hypothesis, POSITION, Stance, Point, Main Point, Inference, Implication, Generalization, Contention, Consequence, Argument, To Posit, etc.
  • 33. © Sandeep Gupta – Ivy-GMAT, Koramangala Bangalore. sandeepgupta01@gmail.com, 9739561394 • Sometimes the argument may contain two conclusions. • The statement that can be used to prove the main conclusion is usually the sub-conclusion. • In such a situation, the ONE LINE CRISP MESSAGE of the argument is the MAIN conclusion and the other is the sub-conclusion. Asking yourself about author’s position will help you get the main conclusion. • Sometimes, the two conclusions may be contradictory to each other (especially when two sides are involved – author and the critics etc.)
  • 34. © Sandeep Gupta – Ivy-GMAT, Koramangala Bangalore. sandeepgupta01@gmail.com, 9739561394 Steps to solve BF questions – STEP 1: Identify the conclusion (s). Do not jump to choices. – STEP 2: Map the argument (F/F/C… etc.) and have the flow in mind. Which statement supports the conclusion and which supports some alternative conclusion. – STEP 3: Go to choices and eliminate.
  • 35. © Sandeep Gupta – Ivy-GMAT, Koramangala Bangalore. sandeepgupta01@gmail.com, 9739561394 1. Astronomer: Observations of the Shoemaker-Levi comet on its collision course with Jupiter showed that the comet broke into fragments before entering Jupiter’s atmosphere in 1994, but they did not show how big those fragments were. Nevertheless, some indication of their size can be inferred from spectrographic analyses of Jupiter’s outer atmosphere. After the fragments’ entry, these analyses revealed unprecedented traces of sulfur. The fragments themselves almost certainly contained no sulfur, but astronomers believe that the cloud layer below Jupiter’s outer atmosphere does contain sulfur. Since sulfur would have seeped into the outer atmosphere if comet fragments had penetrated this cloud layer, it is likely that some of the fragments were at least large enough to have passed through Jupiter’s outer atmosphere without being burned up. In the astronomer’s argument, the two portions in BOLDFACE play which of the following roles? A. The first is a claim that the astronomer seeks to show is true; the second acknowledges a consideration that weighs against the truth of that claim. B. The first is a claim that the astronomer seeks to show is true; the second provides evidence in support of the truth of that claim. C. The first and the second are each considerations advanced in support of the conclusion of the argument. D. The first provides evidence in support of the conclusion of the argument; the second is that conclusion. E. The first is a circumstance for which the astronomer seeks to provide an explanation; the second acknowledges a consideration that weighs against the explanation provided by the astronomer.
  • 36. © Sandeep Gupta – Ivy-GMAT, Koramangala Bangalore. sandeepgupta01@gmail.com, 9739561394 2. Economist: Tropicorp, which constantly seeks profitable investment opportunities, has been buying and clearing sections of tropical forest for cattle ranching, although pastures newly created there become useless for grazing after just a few years. The company has not gone into rubber tapping, even though greater profits can be made from rubber tapping, which leaves the forest intact. Thus, some environmentalists conclude that Tropicorp has not acted wholly out of economic self-interest. However, these environmentalists are probably wrong. The initial investment required for a successful rubber-tapping operation is larger than that needed for a cattle ranch. Furthermore, there is a shortage of workers employable in rubber-tapping operations, and finally, taxes are higher on profits from rubber tapping than on profits from cattle ranching. In the economist’s argument, the two BOLDFACED portions play which of the following roles? A. The first supports the conclusion of the economist’s argument; the second calls that conclusion into question. B. The first states the conclusion of the economist’s argument; the second supports that conclusion. C. The first supports the environmentalists’ conclusion; the second states that conclusion. D. The first states the environmentalists’ conclusion; the second states the conclusion of the economist’s argument. E. Each supports the conclusion of the economist’s argument.
  • 37. © Sandeep Gupta – Ivy-GMAT, Koramangala Bangalore. sandeepgupta01@gmail.com, 9739561394 3. Local authorities are considering an amendment to the litter law that would raise the fine for littering in the community picnic area to $1,000. Since the inception of the litter law, incremental increases in the littering fine have proven to be consistently effective at further reducing the amount of litter in the community picnic area. However, raising the fine to $1,000 would actually have the unintended effect of increasing the amount of litter in the picnic area. Picnic area users would perceive this fine to be unreasonable and unenforceable, and would disregard the litter law altogether. In the argument, the two portions in BOLDFACE play which of the following roles? A. The first is irrefutable evidence that the author offers in support of a prediction; the second is that prediction. B. The first is a statement of causation that the author predicts will be repeated in the case at hand; the second raises evidence against this prediction. C. The first is a statement of fact that the author accepts to be true; the second is presented as a consequence of this fact. D. The first is evidence that weakens the main position that the author defends; the second is that position. E. The first is a statement of causation that the author predicts will not hold in the case at hand; the second offers a line of reasoning to support this prediction.
  • 38. © Sandeep Gupta – Ivy-GMAT, Koramangala Bangalore. sandeepgupta01@gmail.com, 9739561394 4. Consumer advocate: it is generally true, at least in this state, that lawyers who advertise a specific service charge less for that service than lawyers who do not advertise. It is also true that each time restrictions on the advertising of legal services have been eliminated, the number of lawyers advertising their services has increased and legal costs to consumers have declined in consequence. However, eliminating the state requirement that legal advertisements must specify fees for specific services would almost certainly increase rather than further reduce consumer’s legal costs. Lawyers would no longer have an incentive to lower their fees when they begin advertising and if no longer required to specify fee arrangements, many lawyers who now advertise would increase their fees. In the consumer advocate’s argument, the two portions in boldface play which of the following roles? A. The first is a generalization that the consumer advocate accepts as true; the second is presented as a consequence that follows from the truth of that generalization. B. The first is a pattern of cause and effect that the consumer advocate argues will be repeated in the case at issue; the second acknowledges a circumstance in which that pattern would not hold. C. The first is pattern of cause and effect that the consumer advocate predicts will not hold in the case at issue; the second offers a consideration in support of that prediction. D. The first is evidence that the consumer advocate offers in support of a certain prediction; the second is that prediction. E. The first acknowledges a consideration that weighs against the main position that the consumer advocate defends; the second is that position.
  • 39. © Sandeep Gupta – Ivy-GMAT, Koramangala Bangalore. sandeepgupta01@gmail.com, 9739561394 5. Traditionally, video game manufacturers have been most strongly influenced by serious video gamers. Because devoted gamers have historically purchased the majority of video games, companies react to the desires of this market segment. Normally, devoted gamers crave speed and action; thus, most manufacturers continue to produce games with faster chips and flashier graphics. Unfortunately, faster chips and flashier graphics are no longer in the industry’s best interest. The devoted gaming market is deeply stagnant, and it won’t soon expand. To infuse new life into the video game market, manufacturers must simplify the functionality of their games. By doing so, current non-gamers will be attracted to join the ranks of video game fans. In the argument, the two portions in boldface play which of the following roles? A. The first is a situation that the author believes to be true; the second offers evidence to explain this situation. B. The first is a situation that the author argues should not continue; the second provides evidence that supports the author’s position. C. The first is a statement of fact that contradicts the author’s position; the second is the author’s position. D. The first is a statement of fact that supports the author’s position; the second is a consideration that weighs against the author’s position. E. The first is a prediction that the author believes should not hold in this case; the second is an assumption that weighs against the author’s position.
  • 40. © Sandeep Gupta – Ivy-GMAT, Koramangala Bangalore. sandeepgupta01@gmail.com, 9739561394 CR Type 3: Assumption Questions • Assumptions are unstated facts that must be true if the conclusion is true… • Approach: The Assumption Formula (GAP FILL) Facts + Assumptions = Conclusion. • So the steps are: – ID the conclusion (every other statement becomes FACT) – Try to place answer choices between the fact (s) and the conclusion. The choice that makes a flowing, sound and convincing argument is the assumption.
  • 41. © Sandeep Gupta – Ivy-GMAT, Koramangala Bangalore. sandeepgupta01@gmail.com, 9739561394 Assumption as GAP Fills For example: 1. Amy is less than 5’6” tall. Therefore she can not be a successful model. 1. Every male above 18 is allowed to vote in city X. Therefore, Jack must be allowed to vote in city X. 2. Company owner to manager: “Give this man this job. He will remain jobless otherwise.” 3. Company X doesn’t pay very high salaries to inexperienced people. Therefore John’s salary should be lowered.
  • 42. © Sandeep Gupta – Ivy-GMAT, Koramangala Bangalore. sandeepgupta01@gmail.com, 9739561394 • Assumption Questions will use one of these words: – Assumption / Assumed / Assumes – Presupposition / Presupposed / Presupposes – Justify the conclusion – Inserted as an additional premise – The conclusion can not be true unless which of the following is true – The conclusion will be more properly drawn if…
  • 43. © Sandeep Gupta – Ivy-GMAT, Koramangala Bangalore. sandeepgupta01@gmail.com, 9739561394 SMART TIPS 1. Something that is not mentioned in the facts but mentioned in the conclusion has to be mentioned in the assumption. Also, a connection (BRIDGE/LINK) between X and Y should occur in the assumption. 2. Something that is mentioned both in the facts and in the conclusion will usually not figure in the assumption.
  • 44. © Sandeep Gupta – Ivy-GMAT, Koramangala Bangalore. sandeepgupta01@gmail.com, 9739561394 Assumptions Type 2 : The only (or the best) way • If the conclusion says: the only way (or the best way) to achieve X is Y, the following is a valid assumption: – There is no other (or no better) way to achieve X than Y. 5.Girl Power magazine published an article proclaiming that one can lose up to 20 pounds a month by eating only soup. Kelly concludes that the only way for her to lose 40 pounds in two months is to eat only soup for 2 months.
  • 45. © Sandeep Gupta – Ivy-GMAT, Koramangala Bangalore. sandeepgupta01@gmail.com, 9739561394 Assumptions Type 3: Cause and Effect • If the conclusion of an argument is A causes B, the following are some valid assumptions on the GMAT: – B does not cause A – C does not cause B • For example: 6.Researchers in the field have noticed that older antelope are more cautious. Therefore, they have concluded that the quality of caution increases with age in antelope.
  • 46. © Sandeep Gupta – Ivy-GMAT, Koramangala Bangalore. sandeepgupta01@gmail.com, 9739561394 Type 4 Assumption: Negate and Weaken An argument can be analogized to a house: • The FACTS are like the walls • The CONCLUSION is like the roof • The ASSUMPTIONS are like the foundation – As with a house foundation, an assumption is a hidden part of the structure, but critical to the integrity of the structure—all the other elements rest upon it. • If the conclusion is valid, then the assumption must be true. • So if the assumption is negated, the argument falls apart.
  • 47. © Sandeep Gupta – Ivy-GMAT, Koramangala Bangalore. sandeepgupta01@gmail.com, 9739561394 METHOD: • ID the conclusion • Logically negate the choices • The negated choice MUST UNDERMINE the conclusion Look at this example: 7. His get-rich-quick scheme is simple: he will use a metal detector to find hidden treasures in the sand. Then he will sell the treasures to a local pawn broker.
  • 48. © Sandeep Gupta – Ivy-GMAT, Koramangala Bangalore. sandeepgupta01@gmail.com, 9739561394 1. If something would have been justifiably regretted if it had occurred, then it is something that one should not have desired in the first place. Many forgone pleasures would have been justifiably regretted. It follows that many forgone pleasures should not have been desired in the first place. The conclusion above follows logically if which one of the following is assumed? A. One should never regret one’s pleasures. B. Forgone pleasures that were not desired would not have been justifiably regretted. C. Everything that one desires and then regrets not having is a forgone pleasure. D. Many forgone pleasures would have been justifiably regretted. E. Nothing that one should not have desired in the first place fails to be a pleasure.
  • 49. © Sandeep Gupta – Ivy-GMAT, Koramangala Bangalore. sandeepgupta01@gmail.com, 9739561394 2. Psychiatrist: Take any visceral emotion you care to consider. There are always situations in which it is healthy to try to express that emotion. Anger is a visceral emotion. So, there are always situations in which it is healthy to try to express one’s anger. The conclusion of the argument follows logically if which one of the following is assumed? A. Anger is always expressible. B. Anger is a visceral emotion. C. Some kinds of emotions are unhealthy to express. D. All emotions that are healthy to express are visceral. E. An emotion is visceral only if it is healthy to express.
  • 50. © Sandeep Gupta – Ivy-GMAT, Koramangala Bangalore. sandeepgupta01@gmail.com, 9739561394 3. To prevent some conflicts of interest, Congress could prohibit high- level government officials from accepting positions as lobbyists for three years after such officials leave government service. One such official concluded, however, that such a prohibition would be unfortunate because it would prevent high-level government officials from earning a livelihood for three years. The official’s conclusion logically depends on which of the following assumptions? A. Laws should not restrict the behavior of former government officials. B. Lobbyists are typically people who have previously been high-level government officials. C. Low-level government officials do not often become lobbyists when they leave government service. D. High-level government officials who leave government service are capable of earning a livelihood only as lobbyists. E. High-level government officials who leave government service are currently permitted to act as lobbyists for only three years.
  • 51. © Sandeep Gupta – Ivy-GMAT, Koramangala Bangalore. sandeepgupta01@gmail.com, 9739561394 4. When limitations were in effect on nuclear-arms testing, people tended to save more of their money, but when nuclear-arms testing increased, people tended to spend more of their money. The perceived threat of nuclear catastrophe, therefore, decreases the willingness of people to postpone consumption for the sake of saving money. The argument above assumes that A. the perceived threat of nuclear catastrophe has increased over the years. B. most people supported the development of nuclear arms C. people’s perception of the threat of nuclear catastrophe depends on the amount of nuclear-arms testing being done D. the people who saved the most money when nuclear-arms testing was limited were the ones who supported such limitations E. there are more consumer goods available when nuclear-arms testing increases
  • 52. © Sandeep Gupta – Ivy-GMAT, Koramangala Bangalore. sandeepgupta01@gmail.com, 9739561394 5. If the airspace around centrally located airports were restricted to commercial airliners and only those private planes equipped with radar, most of the private-plane traffic would be forced to use outlying airfields. Such a reduction in the amount of private-plane traffic would reduce the risk of midair collision around the centrally located airports. The conclusion drawn in the first sentence depends on which of the following assumptions? A. Outlying airfields would be as convenient as centrally located airports for most pilots of private planes. B. Most outlying airfields are not equipped to handle commercial-airline traffic. C. Most private planes that use centrally located airports are not equipped with radar. D. Commercial airliners are at greater risk of becoming involved in midair collisions than are private planes. E. A reduction in the risk of midair collision would eventually lead to increases in commercial-airline traffic.
  • 53. © Sandeep Gupta – Ivy-GMAT, Koramangala Bangalore. sandeepgupta01@gmail.com, 9739561394 6. In recent years many cabinetmakers have been winning acclaim as artists. But since furniture must be useful, cabinetmakers must exercise their craft with an eye to the practical utility of their product. For this reason, cabinet-making is not art. Which of the following is an assumption that supports drawing the conclusion above from the reason given for that conclusion? A. Some furniture is made to be placed in museums, where it will not be used by anyone. B. Some cabinetmakers are more concerned than others with the practical utility of the products they produce. C. Cabinetmakers should be more concerned with the practical utility of their products than they currently are. D. An object is not an art object if its maker pays attention to the object’s practical utility. E. Artists are not concerned with the monetary value of their products.
  • 54. © Sandeep Gupta – Ivy-GMAT, Koramangala Bangalore. sandeepgupta01@gmail.com, 9739561394 7. Traditionally, decision-making by managers that is reasoned step-by-step has been considered preferable to intuitive decision-making. However, a recent study found that top managers used intuition significantly more than did most middle-or lower-level managers. This confirms the alternative view that intuition is actually more effective than careful, methodical reasoning. The conclusion above is based on which of the following assumptions? A. Methodical, step-by-step reasoning is inappropriate for making many real-life management decisions. B. Top managers have the ability to use either intuitive reasoning or methodical, step-by-step reasoning in making decisions. C. The decisions made by middle-and lower-level managers can be made as easily by using methodical reasoning as by using intuitive reasoning. D. Top managers use intuitive reasoning in making the majority of their decisions. E. Top managers are more effective at decision-making than middle-or lower- level managers
  • 55. © Sandeep Gupta – Ivy-GMAT, Koramangala Bangalore. sandeepgupta01@gmail.com, 9739561394 8. Emissions from automobiles that burn gasoline and automobiles that burn diesel fuel are threatening the quality of life on our planet, contaminating both urban air and global atmosphere. Therefore, the only effective way to reduce such emissions is to replace the conventional diesel fuel and gasoline used in automobiles with cleaner-burning fuels, such as methanol, that create fewer emissions. Which one of the following is an assumption on which the argument depends? A. Reducing the use of automobiles would not be a more effective means to reduce automobile emissions than the use of methanol. B. There is no fuel other than methanol that is cleaner-burning than both diesel fuel and gasoline. C. If given a choice of automobile fuels, automobile owners would not select gasoline over methanol. D. Automobile emissions constitute the most serious threat to the global environment. E. At any given time there is a direct correlation between the level of urban air pollution and the level of contamination present in the global atmosphere.
  • 56. © Sandeep Gupta – Ivy-GMAT, Koramangala Bangalore. sandeepgupta01@gmail.com, 9739561394 9. Every year many people become ill because of airborne mold spores in their homes. After someone becomes ill, specialists are often hired to eradicate the mold. These specialists look in damp areas of the house, since mold is almost always found in places where there is substantial moisture. If one wishes to avoid mold poisoning, then, one should make sure to keep all internal plumbing in good condition to prevent leakage that could serve as a breeding ground for mold. Which of the following is an assumption on which the argument depends? A. Mold itself does not create moisture. B. Most homeowners know enough about plumbing to determine whether theirs is in good condition. C. Mold cannot grow in dry areas. D. No varieties of mold are harmless. E. Mold spores cannot be filtered from the air.
  • 57. © Sandeep Gupta – Ivy-GMAT, Koramangala Bangalore. sandeepgupta01@gmail.com, 9739561394 10. Smoking is a known cause of certain serious health problems, including emphysema and lung cancer. Now, an additional concern can be added to the list of maladies caused by smoking. A recent study surveyed both smokers and nonsmokers, and found that smokers are significantly more anxious and nervous than nonsmokers. Which of the following is an assumption on which the argument rests? A. Anxiety and nervousness can lead to serious health problems. B. Anxiety and nervousness do not make individuals more likely to start smoking. C. Equivalent numbers of smokers and nonsmokers were surveyed for the study. D. Smokers are aware of the various health problems attributed to smoking, including lung cancer and emphysema. E. Smokers who had smoked a cigarette immediately before responding to the survey were more anxious and nervous than smokers who had not smoked for several hours.
  • 58. © Sandeep Gupta – Ivy-GMAT, Koramangala Bangalore. sandeepgupta01@gmail.com, 9739561394 11. A researcher discovered that people who have low levels of immune- system activity tend to score much lower on tests of mental health than do people with normal or high immune-system activity. The researcher concluded from this experiment that the immune system protects against mental illness as well as against physical disease. The researcher’s conclusion depends on which of the following assumptions? A. High immune-system activity protects against mental illness better than normal immune-system activity does. B. Mental illness is similar to physical disease in its effects on body systems. C. People with high immune-system activity cannot develop mental illness. D. Mental illness does not cause people’s immune-system activity to decrease. E. Psychological treatment of mental illness is not as effective as is medical treatment.
  • 59. © Sandeep Gupta – Ivy-GMAT, Koramangala Bangalore. sandeepgupta01@gmail.com, 9739561394 12. A famous singer recently won a lawsuit against an advertising firm for using another singer in a commercial to evoke the famous singer’s well-known rendition of a certain song. As a result of the lawsuit, advertising firms will stop using imitators in commercials. Therefore, advertising costs will rise, since famous singers’ services cost more than those of their imitators. The conclusion above is based on which of the following assumptions? A. Most people are unable to distinguish a famous singer’s rendition of a song from a good imitator’s rendition of the same song. B. Commercials using famous singers are usually more effective than commercials using imitators of famous singers. C. The original versions of some well-known songs are unavailable for use in commercials. D. Advertising firms will continue to use imitators to mimic the physical mannerisms of famous singers. E. The advertising industry will use well-known renditions of songs in commercials.
  • 60. © Sandeep Gupta – Ivy-GMAT, Koramangala Bangalore. sandeepgupta01@gmail.com, 9739561394 13. Although there has been great scientific debate for decades over global warming, most scientists now agree that human activity is causing the Earth’s temperature to rise. Though predictions vary, many global warming experts believe that average global temperatures will rise between three and eight degrees Fahrenheit during the next century. Such an increase would cause an alarming rise in sea levels, displacing millions of people by destroying major population centers along the world’s coastlines. Which of the following is an assumption in support of the argument’s conclusion? A. New technological developments in the next century will not divert rising seas from the world’s coastal cities. B. Individuals will not become more aware of the steps they can take to reduce the emission of greenhouse gases. C. Rising sea levels similarly affect all coastal population centers. D. Some global warming experts predict a greater than eight degree Fahrenheit increase in global temperatures during the next century. E. Human activity is the sole cause of increasing global temperatures.
  • 61. © Sandeep Gupta – Ivy-GMAT, Koramangala Bangalore. sandeepgupta01@gmail.com, 9739561394 14. The media claim that the economy is entering a phase of growth and prosperity. They point to lower unemployment rates and increased productivity. This analysis is false, though. The number of people filing for bankruptcy has increased every month for the last six months, and bankruptcy lawyers report that they are busier than they have been in years. Which of the following is an assumption on which the argument depends? A. Unemployment rates are not useful indicators of growth and prosperity. B. Economic growth cannot be measured in terms of productivity. C. Legislation has not been recently passed to make legal bankruptcy easier to obtain. D. There has not been an increase in the number of bankruptcy lawyers. E. The media often misrepresent the current state of economic affairs.
  • 62. © Sandeep Gupta – Ivy-GMAT, Koramangala Bangalore. sandeepgupta01@gmail.com, 9739561394 15. Doctor: Research shows that adolescents who play video games on a regular basis are three times as likely to develop carpal tunnel syndrome as are adolescents who do not play video games. Federal legislation that prohibits the sale of video games to minors would help curb this painful wrist condition among adolescents. The doctor’s conclusion depends on which of the following assumptions? A. The majority of federal legislators would vote for a bill that prohibits the sale of video games to minors. B. Not all adolescents who play video games on a regular basis suffer from carpal tunnel syndrome. C. Playing video games is the only way an adolescent can develop carpal tunnel syndrome. D. Most parents would refuse to purchase video games for their adolescent children. E. The regular playing of video games by adolescents does not produce such beneficial effects as better hand-eye coordination and improved reaction time.
  • 63. © Sandeep Gupta – Ivy-GMAT, Koramangala Bangalore. sandeepgupta01@gmail.com, 9739561394 CR Type 4: Weaken • All weaken questions will have either a FACT- CONCLUSION or a FACT-REASON (A causes B) relationship. • We can never negate a fact. We can negate only the conclusion or the reason. So the model will be: F  C  OR F  R 
  • 64. © Sandeep Gupta – Ivy-GMAT, Koramangala Bangalore. sandeepgupta01@gmail.com, 9739561394 Weaken Questions • Options have to contain OUTSIDE (EXTRA) INFORMATION (FACT) • Weaken does not mean DESTROY. • So even mild choices may be true.
  • 65. © Sandeep Gupta – Ivy-GMAT, Koramangala Bangalore. sandeepgupta01@gmail.com, 9739561394 Weaken 1.Example: As the Indian cricket team performed badly in the last World Cup, it should not be sent for the next World Cup. – FACT: the Indian cricket team performed badly in the last World Cup – Conclusion: it should not be sent for the next World Cup. • How to weaken it?
  • 66. © Sandeep Gupta – Ivy-GMAT, Koramangala Bangalore. sandeepgupta01@gmail.com, 9739561394 Weaken 2. Over the last 2 months, a police station has started receiving many more calls about crime in the adjoining areas. This is a clear indication of the fact that crime is on the rise in the adjoining areas. – FACT: Over the last 2 month, a police station has started receiving many more calls about crime in the adjoining areas. – Conclusion: crime is on the rise in the adjoining areas. • How to weaken it? • This is A causes B… i.e., more calls about crime  more crime… • We can not question facts but only the reason…
  • 67. © Sandeep Gupta – Ivy-GMAT, Koramangala Bangalore. sandeepgupta01@gmail.com, 9739561394 Process • ID the conclusion • Try to come up with your own answer • New-information choices are welcome • Undermine the conclusion with the help of a choice • Look out for “ALTERNATIVE EXPLANATIONS” in FACT-REASON relationships.
  • 68. © Sandeep Gupta – Ivy-GMAT, Koramangala Bangalore. sandeepgupta01@gmail.com, 9739561394 How to weaken A causes B • Show that C (something else) causes B • Show that when A happened, B did not happen. • Show that when B happened, A did not happen. • Show that B causes A (reverse relation)
  • 69. © Sandeep Gupta – Ivy-GMAT, Koramangala Bangalore. sandeepgupta01@gmail.com, 9739561394 A causes B can be written as… • caused by • because of • responsible for • reason for • leads to • induced by • promoted by • determined by • produced by • product of • played a role in • was a factor in • is an effect of
  • 70. © Sandeep Gupta – Ivy-GMAT, Koramangala Bangalore. sandeepgupta01@gmail.com, 9739561394 Common Words Used… • weaken • attack • undermine • refute • argue against • call into question • cast doubt • challenge • damage • counter • absurdity • vulnerable • flaw • counter • questionable • fallacious
  • 71. © Sandeep Gupta – Ivy-GMAT, Koramangala Bangalore. sandeepgupta01@gmail.com, 9739561394 1. The number of airplanes equipped with a new anti-collision device has increased steadily during the past two years. During the same period, it has become increasingly common for key information about an airplane’s altitude and speed to disappear suddenly from air traffic controllers’ screens. The new anti-collision device, which operates at the same frequency as air traffic radar, is therefore responsible for the sudden disappearance of key information. Which one of the following, if true, most seriously weakens the argument? A. The new anti-collision device has already prevented a considerable number of mid-air collisions. B. It was not until the new anti-collision device was introduced that key information first began disappearing suddenly from controllers’ screens. C. The new anti-collision device is scheduled to be moved to a different frequency within the next two to three months. D. Key information began disappearing from controllers’ screens three months before the new anti-collision device was first installed. E. The sudden disappearance of key information from controllers’ screens has occurred only at relatively large airports.
  • 72. © Sandeep Gupta – Ivy-GMAT, Koramangala Bangalore. sandeepgupta01@gmail.com, 9739561394 2. The average life expectancy for the United States population as a whole is 73.9 years, but children born in Hawaii will live an average of 77 years, and those born in Louisiana, 71.7 years. If a newlywed couple from Louisiana were to begin their family in Hawaii, therefore, their children would be expected to live longer than would be the case if the family remained in Louisiana. Which of the following, if true, would most seriously weaken the conclusion drawn in the passage? A. Insurance company statisticians do not believe that moving to Hawaii will significantly lengthen the average Louisianan’s life. B. The governor of Louisiana has falsely alleged that statistics for his state are inaccurate. C. The longevity ascribed to Hawaii’s current population is attributable mostly to genetically determined factors. D. Thirty percent of all Louisianans can expect to live longer than 77 years. E. Most of the Hawaiian Islands have levels of air pollution well below the national average for the United States.
  • 73. © Sandeep Gupta – Ivy-GMAT, Koramangala Bangalore. sandeepgupta01@gmail.com, 9739561394 3. A program instituted in a particular state allows parents to prepay their children’s future college tuition at current rates. The program then pays the tuition annually for the child at any of the state’s public colleges in which the child enrolls. Parents should participate in the program as a means of decreasing the cost for their children’s college education. Which of the following, if true, is the most appropriate reason for parents NOT to participate in the program? A. The parents are unsure about which public college in the state the child will attend. B. The amount of money accumulated by putting the prepayment funds in an interest-bearing account today will be greater than the total cost of tuition for any of the public colleges when the child enrolls. C. The annual cost of tuition at the state’s public colleges is expected to increase at a faster rate than the annual increase in the cost of living. D. Some of the state’s public colleges are contemplating large increases in tuition next year. E. The prepayment plan would not cover the cost of room and board at any of the state’s public colleges.
  • 74. © Sandeep Gupta – Ivy-GMAT, Koramangala Bangalore. sandeepgupta01@gmail.com, 9739561394 4. The ice on the front windshield of the car had formed when moisture condensed during the night. The ice melted quickly after the car was warmed up the next morning because the defrosting vent, which blows on the front windshield, was turned on full force. Which of the following, if true, most seriously jeopardizes the validity of the explanation for the speed with which the ice melted? A. The side windows had no ice condensation on them B. Even though no attempt was made to defrost the back window, the ice there melted at the same rate as did the ice on the front windshield. C. The speed at which ice on a window melts increases as the temperature of the air blown on the window increases D. The warm air from the defrosting vent for the front windshield cools rapidly as it dissipates throughout the rest of the car. E. The defrosting vent operates efficiently even when the heater, which blows warm air toward the feet or faces of the driver and passengers, is on.
  • 75. © Sandeep Gupta – Ivy-GMAT, Koramangala Bangalore. sandeepgupta01@gmail.com, 9739561394 5. A conservation group in the United States is trying to change the long-standing image of bats as frightening creatures. The group contends that bats are feared and persecuted solely because they are shy animals that are active only at night. Which of the following, if true, would cast the most serious doubt on the accuracy of the group’s contention? A. Bats are steadily losing natural roosting places such as caves and hollow trees and are thus turning to more developed areas for roosting. B. Bats are the chief consumers of nocturnal insects and thus can help make their hunting territory more pleasant for humans. C. Bats are regarded as frightening creatures not only in the United States but also in Europe, Africa, and South America. D. Raccoons and owls are shy and active only at night; yet they are not generally feared and persecuted. E. People know more about the behavior of other greatly feared animal species, such as lions, alligators, and greatly feared animal species, such as lions, alligators, and snakes, than they do about the behavior of bats.
  • 76. © Sandeep Gupta – Ivy-GMAT, Koramangala Bangalore. sandeepgupta01@gmail.com, 9739561394 6. Beverage company representative: The plastic rings that hold six-packs of beverage cans together pose a threat to wild animals, which often become entangled in the discarded rings and suffocate as a result. Following our lead, all beverage companies will soon use only those rings consisting of a new plastic that disintegrates after only three days’ exposure to sunlight. Once we all complete the switchover from the old to the new plastic rings, therefore, the threat of suffocation that plastic rings pose to wild animals will be eliminated. Which one of the following, if true, most seriously weakens the representative’s argument? A. The switchover to the new plastic rings will take at least two more years to complete. B. After the beverage companies have switched over to the new plastic rings, a substantial number of the old plastic rings will persist in most aquatic and woodland environments. C. The new plastic rings are slightly less expensive than the old rings. D. The new plastic rings rarely disintegrate during shipping of beverage six-packs because most trucks that transport canned beverages protect their cargo from sunlight. E. The new plastic rings disintegrate into substances that are harmful to aquatic animals when ingested in substantial quantities by them.
  • 77. © Sandeep Gupta – Ivy-GMAT, Koramangala Bangalore. sandeepgupta01@gmail.com, 9739561394 7. Opponents of laws that require automobile drivers and passengers to wear seat belts argue that in a free society people have the right to take risks as long as the people do not harm other as a result of taking the risks. As a result, they conclude that it should be each person’s decision whether or not to wear a seat belt. Which of the following, if true, most seriously weakens the conclusion drawn above? A. Many new cars are built with seat belts that automatically fasten when someone sits in the front seat. B. Automobile insurance rates for all automobile owners are higher because of the need to pay for the increased injuries or deaths of people not wearing seat belts. C. Passengers in airplanes are required to wear seat belts during takeoffs and landings. D. The rate of automobile fatalities in states that do not have mandatory seat belt laws is greater than the rate of fatalities in states that do have such laws. E. In automobile accidents, a greater number of passengers who do not wear seat belts are injured than are passengers who do wear seat belts.
  • 78. © Sandeep Gupta – Ivy-GMAT, Koramangala Bangalore. sandeepgupta01@gmail.com, 9739561394 8. During the Second World War, about 375,000 civilians died in the United States and about 408,000 members of the United States armed forces died overseas. On the basis the those figures, it can be concluded that it was not much more dangerous to be overseas in the armed forces during the Second World War than it was to stay at home as a civilian. Which of the following would reveal most clearly the absurdity of the conclusion drawn above? A. Counting deaths among members of the armed forces who served in the United State in addition to deaths among members of the armed forces serving overseas B. Expressing the difference between the numbers of deaths among civilians and members of the armed forces as a percentage of the total number of deaths C. Separating deaths caused by accidents during service in the armed forces from deaths caused by combat injuries D. Comparing death rates per thousand members of each group rather than comparing total numbers of deaths E. Comparing deaths caused by accidents in the United States to deaths caused by combat in the armed forces
  • 79. © Sandeep Gupta – Ivy-GMAT, Koramangala Bangalore. sandeepgupta01@gmail.com, 9739561394 9. Recently in City X, developers have stopped buying land, contractors have found themselves going without work for longer periods, and banks have issued fewer mortgages. There must be fewer new residents moving to City X than there were previously. Which of the following indicates a flaw in the reasoning above? A. This year several housing blocks have gone on the market after being held up for months by legal red tape. B. The average size of a new home has increased significantly over the past several years. C. Re-sales of condominiums have increased over the past six months. D. The cost of materials such as lumber and cement has decreased over the past year. E. Sales of other big-ticket items, such as automobiles and boats, has remained steady over the past year.
  • 80. © Sandeep Gupta – Ivy-GMAT, Koramangala Bangalore. sandeepgupta01@gmail.com, 9739561394 10. Companies that offer “employer sponsored insurance” (ESI) pay a portion of employees’ health care costs. In the manufacturing sector last year, companies that offered ESI had worker absentee rates 22% lower, on average, than those at companies that did not offer ESI. If, on the basis of the evidence above, it is argued that ESI decreases worker absenteeism, which of the following, if true, would most seriously weaken that argument? A. Results similar to those cited for the manufacturing sector have been found in other sectors of the economy where ESI is offered. B. At companies that offer ESI, employees have access to preventative health care such as regular check-ups, routine laboratory tests, and nutrition counseling. C. Because initiating an ESI plan requires a lot of paperwork for the company, employees, and the insurance provider, doing so is complex and time- consuming. D. Many firms in the manufacturing sector have improved workplace safety and decreased the occurrence of on-the-job injuries in the last five years, and most of these companies introduced ESI at the same time. E. In manufacturing firms where ESI is offered, the average productivity is 2% higher than it is in those firms where workers are not covered by an ESI plan.
  • 81. © Sandeep Gupta – Ivy-GMAT, Koramangala Bangalore. sandeepgupta01@gmail.com, 9739561394 Numbers and Percentages Misconceptions 1. Increasing percentages automatically lead to increasing numbers. 2. Decreasing percentages automatically lead to decreasing numbers. 3. Increasing numbers automatically lead to increasing percentages. 4. Decreasing numbers automatically lead to decreasing percentages. 5. Large numbers automatically mean large percentages, and small numbers automatically mean small percentages. 6. Large percentages automatically mean large numbers, and small percentages automatically mean small numbers.
  • 82. © Sandeep Gupta – Ivy-GMAT, Koramangala Bangalore. sandeepgupta01@gmail.com, 9739561394 Common terms Words used to introduce NUMBERS: • Amount • Quantity • Sum • Total • Count • Tally Words used to introduce percentage ideas: • Percent • Proportion • Fraction • Ratio • Incidence • Likelihood • Probability • Segment • Share
  • 83. © Sandeep Gupta – Ivy-GMAT, Koramangala Bangalore. sandeepgupta01@gmail.com, 9739561394 Market Share Market share is a comparative term, as opposed to an absolute term. Market share is simply the portion of a market that a company controls. The market share can be measured either in terms of revenues (sales) or units sold. A company can gain market share (percentage) if the market shrinks and it maintains a constant size, or if it grows in an unchanging market. However, a company losing market share does not mean that its sales decreased, only that they became a smaller entity in the market relative to the whole (the market grew and they stayed the same size, for example). Similarly, a company could lose sales and still gain market share if the overall market became smaller. The total market share must always add up to 100%.
  • 84. © Sandeep Gupta – Ivy-GMAT, Koramangala Bangalore. sandeepgupta01@gmail.com, 9739561394 CR Type 5: Strengthen • All strengthen questions will have either a FACT- CONCLUSION or a FACT-REASON (A causes B) relationship. • We can never strengthen a fact. We can strengthen only the conclusion or the reason by providing some extra facts. So the model will be: F  C  OR F  R 
  • 85. © Sandeep Gupta – Ivy-GMAT, Koramangala Bangalore. sandeepgupta01@gmail.com, 9739561394 Strengthen Questions • Options have to contain OUTSIDE (EXTRA) INFORMATION (FACT) • Strengthen does not mean PROVE. • So even mild choices may be true.
  • 86. © Sandeep Gupta – Ivy-GMAT, Koramangala Bangalore. sandeepgupta01@gmail.com, 9739561394 Strengthen • Example: As the Indian cricket team performed badly in the last World Cup, it should not be sent for the next World Cup. – FACT: the Indian cricket team performed badly in the last World Cup – Conclusion: it should not be sent for the next World Cup. • How to strengthen it?
  • 87. © Sandeep Gupta – Ivy-GMAT, Koramangala Bangalore. sandeepgupta01@gmail.com, 9739561394 Terms used… • strengthens • supports • helps • most justifies • bolsters • buttresses
  • 88. © Sandeep Gupta – Ivy-GMAT, Koramangala Bangalore. sandeepgupta01@gmail.com, 9739561394 How to strengthen A causes B • Show that nothing else causes B • Show that when A did not happen, B did not happen. • Show that when B did not happen, A did not happen. • Show that B does not cause A
  • 89. © Sandeep Gupta – Ivy-GMAT, Koramangala Bangalore. sandeepgupta01@gmail.com, 9739561394 Strengthen – cause effect… Navigation systems found in most aircraft are made with low-power circuitry, which is susceptible to interference. Recently, one plane veered off course during landing, when a passenger turned on a laptop computer. Clearly, aircraft navigation systems are being put at risk by the electronic devices that passengers carry on board, such as cassette players and laptop computers. Find at least 2 ways to strengthen it…
  • 90. © Sandeep Gupta – Ivy-GMAT, Koramangala Bangalore. sandeepgupta01@gmail.com, 9739561394 Process • ID the conclusion • Try to come up with your own answer • New-information choices are welcome • Strengthen the conclusion with the help of a choice
  • 91. © Sandeep Gupta – Ivy-GMAT, Koramangala Bangalore. sandeepgupta01@gmail.com, 9739561394 1. Medical doctor: Sleep deprivation is the cause of many social ills, ranging from irritability to potentially dangerous instances of impaired decision making. Most people today suffer from sleep deprivation to some degree. Therefore we should restructure the workday to allow people flexibility in scheduling their work hours. Which one of the following, if true, would most strengthen the medical doctor’s argument? A. The primary cause of sleep deprivation is overwork. B. Employees would get more sleep if they had greater latitude in scheduling their work hours. C. Individuals vary widely in the amount of sleep they require. D. More people would suffer from sleep deprivation today than did in the past if the average number of hours worked per week had not decreased. E. The extent of one’s sleep deprivation is proportional to the length of one’s workday.
  • 92. © Sandeep Gupta – Ivy-GMAT, Koramangala Bangalore. sandeepgupta01@gmail.com, 9739561394 2. Galanin is a protein found in the brain. In an experiment, rats that consistently chose to eat fatty foods when offered a choice between lean and fatty foods were found to have significantly higher concentrations of galanin in their brains than did rats that consistently chose lean over fatty foods. These facts strongly support the conclusion that galanin causes rats to crave fatty foods. Which one of the following, if true, most supports the argument? A. The craving for fatty foods does not invariably result in a rat’s choosing those foods over lean foods. B. The brains of the rats that consistently chose to eat fatty foods did not contain significantly more fat than did the brains of rats that consistently chose lean foods. C. The chemical components of galanin are present in both fatty foods and lean foods. D. The rats that preferred fatty foods had the higher concentrations of galanin in their brains before they were offered fatty foods. E. Rats that metabolize fat less efficiently than do other rats develop high concentrations of galanin in their brains.
  • 93. © Sandeep Gupta – Ivy-GMAT, Koramangala Bangalore. sandeepgupta01@gmail.com, 9739561394 3. In the 18th and 19th centuries, it was believed in many coastal American cities that the waterfront was an undesirable location for residential buildings. As a result, much of the waterfront in these cities was never developed aesthetically and instead was left to industry and commerce. Today, however, waterfront properties are generally seen as prestigious, as evidenced by the large sums paid for homes along the beach front. A developer who wishes to make a large profit would be wise to buy urban waterfront lots and erect residential buildings on them. Which of the following, if true, most strongly supports the claim made about urban waterfront properties? A. People today have more money, relatively speaking, to spend on real estate than they did in previous centuries. B. Homeowners will be willing to spend large sums on residential properties in traditionally industrial or commercial districts. C. Many urban waterfront lots are available for purchase. D. Many coastal American cities are encouraging developers to rehabilitate the waterfront through tax incentives. E. Properties in interior residential districts in coastal American cities are significantly more expensive than those along the waterfront.
  • 94. © Sandeep Gupta – Ivy-GMAT, Koramangala Bangalore. sandeepgupta01@gmail.com, 9739561394 4. Some animals, such as dolphins, dogs, and African grey parrots, seem to exhibit cognitive functions typically associated with higher- order primates such as chimpanzees, gorillas, and humans. Some parrots, for example, have vocabularies of hundreds of words that they can string together in a comprehensible syntax. This clearly shows that humans and primates are not the only animals capable of using language to communicate. One parrot, named Alex, has been known to ask to be petted or kissed and will exhibit aggression if the gesture offered is not the specific one requested. Which of the following, if true, would most strengthen the conclusion above? A. Dolphins can be trained to assist divers in ocean rescues. B. Gorillas in captivity often learn hand signals for food and water. C. Dogs are capable of sensing their owners' moods and often exhibit concern if they sense sadness. D. Chimpanzees can memorize long sequences of key punches on machines that dispense food. E. Alex does not exhibit aggression when offered a gesture that he specifically requested.
  • 95. © Sandeep Gupta – Ivy-GMAT, Koramangala Bangalore. sandeepgupta01@gmail.com, 9739561394 5. In general, jobs are harder to get in times of economic recession because many businesses cut back operations. However, any future recessions in Vargonia will probably not reduce the availability of teaching jobs at government-funded schools. This is because Vargonia has just introduced a legal requirement that education in government-funded schools be available, free of charge, to all Vargonian children regardless of the state of the economy, and that current student-teacher ratios not be exceeded. Which of the following, if true, most strengthens the argument? A. The current student-teacher ratio at Vargonia’s government-funded schools is higher than it was during the most recent period of economic recession. B. During recent periods when the Vargonian economy has been strong, almost 25 percent of Vargonian children have attended privately funded schools, many of which charge substantial fees. C. Nearly 20 percent more teachers are currently employed in Vargonia’s government-funded schools than had been employed in those schools in the period before the last economic recession. D. Teachers in Vargonia’s government-funded schools are well paid relative to teachers in most privately funded schools in Vargonia, many of which rely heavily on part-time teachers. E. During the last economic recession in Vargonia, the government permanently closed a number of the schools that it had funded.
  • 96. © Sandeep Gupta – Ivy-GMAT, Koramangala Bangalore. sandeepgupta01@gmail.com, 9739561394 6. The average life expectancy for the United States population as a whole is 73.9 years, but children born in Hawaii will live an average of 77 years, and those born in Louisiana, 71.7 years. If a newlywed couple from Louisiana were to begin their family in Hawaii, therefore, their children would be expected to live longer than would be the case if the family remained in Louisiana. Which of the following statements, if true, would most significantly strengthen the conclusion drawn in the passage? A. As population density increases in Hawaii, life expectancy figures for that state are likely to be revised downward. B. Environmental factors tending to favor longevity are abundant in Hawaii and less numerous in Louisiana. C. Twenty-five percent of all Louisianans who move to Hawaii live longer than 77 years. D. Over the last decade, average life expectancy has risen at a higher rate for Louisianans than for Hawaiians. E. Studies show that the average life expectancy for Hawaiians who move permanently to Louisiana is roughly equal to that of Hawaiians who remain in Hawaii.
  • 97. © Sandeep Gupta – Ivy-GMAT, Koramangala Bangalore. sandeepgupta01@gmail.com, 9739561394 7. A cost-effective solution to the problem of airport congestion is to provide high-speed ground transportation between major cities lying 200 to 500 miles apart. The successful implementation of this plan would cost far less than expanding existing airports and would also reduce the number of airplanes clogging both airports and airways. Which of the following, if true, could be proponents of the plan above most appropriately cite as a piece of evidence for the soundness of their plan? A. An effective high-speed ground-transportation system would require major repairs to many highways and mass-transit improvements. B. One-half of all departing flights in the nation’s busiest airport head for a destination in a major city 225 miles away. C. The majority of travelers departing from rural airports are flying to destinations in cities over 600 miles away. D. Many new airports are being built in areas that are presently served by high-speed ground-transportation systems. E. A large proportion of air travelers are vacationers who are taking long- distance flights.
  • 98. © Sandeep Gupta – Ivy-GMAT, Koramangala Bangalore. sandeepgupta01@gmail.com, 9739561394 8. A recent spate of launching and operating mishaps with television satellites led to a corresponding surge in claims against companies underwriting satellite insurance. As a result, insurance premiums shot up, making satellites more expensive to launch and operate. This, in turn, has added to the pressure to squeeze more performance out of currently operating satellites. Which of the following, if true, taken together with the information above, best supports the conclusion that the cost of television satellites will continue to increase? A. Since the risk to insurers of satellites is spread over relatively few units, insurance premiums are necessarily very high. B. When satellites reach orbit and then fail, the causes of failure are generally impossible to pinpoint with confidence. C. The greater the performance demands placed on satellites, the more frequently those satellites break down. D. Most satellites are produced in such small numbers that no economies of scale can be realized. E. Since many satellites are built by unwieldy international consortia, inefficiencies are inevitable.
  • 99. © Sandeep Gupta – Ivy-GMAT, Koramangala Bangalore. sandeepgupta01@gmail.com, 9739561394 CR Type 6: Evaluate the Argument • “EVALUATE” means to check the soundness of conclusion… • All EVALUATE questions will have a FACT- CONCLUSION relationship. • The given argument will be unsound, i.e., there will be inadequate facts to support the conclusion. • In order to prove or disprove the argument, one will need more information. This is what the question asks… – What extra piece of information will help one evaluate the conclusion better?
  • 100. © Sandeep Gupta – Ivy-GMAT, Koramangala Bangalore. sandeepgupta01@gmail.com, 9739561394 Approach… • ID the conclusion. • Take the choices to the extremes. • The Extremes Test: One extreme should weaken and the other should strengthen the conclusion or vice versa.
  • 101. © Sandeep Gupta – Ivy-GMAT, Koramangala Bangalore. sandeepgupta01@gmail.com, 9739561394 An Example… 1. Advertisement: Only Gem World sends its jewelry to the most reliable Gem Testing Lab in the country. No other shop sends its jewelry for any kind of testing whatsoever. So buy jewelry from Gem World, the most reliable jewelry shop in town, and be assured of the quality of gems you purchase! The answer to which question will help one evaluate the authenticity of the claim made in the advertisement above?
  • 102. © Sandeep Gupta – Ivy-GMAT, Koramangala Bangalore. sandeepgupta01@gmail.com, 9739561394 Another Example… 2. Because of a rare type of fungus that killed off many cacao trees in Brazil, there was an unusually meager harvest of cocoa beans this year. The wholesale price of cocoa solids and cocoa butter has increased significantly and does not look likely to fall in the foreseeable future. As a result, the retail price of chocolate is certain to increase within six months. The answer to which question would provide information relevant to evaluating the claims made in the argument above?
  • 103. © Sandeep Gupta – Ivy-GMAT, Koramangala Bangalore. sandeepgupta01@gmail.com, 9739561394 Another Example… 3. In an experiment, two different types of recorded music were played for neonates in adjacent nurseries in a hospital. In nursery A, classical music was played; in nursery B, rock music was played. After two weeks, it was found that the babies in nursery A cried less, suffered fewer minor ailments, and gained more weight than did the babies in nursery B. In evaluating the validity of the conclusion suggested by the experiment above, it would be most important to know which of the following?
  • 104. © Sandeep Gupta – Ivy-GMAT, Koramangala Bangalore. sandeepgupta01@gmail.com, 9739561394 1. Profits for one of Company X's flagship products have been declining slowly for several years. The CFO investigated and determined that inflation has raised the cost of producing the product but consumers who were surveyed reported that they weren't willing to pay more than the current price. As a result, the CFO recommended that the company stop producing this product because the CEO only wants products whose profit margins are increasing. The answer to which of the following questions would be most useful in evaluating whether the CFO's decision to divest the company of its flagship product is warranted? A. Does the company have new and profitable products available with which to replace the flagship product? B. Will the rest of Company X's management team agree with the CFO's recommendation? C. Are there additional features which could be added to the product and for which consumers might be willing to pay a higher price? D. Is there a way to alter the manufacturing or distribution processes in order to reduce the cost to produce the flagship product? E. What percentage of Company X's revenues is represented by sales of the flagship product in question?
  • 105. © Sandeep Gupta – Ivy-GMAT, Koramangala Bangalore. sandeepgupta01@gmail.com, 9739561394 2. Scientists have determined that an effective way to lower cholesterol is to eat three servings of whole grains every day. Studies have shown that the cholesterol levels of people who did so were significantly lower after six months than were those of people who did not, even though the cholesterol levels of the two groups were the same before the studies began. Clearly, eating whole grains can have an appreciable effect on cholesterol levels. The answer to which of the following questions, if true, would be most useful in evaluating the claim about whole grains above? A. Is it realistic to expect people to eat three servings of whole grains per day? B. Were the two groups of people in the study involved in the same exercise program? C. Can the same drop in cholesterol be achieved through medication? D. Did the study continue to track the subjects beyond six months? E. Are most consumers aware of the different between whole grains and processed grains?
  • 106. © Sandeep Gupta – Ivy-GMAT, Koramangala Bangalore. sandeepgupta01@gmail.com, 9739561394 3. Guitar strings often go “dead”—become less responsive and bright in tone—after a few weeks of intense use. A researcher whose son is a classical guitarist hypothesized that dirt and oil, rather than changes in the material properties of the string, were responsible. Which of the following investigations is most likely to yield significant information that would help to evaluate the researcher’s hypothesis? A. Determining if a metal alloy is used to make the strings used by classical guitarists B. Determining whether classical guitarists make their strings go dead faster than do folk guitarists C. Determining whether identical lengths of string, of the same gauge, go dead at different rates when strung on various brands of guitars. D. Determining whether a dead string and a new string produce different qualities of sound E. Determining whether smearing various substances on new guitar strings causes them to go dead
  • 107. © Sandeep Gupta – Ivy-GMAT, Koramangala Bangalore. sandeepgupta01@gmail.com, 9739561394 4. The recording industry is fighting a losing battle: it simply does not have the resources to prosecute all of the individuals who illegally download music from the Internet. Because the number of individuals who will be charged with a crime is so limited, the actions of the recording industry will have a minimal impact on the number of people who illegally download music. The answer to which of the following questions would best help evaluate the accuracy of the conclusion above? A. Will recording industry lawyers dedicate the majority of their time to prosecuting those who illegally download music? B. Is a small minority of individuals responsible for the majority of illegal song downloads? C. Do many individuals who illegally download songs share their music files with other Internet users? D. Will new Internet security technology permit the recording industry to more quickly and easily identify individuals who illegally download music? E. Will the threat of prosecution alter the behavior of those who illegally download music?
  • 108. © Sandeep Gupta – Ivy-GMAT, Koramangala Bangalore. sandeepgupta01@gmail.com, 9739561394 5. Columnist: George Orwell’s book 1984 has exercised much influence on a great number of this newspaper’s readers. One thousand readers were surveyed and asked to name the one book that had the most influence on their lives. The book chosen most often was the Bible; 1984 was second. The answer to which one of the following questions would most help in evaluating the columnist’s argument? A. How many books had each person surveyed read? B. How many people chose books other than 1984? C. How many people read the columnist’s newspaper? D. How many books by George Orwell other than 1984 were chosen? E. How many of those surveyed had actually read the books they chose?
  • 109. © Sandeep Gupta – Ivy-GMAT, Koramangala Bangalore. sandeepgupta01@gmail.com, 9739561394 6. The proportion of women among students enrolled in higher education programs has increased over the past decades. This is partly shown by the fact that in 1959, only 11 percent of the women between twenty and twenty-one were enrolled in college, while in 1981, 30 percent of the women between twenty and twenty-one were enrolled in college. To evaluate the argument above, it would be most useful to compare 1959 and 1981 with regard to which of the following characteristics? A. The percentage of women between twenty and twenty-one who were not enrolled in college B. The percentage of women between twenty and twenty-five who graduated from college C. The percentage of women who, after attending college, entered highly paid professions D. The percentage of men between twenty and twenty-one who were enrolled in college E. The percentage of men who graduated from high school
  • 110. © Sandeep Gupta – Ivy-GMAT, Koramangala Bangalore. sandeepgupta01@gmail.com, 9739561394 CR Type 7: Resolve the Paradox • Structure: – Two seemingly contradictory facts are presented in the passage, creating a paradox (a surprising phenomenon). F1  F2 – One has to resolve (explain) the Paradox by choosing the most credible choice that explains both sides. – MODEL: F1F1 F2F2
  • 111. © Sandeep Gupta – Ivy-GMAT, Koramangala Bangalore. sandeepgupta01@gmail.com, 9739561394 Paradox Questions • No conclusion • Language of contradiction (But, However, Yet, Although, Paradoxically, Surprisingly • Two sides presented (both to be resolved) • Eliminate Out-of-scope choices • Choices are taken as TRUE, no matter howsoever absurd / ridiculous / counter- intuitive…
  • 112. © Sandeep Gupta – Ivy-GMAT, Koramangala Bangalore. sandeepgupta01@gmail.com, 9739561394 Terms generally used… • Resolve • Explain • Reconcile • Justify • Paradox • Discrepancy • Contradiction • Conflict • Puzzle
  • 113. © Sandeep Gupta – Ivy-GMAT, Koramangala Bangalore. sandeepgupta01@gmail.com, 9739561394 Paradox Questions 1. An anti-theft device is known to reduce theft, but cars using the antitheft device are stolen at a higher rate than cars without the device. 2. A surgeon has a low success rate while operating, but the director of the hospital claims the surgeon is the best on the staff. 3. Country X contains many rivers that flow down from its high mountains. These rivers have been dammed to harness the hydroelectric power that can be derived from this resource. More than enough power is generated from these dams to meet the country's energy needs. Yet, citizens of Country X often experience power shortages or even outages.
  • 114. © Sandeep Gupta – Ivy-GMAT, Koramangala Bangalore. sandeepgupta01@gmail.com, 9739561394 Paradox Questions 4. Some communities in Florida are populated almost exclusively by retired people and contain few, if any, families with small children. Yet these communities are home to thriving businesses specializing in the rental of furniture for infants and small children.
  • 115. © Sandeep Gupta – Ivy-GMAT, Koramangala Bangalore. sandeepgupta01@gmail.com, 9739561394 1. Calories consumed in excess of those with which the body needs to be provided to maintain its weight are normally stored as fat and the body gains weight. Alcoholic beverages are laden with calories. However, those people who regularly drink two or three alcoholic beverages a day and thereby exceed the caloric intake necessary to maintain their weight do not in general gain weight. Which one of the following, if true, most helps to resolve the apparent discrepancy? A. Some people who regularly drink two or three alcoholic beverages a day avoid exceeding the caloric intake necessary to maintain their weight by decreasing caloric intake from other sources. B. Excess calories consumed by people who regularly drink two or three alcoholic beverages a day tend to be dissipated as heat. C. Some people who do not drink alcoholic beverages but who eat high- calorie foods do not gain weight. D. Many people who regularly drink more than three alcoholic beverages a day do not gain weight. E. Some people who take in fewer calories than are normally necessary to maintain their weight do not lose weight.
  • 116. © Sandeep Gupta – Ivy-GMAT, Koramangala Bangalore. sandeepgupta01@gmail.com, 9739561394 2. In Asia, where palm trees are non-native, the trees’ flowers have traditionally been pollinated by hand, which has kept palm fruit productivity unnaturally low. When weevils known to be efficient pollinators of palm flowers were introduced into Asia in 1980, palm fruit productivity increased-by up to fifty percent in some areas-but then decreased sharply in 1984. Which of the following statements, if true, would best explain the 1984 decrease in productivity? A. Prices for palm fruit fell between 1980 and 1984 following the rise in production and a concurrent fall in demand. B. Imported trees are often more productive than native trees because the imported ones have left behind their pests and diseases in their native lands. C. Rapid increases in productivity tend to deplete trees of nutrients needed for the development of the fruit-producing female flowers. D. The weevil population in Asia remained at approximately the same level between 1980 and 1984. E. Prior to 1980 another species of insect pollinated the Asian palm trees, but not as efficiently as the species of weevil that was introduced in 1980.
  • 117. © Sandeep Gupta – Ivy-GMAT, Koramangala Bangalore. sandeepgupta01@gmail.com, 9739561394 3. Provinces and states with stringent car safety requirements, including required use of seat belts and annual safety inspections, have on average higher rates of accidents per kilometer driven than do provinces and states with less stringent requirements. Nevertheless, most highway safety experts agree that more stringent requirements do reduce accident rates. Which one of the following, if true, most helps to reconcile the safety experts’ belief with the apparently contrary evidence described above? A. Annual safety inspections ensure that car tires are replaced before they grow old. B. Drivers often become overconfident after their cars have passed a thorough safety inspection. C. The roads in provinces and states with stringent car safety programs are far more congested and therefore dangerous than in other provinces and states. D. Psychological studies show that drivers who regularly wear seat belts often come to think of themselves as serious drivers, which for a few people discourages reckless driving. E. Provinces and states with stringent car safety requirements have, on average, many more kilometers of roads then do other provinces and states.
  • 118. © Sandeep Gupta – Ivy-GMAT, Koramangala Bangalore. sandeepgupta01@gmail.com, 9739561394 4. Industry experts expect improvements in job safety training to lead to safer work environments. A recent survey indicated, however, that for manufacturers who improved job safety training during the 1980s, the number of on-the-job accidents tended to increase in the months immediately following the changes in the training programs. Which one of the following, if true, most helps to resolve the apparent discrepancy in the passage above? A. A similar survey found that the number of on- the-job accidents remained constant after job safety training in the transportation sector was improved. B. Manufacturers tend to improve their job safety training only when they are increasing the size of their workforce. C. Manufacturers tend to improve job safety training only after they have noticed that the number of on-the-job accidents has increased. D. It is likely that the increase in the number of on-the-job accidents experienced by many companies was not merely a random fluctuation. E. Significant safety measures, such as protective equipment and government safety inspections, were in place well before the improvements in job safety training.
  • 119. © Sandeep Gupta – Ivy-GMAT, Koramangala Bangalore. sandeepgupta01@gmail.com, 9739561394 5. Cigarette companies claim that manufacturing both low- and high- nicotine cigarettes allows smokers to choose how much nicotine they want. However, a recent study has shown that the levels of nicotine found in the blood of smokers who smoke one pack of cigarettes per day are identical at the end of a day’s worth of smoking, whatever the level of nicotine in the cigarettes they smoke. Which one of the following, if true, most helps to explain the finding of the nicotine study? A. Blood cannot absorb more nicotine per day than that found in the smoke from a package of the lowest-nicotine cigarettes available. B. Smokers of the lowest-nicotine cigarettes available generally smoke more cigarettes per day than smokers of high-nicotine cigarettes. C. Most nicotine is absorbed into the blood of a smoker even if it is delivered in smaller quantities. D. The level of tar in cigarettes is higher in low-nicotine cigarettes than it is in some high-nicotine cigarettes. E. When taking in nicotine by smoking cigarettes is discontinued, the level of nicotine in the blood decreases steadily.

Editor's Notes

  1. All Facts: It is a fact set. No conclusion.
  2. Journalist: A free marketplace of ideas ensures that all ideas get a fair hearing. Even ideas tainted with prejudice and malice can prompt beneficial outcomes. In most countries, however, the government is responsible for over half the information released to the public through all media. For this reason, the power of governments over information needs to be curtailed. Everyone grants that governments should not suppress free expression, yet governments continue to construct near monopolies on the publication and dissemination of enormous amounts of information.
  3. Last month OCF, Inc., announced what it described as a unique new product: an adjustable computer workstation. Three days later ErgoTech unveiled an almost identical product. The two companies claim that the similarities are coincidental and occurred because the designers independently reached the same solution to the same problem. The similarities are too fundamental to be mere coincidence, however. The two products not only look alike, but they also work alike. Both are oddly shaped with identically placed control panels with the same types of controls. Both allow the same types of adjustments and the same types of optional enhancements.
  4. Conclusion: This characteristic [altering the environment] is actually quite common FACT: It is well known that many species adapt to their environment, but it is usually assumed that only the most highly evolved species alter their environment in ways that aid their own survival. FACT: Certain species of plankton, for example, generate a gas that is converted in the atmosphere into particles of sulfate. FACT: These particles cause water vapor to condense, thus forming clouds. FACT: Indeed, the formation of clouds over the ocean largely depends on the presence of these particles. FACT: More cloud cover means more sunlight is reflected, and so the Earth absorbs less heat. SUB-CONCLUSION: Thus plankton cause the surface of the Earth to be cooler and this benefits the plankton. The main conclusion is actually the second sentence. There is another conclusion in the argument, in the last sentence, but this is a sub-conclusion. This sub-conclusion appears in the plankton example, and like all examples, it is used to illustrate the main conclusion.
  5. FACT: Rain-soaked soil contains less oxygen than does drier soil. FACT: The roots of melon plants perform less efficiently under the low-oxygen conditions present in rain-soaked soil.  FACT: When the efficiency of melon roots is impaired, the roots do not supply sufficient amounts of the proper nutrients for the plants to perform photosynthesis at their usual levels. SUB-CONCLUSION: It follows that melon plants have a lower-than-usual rate of photosynthesis when their roots are in rain-soaked soil.  FACT: When the photosynthesis of the plants slows, sugar stored in the fruits is drawn off to supply the plants with energy.  MAIN CONCLUSION: Therefore, ripe melons harvested after a prolonged period of heavy rain should be less sweet than other ripe melons. If there are 2 conclusions, there will be one main and one sub-conclusion. Now this is a flowing argument so there is no statement that opposes any other statement. Also, let’s analyze each choice one-by-one: The first states the conclusion of the argument as a whole ; the second provides support for that conclusion. WRONG. The conclusion is the last statement, not the first. The first provides support for the conclusion of the argument as a whole; the second provides evidence that supports an objection to that conclusion. The word objection makes it wrong. The first provides support for an intermediate conclusion that supports a further conclusion stated in the argument; the second states that intermediate conclusion. – CORRECT The first serves as an intermediate conclusion that supports a further conclusion stated in the argument; the second states the position that the argument as a whole opposes . The word “opposes” makes it wrong. The first states the position that the argument as a whole opposes ; the second supports the conclusion of the argument. The word “opposes” makes it wrong.
  6. A – wrong: may be there are other conditions. Nationality, citizenship, crime record etc. B – wrong: data only about males. Can’t generalize. C – wrong: He may be above 18. Not 18 may be below 18 or above 18. D – wrong: He may be above 18. Not just 18. ANS. E – Right: Translation of the given statement
  7. A – wrong: Not 18 does not mean less than 18. May be more than 18 too. B – wrong: above 18 allowed does not mean below 18 not allowed. C – wrong: above 18 allowed does not mean below 18 not allowed. D – wrong: above 18 allowed does not mean below 18 not allowed. E – wrong: Every male above 18 is allowed to vote in county X, means males below 18 may also be allowed; but we don’t know how the eligibility up to 18 is decided. It is possible that males below 18 may vote only under special conditions etc. so age may not be the only criterion.
  8. Only statement 3 is correct. So if X —>Y, the only valid conclusion is Y (not) —> X (not)
  9. The correct answer choice is (C). The last sentence contains a conclusion, and this conclusion is the primary evidence that supports answer choice (C). Answer choice (A): The word “often” in the first sentence is the key to this answer choice. “Often” means frequently, but frequently is not the same as “most.” Had the stimulus said “more often than not,” that would mean “most” and this answer choice would be correct. Answer choice (B): We cannot determine if readers of mystery stories solve the mystery simply by spotting the errors of the dull companion. Answer choice (C): This is the correct answer. The second sentence indicates that “clues are presented in the story...the detective uses to deduce the correct solution.” Combined with the last sentence, which states “the author’s strategy...gives readers a chance to solve the mystery,” this answer choice is proven by facts. Answer choice (D): Look for the facts in the stimulus—do they support this answer? Although the dull companion diverts readers from the correct solution, we do not know if actions of the brilliant detective rarely divert readers from the actions of the dull companion. Answer choice (E): This is a tricky answer choice if you do not read closely. The stimulus states that the dull companion infers a wrong solution from clues that the brilliant detective ultimately uses to solve the mystery. Answer choice (E) states that the dull companion uncovers misleading clues. This is incorrect; the interpretation of the clues is misleading, not the clues themselves.
  10. The correct answer choice is (B). The passage tells the story of recent physics theories: Newtonian physics was preeminent for over two centuries, and despite widespread acknowledgment and confirmation it was surpassed by Einsteinian physics in the early 1900s. Answer choice (A): The two theories cited in the stimulus are not sufficient to form a pattern, which is the basis of answer choice (A). Answer choice (B): This is the correct answer. As shown by the case of Newtonian physics, success and substantiation is no guarantee of dominance. Answer choice (C): This is an exaggerated answer that takes one instance and exaggerates it into a pattern. Although Newtonian physics was surpassed, this does not prove that every theory of physics will be eventually surpassed. The answer goes farther than the facts of the stimulus and fails the Fact Test. Answer choice (D): Like answer choice (C), this answer goes too far. Although some theories of physics have been dominant for centuries, there is no guarantee that every theory will be dominant for that long. Answer choice (E): Even though Einsteinian physics has enjoyed wide success in surpassing Newtonian physics, nowhere in the stimulus is there evidence to prove that each theory must be surpassed by an equally successful theory.
  11. Ans. E In CR, A → B means B NOT → A NOT. Sentence 1: SFP—> H NOT so we must have H —> SFP NOT.
  12. The cost of flour from the local mill is higher than the cost of the flour from the out-of-state mill. However, when purchasing from the out-of-state mill, Barry’s Bagels must pay shipping and handling fees that would not apply to a purchase from the local mill. Purchasing the flour from the out-of-state mill will only be cheaper if those shipping and handling fees are smaller than the difference in the flour costs of the two suppliers. Also, we cannot assume any additional information or move beyond the scope of the given premises in order to find the conclusion. (A) Lower production costs could explain the lower price of the flour from Isadore’s Interstate Mill, but there may be a variety of other reasons. We cannot state this conclusively. (B) It is possible that the number of local flour mill jobs would be decreased, but no evidence in the passage leads to that conclusion. (C) CORRECT. This statement properly identifies the point that, for ordering from an out-of-state mill to be less expensive, the shipping and handling fees must be less than the difference in the flour costs of the two suppliers. Say, for example, that a batch of flour costs $100 from Larry’s Local Mill. The passage tells us that the same batch would cost $90 from Isadore’s Interstate Mill, yet when purchasing from Isadore’s, shipping and handling fees would apply. We are told that Isadore’s total cost is cheaper than Larry’s, so mathematically that is: $90 + Shipping & Handling < $100, which means that Shipping & Handling < $10 = 10% of the cost of flour from Larry’s. (D) If shipping and handling fees were more than 10 percent, purchasing from the out-of-state supplier would be more expensive, not less. (E) Higher efficiency could explain the lower price of the flour from Isadore’s Interstate Mill, but there may be a variety of other reasons. We cannot state this conclusively.
  13. The argument explains that the new “Click It or Ticket” law is generating controversy. Under the new law, drivers can be cited for not wearing their seat belts, even in the absence of an additional driving infraction. Any acceptable inference must be directly supported by evidence from the text. (A) CORRECT. The entire controversy is based on the new law that allows motorists to be cited, even in the absence of an additional infraction. Thus, it follows that prior to the passage of this law, an additional driving infraction must have been necessary in order to stop and cite an individual for not wearing a seat belt. (B) Search and seizure laws are never mentioned in the text. This answer choice is outside the scope of the argument. (C) Laws in other states are never mentioned in the text. This answer choice is outside the scope of the argument. (D) Though the text states that the new regulation might save countless additional lives, the effectiveness of the previous laws are never mentioned. (E) No preference is stated between law enforcement groups and the citizens' groups. This answer choice is simply an opinion that is unsubstantiated by the text.
  14. If the defense system designers did not plan for the contingency of large meteorite explosions, such explosions would, from the system’s perspective, be unexpected. The system’s response to such explosions is consequently unpredictable. E expresses this inference and is thus the best answer. A cannot be inferred since it is consistent with the stated information that no meteorite explosion will occur within a century. B cannot be inferred since there is no information to suggest that meteorite explosions in the atmosphere would destroy the system. C cannot be inferred since it is consistent with the stated information that an appropriately designed nuclear defense system might be able to distinguish nuclear from meteorite explosions. D cannot be inferred since there is no information to suggest that the location of blasts would determine the appropriateness of defense system’s response.
  15. ANS. E Since the laws are more effective in countries farther from the equator than the United States, the laws would probably do less to prevent collisions in the United States than they do in the countries that now have such laws—countries that are all farther from the equator than the United States. So choice E is the best answer. The passage does not indicate that the use of headlights during the day is totally ineffective, so choice A is incorrect. No information is given about the importance of daylight visibility relative to other causes of collisions, so choice B is incorrect. The passage contains no quantitative information for comparing the United States to countries that have the laws, so neither C nor D is correct.
  16. ANS. A Farmers benefit from governmental price supports only when they produce the same crops from year to year. Farmers who wish to receive the benefit of these price supports will be unlikely to reduce water pollution because they will not follow the experts’ advice regarding diversification and rotation. Thus, A is the best answer. Since the experts’ advice is evidently their favored solution, the notion that the sole solution is something else (choice B) is not supported. The statements mention neither farmers’ cost and revenues nor developments in farming techniques, and thus support no conclusions about prospects for profits (choice C) or future farming techniques (choice D). Because no information is given about either the amount of price support or farmers’ debt, choice E is not supported.
  17. ANS. B The passage explains that the primary way hospitals have covered the cost of unreimbursed care in the past is no longer available to them. It follows that they have three options: finding a new way to cover that cost, reducing it by giving less unreimbused care, or suffering a loss. This is essentially what choice B concludes, so B is the best answer. The passage touches neither on kinds of medical procedures administered in hospitals (choice A) nor on revenue other than that received from patients or their insurers (choice E), so neither choice is correct. The passage gives no hint of who the paying patients are how do not rely on insurance, so choice C is unsupported. Concerning choice D, the passage actually suggests that it is false.
  18. A is the best answer. If applicants who are in fact dishonest claimed to be honest, the survey results would show a smaller proportion of dishonest applicants than actually exists. Therefore, this choice is the best answer. B is inappropriate because generally honest applicants who claimed to be dishonest could contribute to the overestimation, but not to the underestimation, of dishonest applicants. D is inappropriate because applicants who admitted their dishonesty would not contribute to an underestimation of the proportion of dishonest applicants. C and E are inappropriate because the argument is concerned neither with degrees of dishonesty nor with the honesty of non-applicants.
  19. ANS. C Analysis: Conclusion is: it is likely that some of the fragments were at least large enough to have passed through Jupiter’s outer atmosphere without being burned up – giving indication of the size of the fragments! 1st part is a consideration that the author is using to prove something so it is either C or D. 2nd part  is definitely not a conclusion. Hence, C is better.
  20. ANS. C Conclusion: “However, these environmentalists are probably wrong (about their assessment of Tropicorp’s intent).”  It’s not one of the bolded phrases, so…  Back to the boldfaced phrases to determine their relationship to the main conclusion: Boldface 1: Main conclusion of the argument says, essentially, that the environmentalists are wrong in thinking that Tropicorp’s decision not to pursue rubber tapping and, thereby, leave the forests intact INDICATES that Tropicorp is not entirely profit-motivated. In other words, the main conclusion disagrees with the environmentalists’ assessment of Tropicorp’s motives. So, that means that the first BF is reasoning that supports the position that the argument opposes. Boldface 2: This is the environmentalists’ conclusion, which means that it is the conclusion which the argument refutes. This is the easier of the two Boldface roles to work with first so C is the only choice that offers this option so C is the answer. Return to the answer choices and consider the first BF description in remaining choices. Once again, C is the only choice that offers the option for which we are searching.
  21. The author concludes that raising the fine to $1,000 would have the unintended effect of increasing the amount of litter in the picnic area. When determining the function of the two bold statements, we must consider how they relate to this conclusion: the first bold portion weighs against the conclusion, while the second bold portion supports the conclusion. The correct answer will represent these relationships. (A) The “prediction” mentioned here refers to the author’s conclusion (raising the fine to $1,000 would increase the amount of litter). This answer choice incorrectly states that the first bold portion supports this conclusion. Also, this choice incorrectly states that the second bold statement is the prediction, or conclusion. (B) This choice incorrectly states that the author’s prediction, or conclusion, is consistent with the first bold statement when in fact it predicts the exact opposite outcome. Further, this answer states that the second bold portion weighs against the author’s conclusion when in fact it supports the conclusion. (C) The second bold portion does not come as a consequence of the first. In fact, the two bold portions are in complete contrast to one another. (D) The second bold portion is not the main position that the author defends. The main position is that raising the fine to $1,000 would increase the amount of litter in the picnic area. (E) CORRECT. This answer choice correctly identifies the first bold portion as a statement of causation that does not support the author’s claim, and the second bold statement as a line of logic that does support this claim.
  22. In the passage, the conclusion advocate argues for a certain position: …. eliminating the state requirement that legal advertisements must specify fees for specific services would almost certainly increase rather than further reduce consumers’ legal costs. What follows the statement is preceded by two concessions that, the advocate admits, tend to point in the opposite direction; what follows the statement of the position are the reasons the advocate has for holding that position. To answer the question, you must find the choice that correctly describes the roles played by both of the portions that are in boldface. Choice C is the correct answer. The first boldface portion does present a pattern of cause and effect, and the advocate’s prediction is that his time the pattern will be different. In addition, the second boldface portion is one of the considerations that the advocate uses in support of that prediction. While the description of the first boldface portion given in choice A is correct, that of the second is not: the generalization in fact tends to run counter to the prediction made in the second boldface portion. Therefore this choice is incorrect. Choice B is incorrect, since although the first boldface portion presents a pattern of cause and effect, the advocate’s prediction is that in this case that pattern will not hold. Thus the role of the first boldface portion is incorrectly described. Choice D is incorrect: the advocate odes not use the first boldface portion in support of any prediction and instead concedes that it runs counter to the advocate’s own prediction. While the role of the first boldface portion is correctly described in choice E, that of the second is not, since the position the advocate is defending is not the second boldface portion, but rather the position identified above. Thus this choice is incorrect.
  23. The author explains that devoted gamers traditionally dictate the design of video games. However, due to changes in the market, the author argues that this system is no longer in the best interest of the industry. Instead, to infuse new life into the video game market, manufacturers should simplify their games in order to attract non-gamers into the gaming fold. (A) The first boldface portion does relate a situation that the author believes to be true. The second boldface portion, however, does not explain this situation; instead, it offers evidence to demonstrate why this situation should not continue. (B) CORRECT. The first boldface portion is a situation that the author believes to be true now. Due to changes in the market, however, the author believes this situation should not continue. The second boldface portion provides evidence to support the author’s contention that the best way to grow the gaming market is to attract new gamers. (C) The first boldface portion is a statement of fact that contradicts the author’s position. The second boldface portion, however, provides evidence to support the author’s position, but it is not the position itself. (D) The first boldface portion is a statement of fact that contradicts the author’s position. The second boldface portion provides evidence to support the author’s position. (E) The first boldface portion is not a prediction; rather, it is a statement of fact (or description of a situation) that the author believes should not hold in this case. The second statement is not an assumption, nor does it weigh against the author's position; instead, it is a premise that provides evidence in support of the author’s position.
  24. Assumption by the GAP FILL model: Successful models must be 5’6” or taller. Assumptions: 1. Jack is a male. 2. Jack is above 18. Assumption This man will not be able to get any other job. Assumption John is inexperienced.
  25. Kelly is unable to lose 40 pounds weight in 2 months in any other way.
  26. Conclusion: A causes B or Age results in caution. So as per the first possible answer, the assumption can be: B does not cause A OR Increased caution does not enable antelope to live longer. For C does not cause B, we have to assume that there is nothing else that caused more caution in older antelope. Here, let’s say all the older (and only older) antelope were injected with a caution-increasing drug just a day prior to the research. So then the real reason will be the INJECTION, not old age. So the assumption will be C (in this case INJECTION) was not responsible for the increased caution.
  27. There can be a lot of assumptions in this: 1. His metal detector is capable of detecting treasures hidden beneath the sand. 2. There are hidden treasures in the sand. 3. Local pawn brokers will buy treasures from him. If we negate any of these, the argument is weakened.
  28. “ Many forgone pleasures” is a new element that appears only in the conclusion. Only answer choices (B), (C), and (D) contain “forgone pleasures,” and only answer choice (D) contains “many.” Thus, if forced to make a quick decision, answer choice (D) would be the best selection at this point in our analysis. And, fortunately, the technique is so powerful that this analysis does indeed yield the correct answer. “ Should not have (been) desired in the first place” appears in both the premise and the conclusion. This element is not likely to appear in the correct answer choice. “ Justifiably regretted” appears in the premise but not the conclusion. Only answer choices (B) and (D) contain “justifiably regretted.”
  29. A quick mechanistic analysis reveals that the correct answer should contain “anger” and “visceral emotion.” Only answer choice (B) contains these two elements, and as it turns out, (B) is correct.
  30. The official argues that prohibiting high-level government officials from accepting positions as lobbyists for three years would prevent the officials from earning a livelihood for that period. The reasoning tacitly excludes the possibility of such officials earning a living through work other than lobbying. Therefore, D, which expresses this tacit assumption, is the best answer. The official’s argument does not depend on the assumption in A, since the argument would not be invalidated if some restrictions on the behavior of government officials were desirable. The official’s argument does not depend on the assumption in B, since the argument would not be invalidated if lobbyists were not typically former high-level government officials. The official’s argument does not depend on the assumption in C, since the argument would not be invalidated if former low-level government officials did often become lobbyists. The official’s argument does not depend on the assumption in E, since the argument would not be invalidated if former high-level government officials could act as lobbyists indefinitely.
  31. On the basis of an observed correlation between arms testing and people’s tendency to save money, the argument concludes that there is a causal connection between a perception of threat and the tendency not to save. That connection cannot be made unless C, linking the perception of threat to the amount of testing being done, is assumed to be true. Therefore, C is the best answer. The conclusion does not depend on there having been an increase in the perceived threat over time or on how many people supported the development of nuclear arms. Hence, neither of A and B is assumed. The argument does not deal with those who supported arms limitations or with the availability of consumer goods. Thus, D and E are not assumed.
  32. The first sentence concludes that prohibiting private planes that are not radar-equipped from centrally located airports would force most private planes away from those airports. This conclusion cannot be true unless it is true that, as C says, most private planes that use these airports are not radar-equipped. Therefore, the first sentence’s conclusion assumes this choice, which is thus the best answer. The conclusion need not assume that outlying airfields are convenient for private planes (A), since the restrictions would give planes that are not radar equipped no choice. The conclusion concerns only how the radar requirement would affect the volume of private plane traffic, so B, D and E, which deal with commercial planes and with risk of midair collision, need not be assumed.
  33. The argument concludes that cabinet-making is not an art because cabinetmakers must consider the practical utility of their products. If it is true that an object is not a work of art if its maker pays attention to the object’s practical utility, as choice D says, the conclusion is supported. Thus, choice D is the best answer. The argument is concerned with whether or not the cabinetmakers must take the practical utility of their products into consideration, not with either their monetary value (choice E) or what actually happens to them (choice A). The argument is not concerned with precise degree to which individual cabinetmakers take the practical utility of cabinets into consideration. Thus, neither B nor C is appropriate.
  34. If top managers are not the more effective decision makers, then the fact that they use intuition more often than lower-level managers does not support the conclusion that intuition is more effective. Because the argument must assume E, choice E is the best answer. To the extent that less effective methods are inappropriate, the passage does not assume A, but argues for it. Since the argument leaves open the possibility of situations in which top managers are unable to use one of the methods, choice B is inappropriate. Since the ease with which a method is implemented is not at issue, choice C is inappropriate. The argument is consistent with managers at all levels using intuition in the minority of decisions made. Thus, choice D is inappropriate.
  35. The last sentence contains the conclusion of the argument. ERE = Effectively reduce emissions, Replace = Replace the conventional diesel fuel and gasoline used in automobiles with cleaner-burning fuels, such as methanol, that create fewer emissions. ERE → Replace . According to the author, to effectively reduce emissions, conventional fuels must be replaced. Answer choice (A): This is the correct answer. Consider the author’s position that the only effective way to reduce emissions is fuel replacement. Wouldn’t it be more effective to simply stop using cars altogether? Use the Assumption Negation Technique and ask yourself if the following statement would undermine the argument: Reducing the use of automobiles would be a more effective means to reduce automobile emissions than the use of methanol. This answer clearly shows that there are other, more effective ways of reducing emissions and therefore the answer attacks the argument. This must be the correct answer. Answer choice (B): The author is not committed to methanol because the stimulus clearly references “fuels such as methanol.” Accordingly, this answer is not an assumption of the argument. Answer choice (C): The choices automobile owners would make are not part of the argument made by the author. The author simply states that the only effective way to achieve reduced emissions is fuel replacement. No assumption is made about whether automobile owners would follow that way. The problem with the answer can be highlighted by this example: An argument is made that the best way to achieve long-lasting fame is to commit suicide. Does the author assume that people will or will not choose that path? No, because the best way does not involve an assumption about how people will actually act. Answer choice (D): The author clearly believes that automobile emissions are a serious threat to the environment, but this does not mean that the author has assumed they are the most serious threat. Negate the answer and ask yourself, “What would the author say to the negation?” The author would reply that he or she never indicated that emissions were the most serious threat, so it is fine that they are not. Answer choice (E): We know that both urban air and the global atmosphere are contaminated by cars, but the author does not indicate that there is a direct correlation between the two. This answer, when negated, has no effect on the argument (and must therefore be incorrect). In an Assumption question, there can be only one answer that will hurt the argument when negated. If you negate the answers and think that two or more hurt the argument, you have made a mistake.
  36. The basic structure of this argument is fact that “mold is almost always found in places where there is substantial moisture,” so therefore, to avoid mold and the resultant mold poisoning, then people should take steps to prevent wet areas. This argument assumes that wet areas occur first, causing mold to grow. Conversely, this assumption requires that the mold growth itself does not occur first, creating wet areas as a result. (A) CORRECT. The argument depends on the assumption that the reason mold and wetness are observed together is that wet areas cause mold growth. If the reverse causation (mold causes wetness) were true, then keeping all plumbing in good condition to prevent leakage would do little to prevent the growth of mold. This choice eliminates the alternate causation. (B) If most homeowners know enough about plumbing to determine whether theirs is in good condition, then the recommendation made in this argument would be more useful. However, this is not an assumption on which the argument depends. (C) Even if mold could grow in dry areas, the fact that mold is almost always found in wet areas is still valid. This is the fact upon which the argument is based, so the argument does not depend on the unnecessarily absolute assertion that mold cannot grow in dry areas. (D) Even if some varieties of mold are harmless, the conclusion of this argument, that “one should make sure to keep all internal plumbing in good condition to prevent leakage” and minimize mold growth, could still be valid. Therefore, this argument does not depend on the unnecessarily absolute assertion that no varieties of mold are harmless. (E) Whether mold spores can be filtered from the air may be relevant to a conclusion about the health effects of mold in the home, but it is not directly relevant to this conclusion, that “one should make sure to keep all internal plumbing in good condition to prevent leakage” and minimize mold growth.
  37. Although the premises of this argument suggest only a correlation between smoking and anxiety or nervousness, the argument has a causal conclusion: it concludes that smoking causes individuals to be anxious and nervous (i.e., that A causes B). Any assumption in a causal argument must support the causal “direction” of the conclusion, that A causes B as opposed to some other explanation. Often, assumptions support a causal conclusion either by eliminating an alternate cause for the conclusion (that C did not cause B) or by demonstrating that the causation, if one exists, is in the proper direction (that B did not cause A). (A) The argument concludes that smoking causes anxiety and nervousness. Whether these maladies lead to more serious health problems is not relevant to the conclusion. (B) CORRECT. For smoking to be the cause of anxiety and nervousness (i.e., that A caused B) it must be true that these individuals were not more likely to be anxious and nervous before they started smoking. If smokers had these preconditions, which contributed to their decision to begin smoking (i.e., that B caused A), our conclusion – that smoking causes these maladies – would be incorrect. (C) The argument concludes that smoking causes anxiety and nervousness. The number of survey respondents is not relevant to the conclusion. (D) The argument concludes that smoking causes anxiety and nervousness. The awareness of the health problems related to smoking is not relevant to the conclusion. (E) The argument is not based on the immediate impact that smoking has on anxiety and nervousness. Moreover, the argument never compares some smokers to other smokers.
  38. The researcher concludes from the association of low immune-system activity with low mental-health sores that, in effect, immune system activity can inhibit mental illness. If, contrary to D, mental illness can depress immune-system activity, the association mentioned does not support the researcher’s conclusion. So D must be assumed. Normal immune-system activity could protect against mental illness without high-immune system activity offering increased protection or prevention, contrary to what A and C state, so neither of A and C is assumed. The conclusion does not depend on there being a similarity between mental and physical illness or a difference in treatments, so B and E are not assumed.
  39. If choice E were not assumed, the costs of the services of the famous singers of well-known renditions of songs would not be said to affect advertising costs. Since advertising costs are, however, projected to rise because of the relatively high cost of famous singers’ services, choice E is assumed and is the best answer. Choice A is irrelevant to the argument, since famous singers’ service cost more than imitators’ anyway. The argument addresses commercials’ cost, not their effectiveness, so choice B is not assumed. The argument assumes that some well-known renditions of songs are available, but does not require that any versions be unavailable (choice C). Since the argument states that advertising firms will stop using imitators, choice D is not assumed.
  40. The argument concludes that rising sea levels caused by global warming will destroy major coastal population centers and displace millions of people. Any assumption in support of this conclusion would have to corroborate that these events will definitively take place. (A) CORRECT. If new technological developments in the next century allow people to divert rising seas from the world’s cities (i.e., population centers), cities will not be destroyed and millions of people will not be displaced. Thus, a necessary assumption is that these technologies will not be developed. (B) A simple awareness of the steps to reduce emissions in no way undermines the argument’s conclusion, as this answer choice does not describe any action being taken by individuals. Additionally, greenhouse gases are never mentioned as the primary by-product of human activity that causes global warming, and are therefore not sufficient to address the argument. (C) The argument never suggests that all coastal population centers are similarly affected; this choice is too extreme and overreaching for the argument’s conclusion. (D) This might be true, but it is not an assumption on which the conclusion rests. Instead, this answer choice is simply an inference that might be drawn from the premises. (E) The idea that human activity is the sole cause of global warming is neither suggested nor assumed by the argument. In addition, the wording "sole cause" is too extreme.
  41. The conclusion of the argument is that the media are wrong in saying that the economy is entering a phase of growth and prosperity. The basis for that claim is that the number of people filing for bankruptcy has increased every month for the last six months and that bankruptcy lawyers are busier than they have been in years. In order for this argument to be valid, however, the author has to assume that the increase in the number of bankruptcies is a result of the state of the economy and not the result of something unrelated. (A) This statement does not have to be true for the claim that the media are wrong about the economy to hold. Even if unemployment rates are useful indicators of growth and prosperity, the media could still be wrong about the economy (e.g., if there are other indicators that show problems in other areas). (B) This does not have to be true for the conclusion to hold. Productivity could be a good measure of economic growth, but the media could still be wrong about the economy (e.g., if there are other indicators that show problems in other areas). (C) CORRECT. This has to be true for the conclusion to hold. If legislation has recently been passed that makes it easier to obtain bankruptcy, then the evidence cited would be less relevant. The increased number of bankruptcies could have been the result of the easier process rather than of a poor economy. (D) This does not have to be true for the conclusion to hold. An increase in the number of bankruptcy lawyers would not explain the increase in the number of bankruptcy filings. (E) This does not have to be true for the claim that the media are wrong about the economy to hold. Even if the media did not often misrepresent the current state of economic affairs, the argument that the media are wrong might still hold.
  42. The doctor concludes that federal legislation prohibiting the sale of video games to minors would help reduce the incidence of carpal tunnel syndrome. This conclusion hinges on the assumption that the only way for adolescents to access video games is to purchase the games themeslves. (A) Majority consensus in the legislature has no bearing on whether the recommended legislation would actually help to curb carpal tunnel syndrome. (B) This argument states that "adolescents who play video games on a regular basis are three times as likely to develop carpal tunnel syndrome." Thus, the argument directly indicates that carpal tunnel syndrome does not affect all adolescents who play video games. Rather than an assumption, this answer choice is simply an inference drawn from the text. (C) The fact that adolescents can develop carpal tunnel syndrome by means other than playing video games has no bearing on whether the recommended legislation would help to curb carpal tunnel syndrome. (D) CORRECT. In order for the doctor's recommended legislation to reduce the incidence of carpal tunnel syndrome among adolescents, the prohibition from the purchase of video games must result in the actual possession of fewer video games. Thus, it must be assumed that parents will not simply purchase video games for their children. (E) The fact that video games can benefit adolescents in other ways has no bearing on whether the recommended legislation would help to curb carpal tunnel syndrome.
  43. There can be many answers: The team composition has changed. The team has been on a winning streak since then. Playing conditions (pitch etc.), will suit the team this time whereas last time they were unsuitable. The Coach has changed. Learning, not winning, is the objective of the team. The World Cup has a history that the winner of the World Cup in a particular season essentially performed badly in the previous World Cup ETC.
  44. There were only 5 public telephone booths and just 2 months back, more than 100 free telephone booths have been put up in the adjoining areas.
  45. The correct answer choice is (D) The classic error of assuming that because two events occur simultaneously that one must cause the other. D = anti-collision device, SD = sudden disappearance of key information, D → SD. Answer choice (A): This answer presents another effect of the cause, but this additional effect does not weaken the argument. To analogize this answer to the argument, imagine a scenario where a speaker concludes that playing football makes a person more prone to sustaining a leg injury. Would suggesting that playing football makes a person more prone to a head injury (another effect) undermine the first statement? No. Answer choice (B): This is an Opposite answer that supports the conclusion. By showing that the key information did not disappear prior to the appearance of the anti-collision device, the argument is strengthened because the likelihood that the device is at fault is increased. Answer choice (C): This information has no effect on determining if the device causes the information to disappear from the screen because it references an event that has yet to occur. Answer choice (D): “Show that although the effect exists, the cause did not occur.” In this instance, the effect of information disappearing from the screen occurred prior to the creation of the supposed causal agent, the anti-collision device. Answer choice (E): This answer choice has no impact on the argument. We cannot make a judgment based on the size of the airport because the argument did not mention airport size or anything directly related to airport size.
  46. C is the best answer. This choice suggests that a significant proportion of Hawaii’s population is genetically predisposed to be long lived. Since Louisianans are not necessarily so predisposed, and since the Louisianans’ children will acquire their genetic characteristics from their parents, not from their birthplace, this choice presents a reason to doubt that Hawaiian born children of native Louisianans will have an increased life expectancy. Therefore, this choice is the best answer. Because the conclusion concerns people born in Hawaii, not the average Louisianan, A does not weaken the conclusion. Because the governor’s allegation is false, it cannot affect the conclusion. D fails to weaken the conclusion because it is consistent with the information given and the conclusion about life expectancy. By suggesting that Hawaii’s environment is in one respect particularly healthy, E supports the conclusion.
  47. The passage recommends that parents participate in a tuition prepayment program as a means of decreasing the cost of their children’s future college education. If B is true, placing the funds in an interest bearing account would be more cost-effective than participating in the prepayment program. Therefore, B would be a reason for NOT participating and is the best answer. A is not clearly relevant to deciding whether to participate since the program applies to whatever public college the child might attend. C and D, by stating that tuition will increase, provide support for participating in the program. E is not clearly relevant to deciding whether to participate, since the expenses mentioned fall outside the scope of the program.
  48. The speed with which the ice on the windshield melted is attributed to the air blowing full force from the defrosting vent onto the front windshield. This explanation of B is undermined if, as B states, no attempt was made to defrost the back window and the ice on the back window melted as quickly as did the ice on the windshield. Therefore, B is the best answer. In the absence of other information, the lack of ice condensation on the side windows that is mentioned in A is irrelevant to the validity of the explanation. C might support the explanation, since the air from the defrosting vent was warm. Neither of D and E gives a reason to doubt that air from the vent caused the ice’s melting, and thus neither jeopardizes the explanation’s validity.
  49. The group’s contention suggests that animals that are shy and active at night are feared and persecute for that reason. D establishes that raccoons and owls are shy and active at night, but that they are neither feared nor persecuted. Therefore, D is the best answer. Although an increasing prevalence of bats might explain the importance of addressing people’s fear of bats, A does not address the original causes of that fear. B and E, while relevant to the rationality of people’s fear of bats, do not affect the assessment of the accuracy of the group’s contention. That bats are feared outside the United States, as C states, does not conflict with the group’s explanation for fear of bats in the United States.
  50. The conclusion of this argument is the final sentence, which contains the conclusion indicator “therefore,” and the conclusion contains a qualification that the threat of suffocation will be eliminated after the switchover is complete. The premises supporting this conclusion are that the new plastic rings will be used by all companies and that the rings disintegrate after three days’ exposure to sunlight. Personalize this argument and ask yourself—are there any holes in this argument? Yes, there are several. The most obvious is, “What if an animal becomes entangled in the new rings before they can disintegrate?” Answer choice (A): This answer does not hurt the argument because the author qualified the conclusion to account for the date of the switchover, thereby inoculating against this avenue of attack. From a personalizing standpoint, imagine what would happen if you raised this issue to the beverage company representative—he or she would simply say, “Yes, that may be the case, but I noted in my conclusion that the program would be effective once the switchover is complete.” This is an attractive answer because it raises a point that would be a difficult public relations issue to address. Regardless, this does not hurt the argument given by the beverage company representative, and that is the task at hand. Answer choice (B): This is the correct answer. Most people select answer choice (E), but as you will see, (E) is incorrect. This answer undermines the representative’s conclusion by showing that even after the switchover is complete, the threat to animals from plastic rings will persist. Note the carefully worded nature of the conclusion—the representative does not say the threat from new plastic rings will be eliminated, but rather the threat from plastic rings, which includes both old and new rings. Answer choice (C): This out-of-scope answer addresses an issue that is irrelevant to the representative’s argument. Answer choice (D): While this is nice information from a customer service standpoint (you do not want your six-pack of beer falling apart as you walk out of the store), this answer does not affect the conclusion because it does not address the threat of suffocation to animals. Answer choice (E): The conclusion is about wild animals, whereas this choice mentions aquatic animals. The conclusion is specifically about suffocation , and answer choice (E) does not address suffocation . This is the most commonly chosen answer. In this case, the answer preys upon test takers who fail to heed the advice: “Read closely and know precisely what the author said. Do not generalize!” Many test takers read the conclusion and think, “So when they start using these new rings, it will make things better for the animals.” When these test takers get to answer choice (E), the answer looks extremely attractive because it indicates that the implementation of the new rings will also have a harmful effect. With this thinking in mind, many test takers select answer choice (E) thinking it undermines the conclusion and they are certain they have nailed the question. However, the conclusion is specifically about suffocation , and answer choice (E) does not address suffocation . Instead, answer choice (E) attacks a conclusion that is similar but different from the actual conclusion. Remember, one of the rules for weakening arguments is to focus on the conclusion, and knowing the details of the conclusion is part of that focus.
  51. The principle that people are entitled to risk injury provided they do not thereby harm others fails to justify the individual’s right to decide not to wear seat belts if it can be shown, as B shows, that that decision does harm others. Therefore, B is the best answer. A suggests that the law may be irrelevant in some cases, but it does not address the issue of the law’s legitimacy. C cites a requirement analogous to the one at issue, but its existence alone does not bear on the legitimacy of the one at issue. The argument implicitly concedes that individuals take risks by not wearing seat belts; therefore, D and E, which simply confirm this concession, do not weaken the conclusion.
  52. Concluding from the similar numbers of deaths in two groups that the relative danger of death was similar for both groups is absurd if, as here, one group was far smaller. D exposes this absurdity by pointing out the need to compare death rates of the two groups, which would reveal the higher death rate for the smaller group. Therefore, D is the best answer. Since the conclusion acknowledges the difference between the number of civilian and armed forces deaths, expressing this difference as a percentage, as suggested by B, is beside the point. A is inappropriate because it simply adds a third group to the two being compared. Because cause of death in not at issue, C and E are irrelevant.
  53. The conclusion of the argument is that "there must be fewer new residents moving to City X than there were previously." Why? Because of several observed factors (e.g., developers not buying land, contractors without work, banks issuing fewer mortgages) that the author assumes result from the fewer people trying to buy new homes. We are asked to find a flaw in the reasoning of this argument. (A) CORRECT. This suggests that there might be another reason for the decline in home construction: the supply of available housing has been increased through the release of many previously built homes. Therefore, the reasoning in the argument is flawed. (B) The size of homes, by itself, does not point to any flaw in the argument. (C) The argument centers on new homes, so re-sales of condominiums are not directly related. (D) If materials cost less, it seems more likely that any decrease in new home construction could be attributed to the stated causes. (E) Sales of cars and boats are not related to construction of new homes.
  54. Last year, firms in the manufacturing sector that offered employees employer sponsored insurance (ESI) plans found that their worker absentee rates were significantly lower than at firms that did not offer ESI. What could have caused the decrease? The argument is made that the existence of the ESI plan caused the decrease in worker absenteeism, but that conclusion would be weakened if another cause were identified. (A) Similar findings in other sectors of the economy strengthen rather than weaken the argument. (B) If workers have access to preventative health care as a result of the ESI plan, they might be healthier and would miss fewer days of work due to illness. This point supports the argument. (C) The difficulty of initiating an ESI plan is irrelevant to a conclusion about what happens after the plan is established. (D) CORRECT. If there are fewer on-the-job injuries, then workers will miss fewer days of work. This is an alternate explanation for the decrease in absenteeism, and thus weakens the argument that the decrease in absenteeism must have been due to the availability of ESI. (E) The higher productivity of workers covered by ESI plans is consistent with the fact that they miss fewer days of work. This statement does not weaken the argument.
  55. Explanations for 1 and 2: Most people assume that if a percentage becomes larger, the number that corresponds to that percentage must also get larger. This is not necessarily true because the overall size of the group under discussion could get smaller. For example, consider the following argument: “Auto manufacturer X increased their United States market share from 10% last year to 25% this year. Therefore, Company X sold more cars in the United States this year than last.” This is true if the size of the U.S. car market stayed the same or became larger. But if the size of the U.S. car market decreased by enough, the argument would not be true, as in the following example: Last Year This Year Total number of cars 1000 200 sold in the United States X’s market share 10% 25% X’s total car sales 100 50 in the United States Thus, even though auto manufacturer X’s market share increased to 25%, because the size of the entire market decreased significantly, X actually sold fewer cars in the United States. Of course, if the overall total remains constant, an increasing percentage does translate into a larger number. But on the GMAT the size of the total is usually not given. If the percentage increases but the corresponding number decreases, then the overall total must have decreased. If the percentage decreases but the corresponding number increases, then the overall total must have increased. If the number increases but the corresponding percentage decreases, then the overall total must have increased. If the number decreases but the corresponding percentage increases, then the overall total must have decreased. Explanations for 3 and 4: Just as increasing percentages do not automatically translate into increasing numbers, the reverse is also true. Consider the following example: “ The number of bicycle-related accidents rose dramatically from last month to this month. Therefore, bicycle-related accidents must make up a greater percentage of all road accidents this month.” This conclusion can be true, but it does not have to be true, as shown by the following example: Last Month This Month Number of bicycle-related 10 30 accidents Total number of road accidents 100 600 Percentage of total accidents 10% 5% that are bicycle-related Thus, even though the number of bicycle-related accidents tripled, the percentage of total road accidents that were bicycle-related dropped because the total number of road accidents rose so dramatically. Explanation for # 5: In 2003, Porsche sold just over 18,000 cars in the United States. While 18,000 is certainly a large number, it represented only about 1/5 of 1% of total U.S. car sales in 2003. Remember, the size of a number does not reveal anything about the percentage that number represents unless you know something about the size of the overall total that number is drawn from. Explanation for # 6: This misconception is the reverse of Misconception #5. A figure such as 90% sounds impressively large, but if you have 90% of $5, that really isn’t too impressive, is it? Knowledge of a percentage is insufficient to allow you to make a determination about the size of the number because the exact size of the overall total is unknown, and changes in the overall total will directly affect the internal numbers and percentages.
  56. Markets and Market Share The makers of the GMAT expect you to understand the operation of markets and the concept of market share. Market operation includes supply and demand, production, pricing, and profit. None of these concepts should be unfamiliar to you as they are a part of everyday life. Market share is simply the portion of a market that a company controls. The market share can be measured either in terms of revenues (sales) or units sold. For example: Heinz has a 60% market share of the $500 million ketchup market. Jif brand peanut butter sold 80 million units last year, a 30% market share. Because market share is a numbers and percentages concept, market share can change when factors in the market change. For example, a company can gain market share (percentage) if the market shrinks and they maintain a constant size, or if they grow in an unchanging market. However, a company losing market share does not mean that their sales decreased, only that they became a smaller entity in the market relative to the whole (the market grew and they stayed the same size, for example). Similarly, a company could lose sales and still gain market share if the overall market became smaller. Regardless of the size of a market and even though the total amount of the market can shift, the total market share must always add up to 100%. Like all numbers and percentages problems, market share is a comparative term, as opposed to an absolute term.
  57. There can be many answers: The same team is slated to play in next WC. The team has been on a losing streak since then. Playing conditions (pitch etc.), will remain the same. The Coach has not changed.
  58. There can be many answers that can strengthen it: After the laptop computer was turned off, the plane regained course. No problems with navigational equipment have been reported on flights with no passenger-owned electronic devices on board.
  59. Following is the structure of the medical doctor’s argument: Premise: Sleep deprivation is the cause of many social ills, ranging from irritability to potentially dangerous instances of impaired decision making. Premise: Most people today suffer from sleep deprivation to some degree. Conclusion: Therefore we should restructure the workday to allow people flexibility in scheduling their work hours. The first premise contains a causal assertion (not a causal conclusion), and the second premise indicates that most people suffer from the stated cause. This combination would lead to the conclusion that most people have a social ill (which could be irritability or impaired decision making, or something in between). However, the conclusion in the argument leaps over this idea to conclude that the workday should be restructured. The missing link—or assumption—in the argument is that restructuring the workday would alleviate the sleep deprivation. As always, whenever you see a gap in the argument, you can strengthen the argument by eliminating that gap. By relating sleep to work, answer choice (B) closes the gap in the argument. Answer choice (A): This is a tricky answer, and the key word is “overwork.” While the author clearly believes that work schedules affect sleep, this does not mean that employees are being overworked. For example, a person may be sleep deprived because they have to come into work at 8 A.M. Perhaps they have children so they must get up very early to take care of their family. The person might then work a normal eight hour day and be sleep deprived not because of overwork but because of rising early. Answer choice (B): This is the correct answer. By indicating that employees would avoid sleep deprivation with a revised workday, this answer affirms that the leap (or gap) made in the argument is not an unreasonable one. Answer choice (C): This answer may hurt the argument by suggesting that some individuals cannot be helped by the restructuring of the workday. At best, this answer has no impact on the argument because we already know that most people suffer from sleep deprivation to some degree. Answer choice (D): This answer addresses the fact that the number of hours worked per week has decreased. But the argument is not about the average number of hours worked, but rather the way that those hours affect sleep. Thus, this answer does not help the conclusion that people should be allowed flexibility in scheduling. Answer choice (E): The argument does not suggest that the workday will be shortened, only that the day will be structured so that people have more flexibility in scheduling their hours. Thus, knowing that the extent of sleep deprivation is proportional to the length of one’s workday does not strengthen the argument.
  60. ANS. D Conclusion: Galanin causes CRAVING for FATTY FOODS ( G → CFF). In this instance, the author simply assumes that galanin is the cause. Why can’t the fatty foods lead to higher concentrations of galanin? Answer choice (A): If anything, this answer choice may hurt the argument by showing that the cravings do not always lead to choosing fatty foods. But, since the author uses the phrase “consistently chose” to describe the choices of the rats, an answer stating that rats did not “invariably” choose fatty foods has no effect on the argument. Answer choice (B): This is a Shell Game answer because the test makers try to get you to fall for an answer that addresses the wrong issue. The argument discusses the concentration of galanin in the brains of rats; no mention is made of the fat content of the brains of rats. This answer, which focuses on the fat content in the brains of rats, therefore offers no support to the argument. Even though the brain might not contain more fat, a rat could still consistently choose and eat foods with a higher fat content. Answer choice (C): The argument is that galanin in the brain causes rats to crave fatty foods. The fact that galanin is in the food does not help that assertion and may actually hurt the argument. Answer choice (D): This is the correct answer. The answer strengthens the argument by eliminating the possibility that the stated causal relationship is reversed: if the rats had higher concentrations of galanin prior to eating the fatty foods, then the fatty foods cannot be the cause of the higher concentration of galanin. As discussed earlier in the chapter, this approach strengthens the argument by making it more likely that the author had the original relationship correct. Answer choice (E): This answer choice hurts the argument by suggesting that the causal relationship in the conclusion is reversed. Remember that in Strengthen questions you can expect to see Opposite answers, and this is one.
  61. The conclusion is that a developer who wishes to make a large profit would be wise to buy urban waterfront lots and erect residential buildings on them. The basis for that claim is that people pay large sums for beach front homes. We are asked to strengthen this argument. (A) This choice states that people have more buying power today than in previous centuries. This does not strengthen the claim that a developer will make money on urban waterfront properties. (B) CORRECT. This choice states that homeowners will be willing to spend large sums of money on residential properties in traditionally industrial or commercial districts. Since we know from the argument that urban waterfronts have traditionally been industrial, this fact strengthens the claim that a developer can make a profit on urban waterfront properties. (C) This choice states that many urban waterfront lots are available for purchase. This does not suggest, however, that a developer will be able to sell them after he or she builds on them. (D) This choice states that many coastal cities are giving tax breaks to developers who rehabilitate the waterfront. But this does not suggest that anyone will buy the developed properties. (E) This choice states that properties in the interior of cities are more expensive than those on the waterfront. Although waterfront properties are therefore cheaper to acquire, this does not necessarily mean that a developer can make a profit after buying such properties.
  62. The conclusion of the argument is that humans and primates are not the only animals capable of communicating with language. The basis for this claim is that a parrot named Alex becomes upset when he is not given the gesture he verbally requests. We are asked to strengthen the claim. (A) The conclusion is about non-primates or non-humans being able to use language to communicate. Assisting divers in ocean rescues is not relevant. (B) The conclusion is about non-primates or non-humans being able to use language to communicate. Gorillas are primates, as stated in the first sentence of the argument. (C) Sensing the mood of one’s owner and exhibiting concern is not a form of language communication. (D) The conclusion is about non-primates or non-humans being able to use language to communicate. Chimpanzees are primates, as stated in the first sentence of the argument. (E) CORRECT. If Alex does not exhibit aggression when offered a gesture that he specifically requested, it suggests that Alex can tell the difference between the gestures that he requests and those that he does not. In other words, he is a non-primate / non-human but he is communicating via language. If he also exhibited aggression when offered the gestures he requested, it would be more difficult to claim that he was communicating via language.
  63. Answer: B A difficult CR question, to be sure. Here's the logic: The Vargonian government has guaranteed the availability of a free education in government-funded schools to ALL Vargonian children and set the requirement that the current student-teacher ratio must be maintained. So, from this alone, we can see that an increase in the number of students dictates the hiring of additional teachers. In B, we find that one-quarter of Vargonian children are enrolled in private, non-government-funded, schools (because the economy is so good and everyone is so rich). Even though these children are not currently enrolled in one of the government-funded schools, they retain the legal right, according to the argument, to a free education in one of these schools on-demand. One can infer that in an economic recession, many of the "rich" families might not be quite so "rich" any longer. Thus, some percentage of those students currently attending private schools might return to public (government-funded) school, under times of economic difficulty. And, as we found earlier, under the new law, more students means more teachers must be hired (to preserve the current ratio), which makes this the choice that most strengthens this argument.
  64. If B is true, the greater abundance of longevity-promoting environmental factors it mentions is probably at least partly responsible for the higher life expectancy in Hawaii. Children born in Hawaii benefit from these factors from birth, and thus Louisianans who have children in Hawaii increase their children’s chances of living longer. Therefore, B is the best answer. If life expectancy in Hawaii is likely to be falling, as A says, the argument is weakened rather than strengthened. C and E, in the absence of other relevant information, have no bearing on the conclusion; thus, they are inappropriate. D is irrelevant, because the information it mentions about rates would already have been incorporated into the statistics cited in the passage.
  65. The plan proposes that high-speed ground transportation would be a less expensive solution to airport congestion than would airport expansion. B indicates that between the cities to be served by the plan there is substantial air travel to which ground transportation would represent an alternative. Therefore, B is the best answer. No other choice could be cited appropriately. A and D both provide some evidence against the plan. A by emphasizing the likely costs of providing high-speed ground transportation is not by itself a solution to airport congestion. D by indicating that such an alternative is not by itself a solution to airport congestion. C and E say that there are many travelers for whom the proposed system would actually provide no alternative.
  66. According to the passage, satellite mishaps caused a surge in insurance claims, which, in turn, caused increased insurance premiums. Higher premiums made the satellites more costly, resulting in increased performance demands. If C is true, the greater demands on performance will lead to further increases in costs by increasing the number of mishaps, and thus pushing insurance premiums still higher. Thus, C is the best answer. A, D and E all describe factors relevant to costs, but there is no reason to think that the situation described in the passage will cause the costs resulting from these factors to increase. Similarly, the impossibility of pinpointing the cause of failure, mentioned in B, is consistent with the cost of satellites remaining stable.
  67. As we can see in this argument, the argument is inadequate (mind you inadequate, not faulty). In order for the claim to be convincing or not, we might need to know more. Here, the best thing that one can ask for is “What are the results of the testing?” Now take the results to two extremes: Extreme one: Very good: then the claim is strengthened. Extreme two: BAD, HORRIBLE QUALITY: then the claim is weakened. So the correct answer will read something like this: “What are the results of the testing?”
  68. The best question will be: Have the prices of other ingredients in chocolate decreased recently? The conclusion of the argument is that the price of chocolate will increase within six months. The basis for that claim is that the wholesale price of cocoa has increased. However, if the price of other ingredients in the chocolate has dropped, the decrease could offset the higher price of cocoa and render the argument invalid.
  69. Whether the newborns in both nurseries were equally healthy and happy at the start of the experiment….
  70. Profits for a particular product have been going down and the CFO has determined that this is because, on the one hand, the cost to make the product has increased and, on the other, consumers won't pay more than the current price (recall that Profit = Revenues – Costs). The CEO only wants Company X to sell products with increasing profit margins; as a result, the CFO decides the solution is to stop making this product. This decision would make sense only if we can be assured that there is no way to have an increasing profit margin for the product in question. (A) Whether there are new, profitable products does not address the issue of whether there is a way to achieve increasing profit margins for the flagship product. (B) Whether the management team agrees with the CFO's recommendation does not address the issue of whether there is a way to achieve increasing profit margins for the flagship product. (C) This may increase the revenues earned by the product, but this choice does not address the additional cost associated with new features, so we still do not know whether we can achieve increasing profit margins for the product. We may be able to, but we may not: the features may cost more than the increased price that consumers would be willing to pay. (D) CORRECT. If the costs for the existing product can be reduced, then the profit margin will increase (again, recall that Profits = Revenues – Costs). If the costs cannot be reduced, then the profit margins will not increase. (E) The flagship product's revenues as a percentage of total revenues does not address the issue of whether there is a way to achieve increasing profit margins for the flagship product (although it does highlight why the company might find itself with a big problem if it follows the CEO's advice!).
  71. The correct answer is B. The conclusion of the argument is that "eating whole grains can have an appreciable effect on cholesterol levels." This assertion is based on the fact that some people who ate three servings of whole grains every day for six months had lower cholesterol than did people who did not, even though their cholesterol levels were the same before the study began. The argument does not take into account, however, other factors, such as exercise, that may have contributed to the difference in cholesterol levels. Choice B asks whether there is indeed another factor – exercise – that should be taken into account.
  72. The hypothesis has two parts: first, that intense use does not bring material changes that cause the string to go dead and, second, that dirt and oil do cause the phenomenon. The experiment suggested in choice E directly tests this hypothesis by contaminating strings that are known to have their original material properties. Thus, E is best answer. Because factors associated with style of play (choice B) and brand of guitar (Choice C) might affect how the strings become contaminated, no result of the investigations in B and C will allow clear evaluation of the hypothesis. Information about the strings’ material (choice A) will need considerable supplementation before its bearing on the hypothesis is clear. The passage already gives the information promised by investigation D.
  73. The argument concludes that the prosecution of a small number of people who download music illegally will have a minimal impact on the overall number of people who engage in illegal downloading. The correct answer must relate specifically to this issue and provide additional insight as to whether it seems reasonable. (A) One premise of the argument states that the recording industry does not have the resources to prosecute all individuals who download music illegally, while a second premise states the number of people who will be charged with a crime is limited. These statements indicate that the legal resources of the recording industry are too limited to have a major impact on the overall number of people who engage in illegal downloading, no matter how these lawyers dedicate their time. (B) If a small minority of individuals were responsible for the majority of illegal song downloads, the actions of the recording industry could have a significant impact on the number of downloaded songs. The conclusion of the argument, however, was about the number of people who download songs illegally; this number would remain unaffected. (C) Whether songs are downloaded illegally and then shared with other Internet users is not relevant to the conclusion. (D) Similar to answer choice A, this choice is limited by the premises of the argument. If new Internet security technology permits the recording industry to more quickly and easily identify individuals who illegally download music, then the recording industry will know who is breaking the law. However, the lack of industry resources still restricts the industry’s ability to prosecute a large number of people, even if they are identified as individuals who illegally download music. (E) CORRECT. The argument concludes that the prosecution of a small number of people who download music illegally will have a minimal impact on the overall number of people who engage in illegal downloading. However, if the threat of prosecution were enough to “alter the behavior” of others (i.e., deter them from illegally downloading music), the actions of the recording industry could have a significant impact on the number of people who illegally download music.
  74. The correct answer choice is (B). The conclusion of the argument is the first sentence: “George Orwell’s book 1984 has exercised much influence on a great number of this newspaper’s readers.” The basis for this conclusion is that 1984 was the second most named book in a survey about influential books. The argument contains a serious error: just because 1984 came in second in the survey does not mean that “a great number” of readers selected it as influential. To illustrate this proposition, consider the following example: Number of people surveyed = 1000, Number of people naming the Bible as the most influential book = 999, Number of people naming 1984 as the most influential book = 1, In this example, 1984 has come in second, but no one would say this second place finish supports a conclusion that “1984 has exercised much influence on a great number of this newspaper’s readers.” You can expect the correct answer to address this issue. Answer choice (A): The survey in the argument asks readers to name the one book with the most influence in their lives; the number of books read does not affect this answer. To apply the Variance Test, try opposite answers of “1” and a large number, say “10,000.” These numbers will not alter the evaluation of the argument, and thus this answer is incorrect. Answer choice (B): This is the correct answer, but it can be difficult since the wording is a bit unusual. The question is intended to reveal how many people selected 1984 relative to the other choices, and this addresses the issue raised in the analysis of the stimulus. Consider how the variance test works for this answer choice: First try the response, “999.” In this case, only one person selected 1984 as the most influential book, and the argument is greatly weakened. Next try the response, “501.” In this instance, 499 people selected 1984 as the most influential book and the conclusion is strengthened (the other 501 people would have selected the Bible). Note that you cannot try a number larger than 501 because that would mean that the Bible was not named most often. Because the varied responses produce different evaluations of the argument, this answer is correct. Answer choice (C): This answer is not relevant to the columnist’s argument. Apply the Variance Test to disprove this answer by using opposite answers of “0” and a very large number, such as “1 million.” Answer choice (D): Because the argument is about Orwell’s 1984, other Orwell books chosen by the readers have no impact on the argument. Apply the Variance Test, using opposite answers of “0” and a small number such as “10” (Orwell wrote dozens of essays, but not dozens of books). Answer choice (E): The survey in the argument addresses influence, not the actual reading of the book. A person might be influenced by a book like the Bible through church teachings, etc. without actually having read the book. To apply the Variance Test, try opposite answers of “0” and “1000.”
  75. The argument presents a substantial increase in the proportion of women between twenty and twenty-one who were enrolled in college as evidence that there was an increase in the proportion of higher education students who were women. This evidence would lack force if a similar increase in college enrollment had occurred among men. Choice D is therefore the best answer. Since percentages of men graduating from high school do not indicate the percentages enrolling in college that year, choice E is incorrect. Choices A, B, and C are incorrect because the information they refer to, being about women only, does not facilitate a comparison of women’s enrollment to men’s enrollment in higher education programs.
  76. Explanation: The device is placed on highly desirable cars that are prone to being stolen, and the device actually lessens the rate at which they are stolen. Explanation: The surgeon operates on the most complex and challenging cases. Explanation: Most of the electricity generated in Country X is sold to other countries.
  77. Many residents of these communities must provide for the needs of visiting grandchildren several weeks a year.
  78. The paradox in this problem is that alcohol drinkers who surpass the threshold for calorie intake should gain weight, but they do not. Most people, upon reading the stimulus, prephrase an answer involving exercise or some other way to work off the expected weight gain. Unfortunately, a perfect match to this prephrase does not appear, and instead students are faced with a tricky answer that preys upon this general idea while at the same time it fails to meet the circumstances in the stimulus. Answer choice (A): Read closely! The stimulus specifies that people who regularly drink two or three alcoholic beverages a day thereby exceed the necessary caloric intake. This answer, which discusses individuals who avoid exceeding the caloric intake necessary, therefore, addresses a different group of people from that in the stimulus. Since information about a different group of people does not explain the situation, this answer is incorrect. This answer is attractive because it uses the idea of getting rid of or avoiding calories, but it violates one of the precepts of the stimulus. Out of scope! Remember, you must look very closely at the circumstances in the stimulus and make sure that the answer you select matches those circumstances. Answer choice (B): This is the correct answer. If the excess calories are dissipated as heat, then there would be no weight gain. Hence, alcohol drinkers can consume excess calories and still not gain weight. Some students object to this answer because the situation seems unrealistic. Can heat dissipation actually work off dozens if not hundreds of calories? According to the question stem, yes! Remember, the question stem tells you that each answer choice should be taken as true. Since this answer choice clearly states that the excess calories tend to be dissipated, you must accept that as true and then analyze what effect that would have. Answer choice (C): The stimulus discusses “people who regularly drink two or three alcoholic beverages a day and thereby exceed the caloric intake necessary.” This answer choice addresses a different group of people than those discussed in the stimulus. Out of scope! Answer choice (D): The first flaw in this answer is that it simply states that individuals consuming alcohol do not gain weight but it offers no explanation for why these people have no weight gain. The second flaw in the problem is that it addresses the wrong group of people. The stimulus discusses people who drink two or three alcoholic beverages a day; this answer addresses people who drink more than three alcoholic beverages a day. Out of scope! Answer choice (E): Again, this answer discusses a different group of people than those in the stimulus. The stimulus discusses people who exceed the necessary caloric intake; this answer addresses people who do not meet the necessary caloric intake. Out of scope!
  79. If C is true, the rapid increase in productivity among Asian palm trees after 1980 probably depleted nutrients needed for the development of fruit-producing flowers. Thus, C explains why the palms’ productivity could subsequently decline, and is the best answer. A relates a drop in the price of palm fruit to a rise in production and a fall in demand, but it does not explain the subsequent drop in the trees’ productivity. B gives no reason for the decrease in productivity of the trees introduced to Asia. D does not explain the decrease in productivity, since the stability of the weevil population described would support stability of palm fruit productivity between 1980 and 1984 rather than a decrease. Because E describes the pollination of the trees prior to 1980, it cannot explain a change occurring in 1984.
  80. The paradox in the argument is that the provinces and states that have more stringent safety requirements also have higher average rates of accidents. Even so, experts agree that the more stringent requirements actually are effective. This type of “surprisingly low/high rate of success” scenario has appeared in a number of Resolve the Paradox questions, including the following: An anti-theft device is known to reduce theft, but cars using the anti- theft device are stolen at a higher rate than cars without the device. Explanation: The device is placed on highly desirable cars that are prone to being stolen, and the device actually lessens the rate at which they are stolen. A surgeon has a low success rate while operating, but the director of the hospital claims the surgeon is the best on the staff. Explanation: The surgeon operates on the most complex and challenging cases. A bill collector has the lowest rate of success in collecting bills, but his manager claims he is the best in the field. Explanation: The bill collector is assigned the toughest cases to handle. These scenarios underscore the issue present in the question: other factors in the situation make it more difficult to be successful. With the car safety requirements, you should look for an answer that shows that there is a situation with the roads that affects the accident rates. A second possible explanation is that the seat belts are not actually used by a majority of drivers and the safety inspections are not made or are rubber-stamp certifications. This answer is less likely to appear because it is fairly obvious. Answer choice (A): The stimulus specifies that annual safety inspections— regardless of what is examined—are already in place. Therefore, this answer does not explain why the average rate of accidents is higher in those states. Answer choice (B): Assuming that overconfidence leads to accidents, the answer could support the assertion that states with more stringent requirements have higher accident rates. But, this answer would also suggest that the experts are wrong in saying that more stringent standards reduce accident rates, so this answer cannot be correct. Answer choice (C): This is the correct answer, and the answer conforms to the discussion above. If the roads are generally more dangerous, then the stringent requirements could reduce the accident rate while at the same time the accident rate could remain relatively high. Since this scenario allows all sides of the situation to be correct and it explains how the situation could occur, this is the correct answer. Answer choice (D): This answer supports only one side of the paradox. The answer confirms that the experts are correct, but it does not explain why these provinces have higher accident rates. Thus, it does not resolve the paradox. Answer choice (E): This answer appears attractive at first, but the number of miles of roadway in the provinces is irrelevant because the stimulus specifically references “accidents per kilometer driven.” Since the accident rate is calculated as per-miles-driven, the actual number of miles of roadway is irrelevant.
  81. The correct answer choice is (B). The paradox in the stimulus is: for manufacturers who improved job safety training during the 1980s there was an increase in the number of on-the-job accidents. Answer choice (A): This answer does not provide an explanation for the paradox in the stimulus. Some students eliminate this answer because it addresses the transportation industry, but information about the transportation industry could be used to analogically explain the issue in the manufacturing industry (but, to be correct the answer would have to offer some further relevant parallel between the two industries). Answer choice (B): This is the correct answer. If the workforce is increasing, more accidents would be expected. Thus, safety training could improve the safety of the work environment (as measured by average number of accidents per worker, for example) while at the same time the number of total accidents could increase. Because this answer allows both sides to be true and it explains the circumstance in the stimulus, this answer is correct. In Chapter Fifteen we will discuss average versus total numbers, and that will further explain the construction of this question. Answer choice (C): This would explain an increase in accidents before job safety training, but the issue in the stimulus is an increase after the safety training. Answer choice (D): This answer further confuses the issue. If the fluctuation was random, that could explain how an increase in accidents could follow safety training. By stating that the increase was not random, a possible cause of the scenario is eliminated. Answer choice (E): This answer shows that the level of safety was at least minimal prior to the safety training, but this does not help explain why an increase in accidents followed the training.
  82. The correct answer choice is (A). In rough terms, the paradox in the stimulus is that smokers of one pack of low-nicotine cigarettes have an identical nicotine level at the end of the day as smokers of one pack of high-nicotine cigarettes. This similarity must be explained by a similarity, not a difference. Answer choice (A): This is the correct answer. The answer choice indicates that there is a similarity in the blood such that the maximum amount of nicotine absorbed is identical for everyone. Because the maximum amount of nicotine absorbed per day is equal to the nicotine in a pack of low-nicotine cigarettes, each person absorbs the amount of nicotine equal to the low-nicotine pack regardless of the type of cigarette smoked. Additional nicotine is not absorbed into the blood of smokers of the high-nicotine brand. Since this answer explains the paradox, this is the correct answer. Answer choice (B): Read closely! The stimulus is specifically about smokers who “smoke one pack of cigarettes per day.” This answer discusses smoking different numbers of cigarettes and thus it fails to meet the circumstances in the stimulus. Answer choice (C): This answer confuses the issue because it indicates that most nicotine is absorbed into the system. From this fact one would expect that those smoking high-nicotine cigarettes would have higher nicotine levels than low-nicotine cigarette smokers. Answer choice (D): The stimulus does not address the level of tar in cigarettes, nor can we make any judgment about how tar affects nicotine levels. Answer choice (E): This would apply to any smoker, and as this addresses an effect that occurs after smoking is stopped, it does not help us understand why the nicotine rose to identical levels regardless of the kind of cigarette smoked.
  83. If the number of men beginning to smoke and the number of women quitting smoking during the year are equal, A would result in an increase, not a decrease, in the number of adults who smoke. Hence, A does NOT explain the facts cited and is the best answer. Given the decrease in the number of adults who smoke, the increase in tobacco sales could be explained by a proportionally greater increase in the non-adults who smoke or the nonsmokers who use tobacco. An increase in total tobacco use by smokers or in the sales in the United States tobacco abroad would also explain the facts cited. Thus, because B, C, D and E could explain the facts cited, none of them can be the best answer.